You are on page 1of 159

Introduction

to
Applied Analysis
Definitions, Theorems and Problems

SUBRATA PAUL

FALL - 2014

1
Contents
1. Real Number System 1
2. Basic Topology 3
3. Problems on Basic Topology 5
3.1. Rudin’s Solution 5
4. Metric Space 7
5. Problems on Metric Space 11
6. Numerical Sequences and Series 46
6.1. Sequences 46
6.2. Series 47
7. Rudin’s Solution: 49
7.1. Problems from other sources 55
8. Compactness 57
8.1. Problems 58
9. Continuity 73
9.1. Completing a Metric Space with an Isometry 74
9.2. Continuous Function on Compact Sets 74
9.3. Proofs of the Theorems 76
9.4. Problems: 77
10. Differentiation of Single Variable 93
10.1. Some Problems 94
11. Sequences and Spaces of Continuous Functions 95
11.1. Space of Continuous Functions 95
11.2. Compact Subset of Continuous Function Space 95
11.3. Proof of the Theorems: 97
11.4. Problems on Sequence of Functions 98
12. Differential Calculus and Banach Space 127
13. Problems on Differentiation 130
14. List of Problems 147
14.1. Problems in Norm and Metric 147
14.2. Cauchy and Compactness 148
14.3. Compactness 150
14.4. Continuity 151
14.5. Sequence of Functions 153
14.6. Differentiation 156
1. Real Number System

Definition 1.1. Let S be a set. An order on S is a relation, denoted by <, with the
following two properties
(i) If x ∈ S and y ∈ S then one and only one of the statements

x < y, x = y, y<x

is true.
(ii) If x, y, z ∈ S, if x < y and y < z then x < z
An order set is a set S in which an order is defined.

Definition 1.2. Suppose S is an order set, and E ⊂ S. If there exists an β ∈ S such


that x ≤ β for every x ∈ E, we say that E is bounded above, and call β an upper bound
of E. Lower bounds are defined in the same way.

Definition 1.3. Suppose S is an ordered set, E ⊂ S, and E is bounded above. Suppose


that there exists an α ∈ S with the following properties:
(i) α is an upper bound of E.
(ii) If γ < α then γ is not an upper bound of E. Then α is called the least upper
bound of E or the supremum of E, and we write,

α = sup E

The greatest lower bound, or infimum, of aset E which is bounded below is defined
in the same manner: The statement

α = inf E

means that α is a lower bound of E and that no β with β > α is a lower bound
of E.

Definition 1.4. An ordered set S is said to have the least-upper-bound property if the
following is true:

If E ⊂ S, E is not empty, and E is bounded above, then sup E exists in S.

Theorem 1.1. Suppose S is an ordered set with the least-upper-bound property, B ⊂ S,


B is nonempty, and B is bounded below. Let L be the set of all lower bounds of B. Then,

α = sup L

exists in S, and α = inf B. In particular inf B exists in S.

Theorem 1.2. There exists an ordered field R which has the least-upper-bound property.
Moreover, R contains Q as a subfield.
1
Theorem 1.3. (a) Archimedean property: If x ∈ R, y ∈ R and x > 0, then there is
a positive integer n such that

nx > y

(b) If x ∈ R, y ∈ R, and x < y, then there exists a p ∈ Q such that x < p < y

Theorem 1.4. For every real x > 0 and every integer n > 0 there is one and only one
real y such that y n = x.

2
2. Basic Topology

Definition 2.1. If there exists a 1-1 mapping of A onto B, we say that A and B can be
put in 1-1 correspondence, or that A and B have the same cardinal number, or briefly,
that A and B are equivalent, and we write A ∼ B. This relation clearly has the following
properties
• It is reflexive: A ∼ A
• It is symmetric: If A ∼ B, then B ∼ A
• It is transitive: If A ∼ B and B ∼ C, then A ∼ C.
Any relation with these properties is called an equivalent relation.

Definition 2.2. For any positive integer n let Jn be the set whose elements are integers
1, 2, . . . . . . , n; let J be the set consisting of all positive integers. For any set A, we say:
(a) A is finite if A ∼ Jn for some n (the empty set is also considered to be finite).
(b) A is infinite if A is not finite.
(c) A is uncountable if A is neither finite nor countable.
(d) A is at most countable if A is finite or countable.
Countable sets are sometimes called enumerable, or denumerable.

Definition 2.3. By a sequence, we mean a function f defined on the set J of all positive
integers. If f (n) = xn , for n ∈ J, it is customary to denote the sequence f by the symbol
{xn } or sometimes by x1 , x2 , . . . . . . The values of f , that is , the elements xn are called
the terms of the sequence. If A is a set and if xn ∈ A for all n ∈ J, then {xn } is said to
be a sequence in A, or a sequence of elements of A.

Theorem 2.1. Every infinte subset of a countable set A is countable.

Theorem 2.2. Let En , n = 1, 2, 3, . . . . . . be a sequence of countable sets, and put



[
S= En
n=1

Then S is countable.

Corollary 1. Suppose A is at most countable, and, for evey α ∈ A, Bα is at most


countable. Put [
T = Bα
α∈A
Then T is at most countable.

Theorem 2.3. Let A be a countable set, and let Bn be the set of all n−tuples (a1 , . . . . . . , an )
where ak ∈ A(k = 1, . . . . . . , n), and the elements a1 , . . . . . . , an need not be distinct. Then
Bn is countable.

Corollary 2. The set of all rational numbers is countable.


3
Theorem 2.4. Let A be the set of all sequences whose elements are the digits 0 and 1.
This set A is uncountable.

Proof 1. Let E be a countable subset of A, and let E consist of the sequence s1 , s2 , . . . . . . .


We construct a sequence s as follows. If the nth digit sn is 1, we let the nth digit of s be
0, and vice versa. Then the sequence s differs from every member of E in at least one
place; hence s ∈
/ E. But clearly s ∈ A, so that E is a proper subset of A.

We have shown that every countable subset of A is a proper subset of A. It follows


that A is uncountable (for otherwise A would be a proper subset of A, which is absurd.)

Theorem 2.5. If p is a limit point of a set E, then every neighborhood of p contains


infinitely many points of E.

Corollary 3. A finite point set has no limit point.

Theorem 2.6. A set E is open if and only if its complement is closed.

4
3. Problems on Basic Topology

3.1. Rudin’s Solution

Problem 3.1. Prove that the set of all algebraic numbers is countable.

Solution.

Definition 3.1. A complex number z is said to be algebraic if there are integers a0 , a1 , . . . . . . , an ,


not all zero, such that

a0 z n + a1 z n−1 + . . . · · · + an−1 z + an = 0.

Let AN be the set of all complex numbers satisfying one of the equation just listed
with
n + |a0 | + |a1 | + . . . · · · + |an | = N
which means the absolute sum of the coefficient is less than N . The set AN is finite
because each equation has only a finite set of solutions and there are only finitely many
equations satisfying this condition since for each coefficient can take only a finite different
values (< N ). Since all the set AN are finite, they are countable and therefore ∞
S
N =1 AN is
either finite or countable. Since all the rational numbers are algebraic the set of algebraic
numbers are countable.

Problem 3.2. Is the set of all irrational real numbers countable?

Solution. No. If it were, the set of all real numbers, being the union of the rational and
irrational numbers, would be countable.

Problem 3.3. Construct a bounded set of real numbers with exactly three limit points.

Solution.
1 1 1
A = { } ∪ {1 + } + {2 + }
n n n
The set A is bounded. The only limit points are 0, 1, 2. 0 is a limit point because any
neighborhood of 0 contains a real number and by Archimedean property there is N such
that 1/n 6= 0 is in the neighborhood. Similarly 1 and 2 are also limit points. Now we
claim that any other point can not be a limit point. Take any point a + m1 in A where
a = 0, 1 or 2. Take r = m1 − m+11
and observe that the neighborhood Br (a + m1 ) does
not contain any point from A other than itself and hence a + m1 can not be a limit point.
Since the point was chosen arbitrarily, we can conclude that the only three limit points
of A are 0,1,2.

Problem 3.4. Let E 0 be the set of all limit points of a set E. Prove that E 0 is closed.
Prove that E and E have the same limit points. Do E and E 0 always have the same limit
points?

Solution. See Problem 16


5
Problem 3.5. Let A1 , A2 , . . . . . . be subsets of a metric space.
(a) If Bn = ni=1 Ai , prove that B n = ni=1 Ai , for n = 1, 2, 3, . . . . . .
S S

(b) If B = ∞
S S∞
i=1 Ai , prove that B ⊃ i=1 Ai
Show by example, that the inclusion can be proper.

Solution. (a) Fist we will show that if E, F are subsets of a metric space then
E ∪ F = E ∪ F . Assume x ∈ A ∪ B. If x ∈ A or x ∈ B then it is obvious that
x ∈ A∪B ⊂ A ∪ B. Now take x ∈ A0 \A. So B (x)∩A\{x} = 6 ∅ for any neighbor-
hood B (x). Therefore B ∩ (A ∪ B \ {x}) 6= ∅ and therefore x ∈ (A ∪ B)0 ⊂ A ∪ B.
Same argument applies if x ∈ B 0 . Therefore A ∪ B ⊂ A ∪ B.

Now take x ∈ A ∪ B. If x ∈ A ∪ B then obviously x ∈ A ∪ B. Assume that


x ∈ A ∪ B \ (A ∪ B). Then there is a sequence {xn } such that xn → x. Then
either A or B or both contains infinitely many points of {xn }. Without loss of
generality if A contains infinite number of points of {xn } then there is a sub-
sequence {xn } ∩ A that converges to x and hence x ∈ A0 . Which implies that
x ∈ A0 ∪ B 0 . Hence x ∈ A ∪ B and therefore A ∪ B ⊂ A ∪ B.

That’s prove that A ∪ B = A ∪ B. By induction we see the result.


(b) Let x ∈ ∞
S
i=1 Ai . If x ∈ Ai for any i then x ∈ B and we are done. If not then
there is i such that x ∈ A0i . Then there is a sequence {xn } such that xn → x.
But {xn } ⊂ B and therefore x is a limit point of B and hence x ∈ B. Hence
B⊂ ∞
S
i=1 Ai .

Let Ai = { 1i } for i ∈ N and then B = { 1i : i ∈ N}. Observe that Ai = { ii }


/ ∞
S
and 0 ∈ i=1 but 0 ∈ B because 0 is a limit point of B.

6
4. Metric Space

Definition 4.1. Let X be an arbitrary nonempty set. A metric on X is a function

d:X ×X →R

with the following properties


(a) d(x, y) ≥ 0 for all x, y ∈ X and d(x, y) = 0 if and only if x = y.
(b) d(x, y) = d(y, x) for all x, y ∈ X
(c) d(x, y) ≤ d(x, z) + d(z, y) for all x, y, z ∈ X
A metric space (X, d) is a set X equipped with a metric d.

Definition 4.2. Let (X, dX ) be a metric space and Y ⊂ X. The metric subspace (Y, dY )
or (X, dX ) is defined by setting dY to be the restriction of dX to the points in Y .

Definition 4.3. If X and Y are sets then the Cartesian product X × Y is the set of
ordered pairs (x, y) with x ∈ X and y ∈ Y . If dX and dY are metrics on X and Y
respectively, then the product metric dX×Y on X × Y is defined by

dX×Y ((x1 , y1 ), (x2 , y2 )) = dX (x1 , x2 ) + dY (y1 , y2 )

for all x1 , x2 ∈ X and y1 , y2 ∈ Y .

Definition 4.4. A norm on a linear space X (over a scalar field F) is a function k.k :
X → R with the following properties:
(a) kxk ≥ 0 for all x ∈ X;
(b) kλxk = |λ| kxk for all x ∈ X and scalar λ ∈ F
(c) kx + yk ≤ kxk + kyk for all x, y ∈ X
(d) kxk = 0 implies x = 0

Theorem 4.1. A normed linear space (X, k.k) is a metric space (X, d) with the norm-
induced metric
d(x, y) = kx − yk
for x, y ∈ X. Moreover, the metric is translation invariant and homogeneous i.e. for any
x, y, z ∈ X, d(x + z, y + z) = d(x, y) and for any scalar λ ∈ F, d(λx, λy) = |λ| d(x, y)

Definition 4.5. A sequence (xn ) in metric space (X, d) converges to x ∈ X if for every
 > 0 there is an N ∈ N such that d(xn , x) <  for all n ≥ N . The sequence is Cauchy if
for every  > 0 there is an N ∈ N such that d(xm , xn ) <  for all m, n ≥ N

Definition 4.6. A metric space (x, d) is complete if every Cauchy sequence in X con-
verges to a limit in X. A subset Y of X is complete if the metric subspace (Y, d) is
complete. A normed linear space that is complete with respect to the norm-induced
metric is called a Banach space.
7
Definition 4.7. Let X be a normed linear space. If (xn ) is a sequence in X, then the

P n
P
series converges to x ∈ X if the sequence of partial sum (sn ) given by sn = xk
n=1 k=1
converges to x.

Definition 4.8. Let A be a nonempty subset of metric space (X, d). The diameter of A
is
diamA = sup{d(x, y) : x, y ∈ A}.
The set A is bounded if its diameter is finite. The distance (x, A) of a point x ∈ X from
the set A is d(x, A) = inf{d(x, y) : y ∈ A}

Definition 4.9. A topology of a nonempty set X is a collection T of subsets of X such


that
(a) ∅, X ∈ T ;
(1) if Gα ∈ T for α ∈ A, then ∪α∈A Gα ∈ T ;
(2) if Gi ∈ T for i = 1, 2, . . . . . . , n, then ∩ni=1 Gi ∈ T
We call the pair (X, T ) a topological space.

Definition 4.10. A subset A of (X, T ) is a closed set if and only if its complement,
AC = X \ A, is open.

Definition 4.11. Let (X, d) be a metric space. The open ball, Br (a), of radius r > 0
and center a ∈ X is the set

Br (a) = {x ∈ X : d(x, a) < r}.

The closed ball B r , is the set

B r (a) = {x ∈ X : d(x, a) ≤ r}.

Definition 4.12. A subset G of a metric space X is open if for every x ∈ G there exists
an r > 0 such that Br (x) ⊂ G. A subset F of X is closed if its complement F c = X \ F
is open.

Theorem 4.2. Let (X, d) be a metric space. The set of all open sets is a topology on X
called the metric topology.

Theorem 4.3. Let (X, d) be a metric space.


(a) The empty set ∅ and the whole set X are both open and closed.
(b) A finite intersection of open sets is open.
(c) An arbitrary union of open sets is open.
(d) A finite union of closed sets is closed.
(e) An arbitrary intersection of closed sets is closed.

Definition 4.13. Let (X, T ) be a topological space. For any x ∈ X, a set U ⊂ X is a


neighborhood of x is there exists an open set G ⊂ U with x ∈ G.
8
Definition 4.14. Let (X, T ) be a topological space. A sequence (xn ) ⊂ X converges
to a limit x ∈ X if for every neighborhood U of x there is an N ∈ N such that n ≥ N
implies xn ∈ U .

Definition 4.15. Let (X, T ) be a topological space. All points and sets below are
elements and subsets of X.
(a) A point x is a limit point of the set A is every neighborhood of p contains a point
y 6= x such that y ∈ A.
(b) If x ∈ A and x is not a limit point of A, then x is an isolated point of A.
(c) A point x is an interior point of A if there is a neighborhood U of x such that
U ⊂ A.
(d) A is perfect if A is closed and if every point of A is a limit point of A.

Theorem 4.4. A subset F of a topological space (X, T ) is closed if and only if every
convergent sequence of elements in F converges to a limit in F . In other words, if
xn → x ∈ X and xn ∈ F for all n, then x ∈ F .

Theorem 4.5. A subset of a complete metric space is complete if and only if it is closed.

Definition 4.16. Let (X, T ) be a topological space. The closure A of a set A ⊂ X is


the smallest closed set containing A.

Definition 4.17. Let (X, T ) be a topological space. A subset A ⊂ X is dense in X if


A = X.

Definition 4.18. A topology T on X is called Hausdorff if every pair of distinct points


x, y ∈ X has a pair of nonintersecting neighborhoods. In other words, there exists
neighborhoods Vx of x and Vy of y such that Vx ∩ Vy = ∅.

Definition 4.19. A subset B of a topology T is a base for T if for every G ∈ T there is


S
a collection of sets Bα ∈ B such that G = α Bα .

Definition 4.20. A collection N of neighborhoods of a point x ∈ X is called a neigh-


borhood base for x if for each neighborhood V of x there is a neighborhood W ∈ N such
that W ⊂ V .

Definition 4.21. A topological space (X, T ) is first countable if every x ∈ X has a


countable neighborhood base, and second countable if T has a countable base.

Definition 4.22. A topological space (X, T ) is separable if it contains a countable, dense


subset.

Definition 4.23. A Hamel basis (or algebraic basis ) B of a linear space X is maximal
linearly independent set of vectors such that every x ∈ X can be written uniquely as a
finite linear combinition of vectors from B.
9
Definition 4.24. Let X be a separable Banach space. A sequence (xn ) is a Schauder
basis of X is for every x ∈ X there exists a unique sequence of scalars (cn ) such that

P
x= cn x n .
n=1

10
5. Problems on Metric Space

Problem 5.1. Show that d(x, y) = kx − yk is a metric on Rn where k·k is the standard
Euclidean norm on Rn .

Solution. d(x, y) ≥ 0 since kxk ≥ 0 by the definition of Euclidean norm.


If d(x, y) = 0 then kx − yk = 0 and hence by definition of norm, x − y = 0 ⇒ x = y.

d(x, y) = kx − yk = k − 1(y − x)k = | − 1|ky − xk = d(y, x)


and hence the commutativity of metric is proved. We now want to prove the triangle
inequality.

d(x, y) = kx − yk = k(x − z) − (y − z)k ≤ kx − zk + ky − zk = d(x, z) + d(y, z)

Hence d(x, y) is indeed a metric.

11
Problem 5.2. Determine which of the following define metrics on R.
(a) d1 (x, y) = (x − y)2 ,
p
(b) d2 (x, y) = |x − y|,
(c) d3 (x, y) = |x2 − y 2 |,
(d) d4 (x, y) = |x − 2y|,
(e) d5 (x, y) = |x − y| /[1 + |x − y|].

Solution. (a) Consider to points x = 5 and y = −5 and z = 0 and observe that

d1 (x, y) = (5 + 5)2 = 25 and , d1 (x, z) = d1 (y, z) = 25, and d1 (x, y)  d1 (x, z) + d1 (y, z)

That is triangle inequality does not hold. Hence d1 is not a metric.


p
(b) d2 (x, y) ≥ 0 and d2 (x, y) = 0 ⇒ |x − y| = 0 ⇒ |x − y| = 0 ⇒ x = y.

p p
Commutativity is trivial because d2 (x, y) = |x − y| = |y − x| = d2 (y, x)

The triangle inequality comes from the triangle inequality of the absolute value
function,
|x − y| ≤ |x − z| + |y − z|
p p
⇒ |x − y| ≤ |x − z| + |y − z| + 2 |x − z| |y − z|
p  p p 2
⇒ |x − y| ≤ |x − z| + |y − z|

⇒d2 (x, y)2 ≤ (d2 (x, z) + d2 (y, z))2


⇒d2 (x, y) ≤ d2 (x, z) + d2 (y, z)
(c) Assume x = 5 and y = −5 then d3 (x, y) = |52 − (−5)2 | = 0 though x 6= y. Hence
d3 is not a metric.
(d)

12
Problem 5.3. For f, g : [0, 1] → R, let d1 (f, g) = supx∈[0,1] {x2 |f (x) − g(x)|}, d2 (f, g) =
R1
supx∈[0,1] |f (x) − g(x)|, and d3 (f, g) = 0 |f (x) − g(x)| dx. Prove that d1 , d2 , and d3 de-
fine metrics on the linear space of C([0, 1]) of continuous functions f : [0, 1] → R.

Solution. Claim: d1 is a metric on C([0, 1]).


Since x2 ≥ 0 and |f (x) − g(x)| ≥ 0 we have d1 (f, g) ≥ 0.
If d1 (f, g) = 0 then x2 |f (x) − g(x)| = 0 for all x ∈ [0, 1]. If x 6= 0 then we must have
f (x) = g(x). Since f and g are equal at all the points and both of the functions are
continuous f (0) = g(0). Therefore f = g for all x ∈ [0, 1].
Commutativity is obvious because x2 |f (x) − g(x)| = x2 |g(x) − f (x)|.
Now we will prove the triangle inequality:
Let h ∈ C([0, 1]). By the triangle inequality of real numbers under usual metric we know,
for all x ∈ [0, 1]
|f (x) − g(x)| ≤ |f (x) − h(x)| + |h(x) − g(x)| .
Let
y =d1 (f, h) + d1 (g, h)
= sup {x2 |f (x) − h(x)|} + sup {x2 |h(x) − g(x)|}
x∈[0,1] x∈[0,1]
2 2
≥{x |f (x) − h(x)|} + {x |h(x) − g(x)|}, ∀x ∈ [0, 1]
≥{x2 |f (x) − g(x)|}
Therefore y is an upper bound of x2 |f (x) − g(x)| and hence y ≥ supx∈[0,1] {x2 |f (x) − g(x)|},
i.e.
d1 (f, g) ≥ d1 (f, h) + d1 (h, g).
Claim: d2 is a metric on C([0, 1]).
It is very similar to the proof for d1 .

Claim: d3 is a metric on C([0, 1]).


Since |f (x) − g(x)| ≥ 0 for all x ∈ [0, 1],
Z 1
d3 (f, g) = |f (x) − g(x)| dx ≥ 0.
0

Let h(x) = |f (x) − g(x)| and so, h(x) ≥ 0 for all x ∈ [0, 1]. If d3 (f, g) ≥ 0 then
R1
0
h(x)dc = 0. If h(x) 6= 0 for at least one point by the continuity of h in [0, 1] and the
R1
property of Riemann Integral 0 h(x)dc > 0 and hence h(x) = 0 for all x ∈ [0, 1] which
implies f = g.
Commutativity is obvious because of commutativity of real numbers with usual metric.
Let h ∈ C([0, 1]). By the triangle inequality of real numbers under usual metric we know,
for all x ∈ [0, 1]
|f (x) − g(x)| ≤ |f (x) − h(x)| + |h(x) − g(x)| .
13
Therefore,
Z 1 Z 1 Z 1
|f (x) − g(x)| dx ≤ |f (x) − h(x)| dx + |h(x) − g(x)| dx.
0 0 0

Hence triangle inequality holds for d3 . So indeed d3 is a metric on C([0, 1])

14
Problem 5.4. Show that Rn with d(x, y) = kx − yk∞ is a metric space, where kxk∞ =
supxi |xi | for i = 1, 2, . . . , n.

Solution. Since |xi | ≥ 0 the supremum is also greater than or equal to 0 and hence
d(x, y) ≥ 0.
If d(x, y) = 0 then sup i = 1, . . . n |xi − yi | = 0. Since |xi − yi | ≥ 0 for all i = 1, 2, . . . . . . n
we have xi − yi = 0 and so xi = yi for all i. Therefore x = y. |xi − yi | = |yi − xi | implies
d(x, y) = d(y, x)

a =d(x, z) + d(z, x) for some z ∈ Rn


= sup |xi − zi | + sup |zi − yi |
i i

≥ |xi − zi | + |zi − yi | , ∀i = 1, 2, . . . . . . n
≥ |xi − yi | , ∀i = 1, 2, . . . . . . n
So, a is upper bound of |xi − gi | and hence supi |xi − yi | ≤ a. Therefore,

d(x, y) ≤ d(x, z) + d(z, y)

for some z ∈ Rn .

15
Problem 5.5. Consider the convergence of sequences (xn ) = (ξn,1 , ξn,2 , ξn,3 , · · · ) to a
point y = (η1 , η2 , η3 , · · · ) in `2 . Show that if xn → y, then ξn,i → ηi for i = 1, 2, 3, · · · .
But, construct an example that shows that ξn,i → ηi for i = 1, 2, 3, · · · does not imply
that xn → y. (This is similar to the idea of pointwise convergence not guaranteeing
metric-convergence for sequences of functions.) Consider the Hilbert Cube subspace of
`2 defined by,

H = x = (ξ1 , ξ2 , ξ3 , · · · ) : x ∈ `2 and |ξi | ≤ 1/i, i = 1, 2, 3, · · · .




Show that a sequence (xn ) in H converges to a point y = (η1 , η2 , η3 , · · · ) if and only if


ξn,i converges to ηi for i = 1, 2, 3, · · · .

Solution. Solution to 1(a) Let  > 0 be given. Since xn → y, there exists N ∈ N such
that d(xn , y) ≤  for all n ≥ N .
d(xn , y) ≤ 
v
u∞
uX
⇒t |ξn,i − ηi |2 ≤ 
i=1

X
⇒ |ξn,i − ηi |2 ≤ 2
i=1

⇒ |ξn,i − ηi |2 ≤ 2 , ∀i = 1, 2, 3, . . . . . .
⇒ |ξn,i − ηi | ≤  ∀i = 1, 2, 3, . . . . . .
From above we conclude that, for any given  > 0 there is N such that |ξn,i − ηi | ≤ 
whenever n > N for all i = 1, 2, 3, . . . . . . and hence ξn,i → ηi for i = 1, 2, 3, . . . . . .
Solution to 1(b) Consider the sequence (xn ) such that

1 if n = i
ξn,i =
0 if n 6= i

Fix an i, then ξn,i → 0 because for any , |ξn,i − 0| = 0 <  for all n > i. But d(xn , 0) = 1
for all n and hence xn does not converge to 0.
Solution to 1(c) Since Hilbert space is also an `2 space by the proof in 1(a) we know
if xn → y then ξn,i → ηi for all i = 1, 2, 3, . . . . . . .

Lemma 1. If s = ∞
P
P∞ i=1 xi converges then for given  > 0 there exists an M such that

i=M +1 xi < .

Proof: The sum convergence if the sequence of partial sum sn = ni=1 xi converges to
P

s that is for given  > 0 there is M such that |s − sn | <  for all n ≥ M . We take
|s − sM | <  which implies ∞
P
i=M +1 si < 

16
Now assume that ξn,i → ηi for i = 1, 2, 3, . . . . . . . We have to show that xn → y that is
for given  > 0 there exists an N such that d(xn , y) <  whenever n ≥ N .
Since |ξn,i | ≤ 1i and ξn,i → ηi by the result from elementary analysis we have |ηi | ≤ 1i .

P∞ 2 P∞ 1 P∞ 2 π2
Observe that j=1 |ηj | ≤ j=1 i2 and by comparison j=1 |ηj | ≤ 6
and hence
y ∈ `2 . Since y ∈ `2 and |ηi | ≤ 1i , y ∈ H.
So, |ξn,i − ηi | ≤ |ξn,i | + |ηi | = 2 1i .

X
2
d(xn , y) = |ξn,i − ηi |2
i=1
P∞
converges by comparison because |ξn,i − ηi |2 ≤ 4
i2
and 4 1
i=1 i2 converges.

P∞ 2
Let  > 0 be given. By Lemma 1 there exists an M such that i=M +1 |ξn,i − ηi |2 < 2
.

For i = 1, 2, . . . . . . , M , since ξn,i → ηi there is an Ni such that |ξn,i − ηi | < √ .


2M
2
Taking N = max(Ni , i < M ) we see that, M 2
P
i=1 |ξn,i − ηi | < 2 .

Hence,
2 2
d(xn , y)2 < + = 2
2 2
which implies d(xn , y) < .

17
Problem 5.6. Let p ≥ 1 be a fixed real number. The real space `p is defined such that
x ∈ `p is a sequence of real numbers x = (ξj ) = (ξ1 , ξ2 , ξ3 , · · · ) such that ∞ p
P
j=1 |ξj | < ∞.
For x, y ∈ `p , where x = (ξj ) and y = (ηj ), let d(x, y) be defined by

!1/p
X
d(x, y) = |ξj − ηj |p .
j=1
P 1/p
∞ p
Show that `p with d defined above is a metric space. (Hint: If j=1 |ξj − ηj | is
finite, then d is a map to R and properties (a) and (b) are clearly satisfied. To show this is
finite, use Minkowski’s inequality, which is also useful in proving the triangle inequality.
You do not need to prove Minkowski’s inequality.)

P∞ p P∞ p
Solution. Since y = (ηi ) ∈ `p , j=1 |ηj | < ∞ and hence j=1 |−ηj | < ∞. Also
x = (ξj ) ∈ `p implies ∞ p
P
j=1 |ξj | < ∞. By Minkowski inequality we get,


!1/p ∞
!1/p ∞
!1/p ∞
!1/p
X p
X p
X p
X p
d(x, y) = |ξj − (−ηj )| = |ξj + ηj | ≤ |ξj | + |ηj | <∞
j=1 j=1 j=1 j=1
P 1/p
∞ p
d(x, y) = 0 implies |ξ
j=1 j − η j | = 0 ⇒ |ξj − ηj | = 0 for all j and hence ξj = ηj
for all j. Therefore x = y.
Commutativity of d follows from commutativity of real numbers under usual metric. Now
we will prove the triangle inequality. To do that let z = (ζj ) ∈ `p


!1/p
X
d(x, y) = |ξj − ηj |p
j=1


!1/p
X
= |ξj − ζj + ζj − ηj |p
j=1


!1/p
X
≤ ||ξj − ζj | + |ζj − ηj ||p
j=1


!1/p ∞
!1/p
X p
X p
≤ |ξj − ζj | + |ζj − ηj |
j=1 j=1

=d(x, z) + d(z, y)
Another way to show this is to prove that

!1/p
X
kxk = |ξj |p
j=1

is a norm. Then by theorem 1 we can say that d is a metric in `p .

18
Theorem 1: A normed linear space (X, k.k) is a metric space (X, d) with the norm-induced
metric d(x, y) = kx − yk for x, y ∈ X. Moreover, the metric is translation invariant and
homogeneous, i.e. for any x, y, z ∈ X, d(x + z, y + z) = d(x, y) and for any scalar λ ∈ F,
d(λx, λy) = |λ| d(x, y).

19
Problem 5.7. Let `∞ be the space defined by real-valued bounded sequences, i.e., x ∈ `∞
implies x = (ξi ) = (ξ1 , ξ2 , · · · ) such that |ζi | ≤ cx for all j ∈ N where cx ≥ 0 is some real
number that may depend on x but not on j. For x, y ∈ `∞ , where x = (ξi ) and y = (ηi ),
show that
d(x, y) = sup |ξj − ηj |
j∈N

defines a metric on ` .

Solution. First we show that the metric maps to R. Observe that,

d(x, y) = sup |ξj − ηj | ≤ sup(|ξj | + |ηj |) ≤ cx + cy < ∞


j∈N j∈N

Since |ξj − ηj | ≥ 0 for all j we have d(x, y) = supj∈N |ξj − ηj | ≥ 0.


Setting d(x, y) = 0 implies supj∈N |ξj − ηj | = 0 together with the fact that |ξj − ηj | ≥ 0
implies that for all j ∈ N, |ξj − ηj | = 0 ⇒ ξj = ηj , ∀j and hence x = y.
Since |a − b| = |b − a| , ∀a, , b ∈ R we have, d(x, y) = supj∈N |ξj − ηj | = supj∈N |ηj − ξj | =
d(y, x)
Finally, we show that the triangle inequality holds. Consider another sequence z = (ζj ).
Observe that,
d(x, z) = sup |ξj − ζj | ≥ |ξk − ζk | , for all k
j∈N

d(y, z) = sup |ηj − ζj | ≥ |ηk − ζk | , for all k


j∈N
Therefore,
d(x, z) + d(y, z) ≥ |ξj − ζj | + |ηj − ζj | ≥ |ξj − ηj | , ∀j.
That is d(x, z) + d(y, z) is a upper bound of |ξj − ηj | and by the definition of supremum,

d(x, z) + d(y, z) ≥ sup |ξj − ηj | = d(x, y)


j∈N

20
Problem 5.8. Clearly `1 ⊂ `∞ . Is `1 dense in `∞ ? Is `1 closed in `∞ ? Justify your
answers.

Solution. (a) Consider the constant sequence x = (1) ∈ `∞ . If y = (ηj ) ∈ `1 then


P∞
j=1 |ηj | < ∞. By the elementary analysis we know that ηj → 0. Therefore for
any y ∈ `1 , d(x, y) ≥ 0 for all y ∈ `1 . Hence a ball centered at x ∈ `∞ with radius
0 <  < 1 does not contain any element form `1 . We conclude that `1 is not dense
in `∞
(b) Consider the sequence xn = ξn,j so that ξn,j = 1j if j ≤ n otherwise ξn,j = 0. This
sequence converges point-wise to y = n1 . Indeed the sequence (xn ) converges in
`∞ because for any given  > 0 there is N ∈ N such that n1 <  for all n > N and
1
<  for all n ≥ N . But since ∞ 1
P
so kxn − yk∞ = n+1 n=1 n does not converge, the
limit point y ∈ / `1 and hence `1 is not closed.

21
Problem 5.9. Prove that if a subsequence of a Cauchy sequence in a metric space
converges, then the full sequence converges to the same limit.

Solution. Consider the Cauchy sequence (xn ) in the metric space (X, d). By definition
there exists an N1 ∈ N such that d(xn , xm ) < 2 for all n, m > N1 . Now suppose the
subsequence (xnk ) of (xn ) converges to x ∈ X and hence for the given  there exists an
N2 ∈ N such that for all nk > N2 , d(xnk , x) < 2 . Therefore,

d(xn , x) ≤ d(xn , xnk ) + d(xn,k , x) < 

for all n ≥ N = max(N1 , N2 ). Hence the sequence (xn ) converges to the same limit of
the subsequence.

22
Problem 5.10. Let (xn ) and (yn ) be Cauchy sequences in a metric space (X, d). Show
that (d(xn , yn )) converges whether or not (xn ) and (yn ) converge.

Solution. Let  > 0 be given, Since (xn ) is a Cauchy sequence, there exists an N1 ∈ N
such that d(xn , xm ) < 2 for all m, n > N1 .
Since (yn ) is a Cauchy sequence, there exists an N2 ∈ N such that d(yn , ym ) < 2 for all
n, m > N2 .

d(xn , yn ) ≤ d(xn , xm ) + d(xm , ym ) + d(yn , ym )


Which implies
 
d(xn , yn ) − d(xm , ym ) ≤ d(xn , xm ) + d(yn , ym ) < + =
2 2
Similarly we can show that

d(xm , ym ) − d(xn , yn ) < 

and hence
|d(xn , yn ) − d(xm , ym )| < 
for all n, m > N = max(N1 , N2 ) Which implies that d(xn , yn ) is a Cauchy sequence in R
and hence is convergent.

23
Problem 5.11. Let (X1 , d1 ) and (X2 , d2 ) be two complete metric spaces. Is the product
metric space X1 × X2 complete?

Solution. Take a Cauchy sequence ((xn , yn )) in X × Y . For given  > 0 there exists an
M such that n, m ≥ M implies

dX×Y ((xn , yn ), (xm , ym )) ≤ .

Since dX (xn , xm ) ≥ 0 and dy (yn , ym ) ≥ 0 we get,

dX×Y ((xn , yn ), (xm , ym )) ≤ 


⇒dX (xn , xm ) + dY (yn , ym ) ≤ 
⇒dX (xn , xm ) ≤  and dY (yn , ym ) ≤ 
Therefore, the sequences (xn ) and (yn ) in the metric space X and Y respectively are
Cauchy sequences. Since X and Y are complete theses sequences converge. Say xn → x
and yn → y for some x ∈ X and y ∈ Y .

Let  > 0 be given. xn → x implies there exists an N1 ∈ N so that dX (xn , x) ≤ 2


whenever n ≥ N1 . yn → y implies that there exists an N2 ∈ N so that dY (yn , y) ≤ 2 .
Take N = max(N1 , N2 ).
 
dX×Y ((xn , yn ), (x, y)) = dx (xn , x) + dy (yn , y) ≤ + = 
2 2
for all n ≥ N . Therefore the sequence (xn , yn ) converges to (x, y). Since (xn , yn ) was
chosen to be an arbitrary Cauchy sequence we can conclude that X × Y is a complete
metric space.

24
Problem 5.12. Prove that a metric space X is complete if and only if the intersection
of every descending sequence of closed balls whose radii approach zero consists of a single
point.

Solution. Let (Bn ) be a descending sequence of closed balls and xn be the center of Bn
for all n ∈ N. Since diam(Bn ) → 0, for given  > 0 there exists N such that diam(Bn ) < 
for all n ≥ N . For m > n > N , Bm ⊂ Bn and so xm ∈ Bn and hence d(xn , xm ) ≤ .
So, (xn ) is a Cauchy sequence and hence converges to x ∈ X (say) since the space X is
complete.

Now we want to show that x ∈ Bn for all n. If not consider there is an m for which x ∈/ Bm .
Since we are considering a nested sequence of closed balls, xi ∈ Bm , ∀i ≥ m. So the
subsequence (xi , i ≥ m) is contained in Bm . A subsequence of a convergent sequence
converges and since the subsequence is entirely contained in Bm and Bm is closed set, by
definition, the subsequence has to converge to a point in Bm which contradicts the fact
that subsequence of a convergence sequence converges to the same limit point. Hence
x ∈ Bn for all n and therefore x ∈ ∞
T
n=1 Bn .
Suppose there is another point y 6= x in ∞
T
n=1 Bn . But by the definition of descending
sequence of closed balls limn→∞ diam(Bn ) → 0. If x 6= y, d(x, y) > 0 say β = d(x, y) and
hence diam ( ∞
T
n=1 ) ≥ β which contradicts the definition of descending sequence of closed
balls.

Now we consider that intersection of every descending sequence of closed balls whose
radii approach zero consists of a single point and want to prove that X is complete.

Let (xn ) be a Cauchy sequence in X. First we construct a descending sequence of closed


balls.

Since (xn ) is Cauchy there exists an N1 such that for all m, n1 ≥ N1 , d(xn1 , xm ) < 21 .
Consider a closed ball B1 = B(xn1 , 1) = {y ∈ X : d(y, xn1 ) ≤ 1}. So, xm ∈ B1 for all
m ≥ n1 .

Now take  = 41 . So ∃N∗ such that d(xn2 , xm ) < 14 for all m > n2 ≥ N∗ . Take N2 =
max(N1 , N∗ ) and that gives, d(xn1 , xn2 ) < 12 and d(xn2 , xm ) < 41 for all m ≥ N2 . Construct
a closed ball B2 = B(xn2 , 12 ). We claim that B2 ⊂ B1 . Take any point xk ∈ B2 and observe
that,
1 1
d(xn1 , xk ) ≤ d(xn1 , xn2 ) + d(xn2 , xk ) ≤ + < 1
2 4
which implies xk ∈ B1 and hence B2 ⊂ B1 .

By induction we can contract such a sequence of closed balls. Suppose d(xnk , xm ) < 21k
1

for all m > nk ≥ Nk and consider the closed ball Bk = B xnk , 2k−1 and so xm ∈ Bk for
25
all m > Nk . Again since (xn ) is Cauchy there is N∗ so that ∀m > nk+1 ≥ N∗ we have
1
. Take Nk+1 = max(Nk , N∗ ) to assert both d xnk , xnk+1 < 21k and

d(xnk+1 , xm ) < 2k+1
1
d(xnk+1 , xm ) < nk+1 for all m > Nk+1 . Now construct the closed ball Bk+1 = B(xnk+1 , 21k ).
We claim that Bk+1 ⊂ Bk and it is direct consequence of the triangle inequality.

Therefore we have a descending sequence of closed balls Bk whose radii 21k approaches
zero and by the hypothesis the intersection of all the closed balls consists of a single point,
say (x ∈ X). We want to show now that x is the limit point of the subsequence (xnk ).

Since x ∈ Bk for all k and xnk is the center of Bk we can write d(xnk , x) < 21k . By
comparison d(xnk , x) → 0 as k → ∞ and hence xnk converges to x. From elementary
analysis we know if a subsequence of a Cauchy sequence converges then the sequence
itself converges to the same point (Proof as lemma is given bellow)

Since (xn ) is chosen to be arbitrary Cauchy sequence we can assert that any Cauchy
sequence in X converges and hence X is complete.

Lemma. If a subsequence of a Cauchy sequence converges then the sequence converges.

Proof: Let (xn ) is a Cauchy sequence in the metric space (X, d) and (xnk ) is a subsequence
that converges to x ∈ X. Let  > 0 be given. Since the sequence is Cauchy, there exists
an N1 such that d(xn , xm ) < 2 for all m, n ≥ N1 . Also since the subsequence converges
to x, there is K such that d(xnk , x) ≤ 2 for all k ≥ K. Let N = max(N1 , K). By triangle
inequality,
d(xn , x) ≤ d(xn , xnk ) + d(xnk , x) < 
since nk ≥ k > N . Therefore (xn ) converges to x.

26
Problem 5.13. For f, g : [0, 1] → R, let d1 (f, g) = supx∈[0,1] {x2 |f (x) − g(x)|}, d2 (f, g) =
R1
supx∈[0,1] |f (x) − g(x)|, and d3 (f, g) = 0 |f (x) − g(x)| dx. Prove that (C([0, 1]), d1 ) is
not complete but (C([0, 1], d2 ) is complete (this requires recalling some useful results from
elementary analysis that we will revisit in more generality in lecture 4). Since these
metrics can be defined by norms, one choice of norm produces a Banach space, while the
other does not (this happens on infinite dimensional spaces where norms are not always
equivalent). If we use d3 is the resulting metric space complete?

Solution. (1) To show that (C([0, 1]), d1 ) is not complete we have to show that there
is a Cauchy sequence that does not converge in C([0, 1]).
Consider the sequence of (fn ) continuous functions

fn = (1 − x)n

Then,

d(fn , fm ) = sup {x2 |(1 − x)n − (1 − x)m |} = sup {x2 |(1 − x)n |} = sup {x2 (1 − x)n }
x∈[0,1] x∈[0,1] x∈[0,1]

At x = 0, d(fn , fm ) = 0 and for x ∈ (0, 1], (1 − x)n → 0, so (fn ) is a Cauchy


sequence. The sequence of function converges to a function f given by f (0) = 0
and f (x) = 1 if x ∈ (01, ], which is not a continuous function and hence f ∈ /
C([0, 1]). We conclude that (C([0, 1]), d1 ) is not complete.
(2) Consider any Cauchy sequence (fn ) in C([0, 1]). Let  > 0 be given. There exists
an N ∈ N such that d2 (fn , fm ) <  for all n, m > N . For each x ∈ [0, 1] we have
that
|fn (x) − fm (x)| ≤ d2 (fn , fm ) < .
So, (fn (x)) is a Cauchy sequence in R. Since R is complete, for each x ∈ [0, 1]
(fn (x)) converges. We define point-wise limit function f (x) = lim fn (x) for each
n→∞
x ∈ [0, 1]. If we can show that fn → f uniformly, then from elementary analysis, f
is continuous on [0, 1] and d(fn , f ) has meaning. We use the result from example
1.25 in the text that says:
Example 1.25 If {xn,α ∈ R|n ∈ N, α ∈ A} is a set of real number indexed by the
natural numbers N and an arbitrary set A, then
h i  
sup lim inf xn,α ≤ lim inf sup xn,α
α∈A n→∞ n→∞ α∈A

Use the result to obtain,

sup = sup |fn (x) − f (x)| = lim |fn (x) − fm (x)| ≤ lim inf sup |fn (x) − fm (x)| .
x∈[0,1] x∈[0,1] m→∞ m→∞ x∈[0,1]

This implies that

sup |fn (x) − f (x)| ≤ lim inf d2 (fn , fm ).


x∈[0,1] m→∞
27
Since (fn ) is Cauchy in (C([0, 1]), d2 ), there exists N such that n, m > N implies
d2 (fn , fm ) < , which implies fn → f uniformly. Moreover, this proves that
fn → f in (C([0, 1])).

28
Problem 5.14. Prove that a discrete metric space X is separable if and only if X is
countable. (A discrete metric space is any set X with the discrete metric d(x, y) = 0 if
x = y and d(x, y) = 1 if x 6= y.)

Solution. By the definition of separable space X to be separable has to contain a count-


able, dense subset. In the discrete metric space no proper subset is dense in X because
if we take a proper subset A of X then there is a point a ∈ X \ A and since all the con-
vergent sequence in discrete metric space are the constant sequence there is no sequence
in A that converges to a ∈ / A. So, the only dense subset of X is the space itself and by
definition, to be separable it has to be countable.
If X is countable then X is the countable dense subset of X and hence it is separable.

29
Problem 5.15. Prove that `∞ is not separable, but `p with 1 ≤ p < +∞ is separable.
Prove that `p with 1 ≤ p ≤ +∞ is complete.

Solution. (1) Let y = (ηj ) be a sequence of 0’s and 1’s. Then y ∈ `∞ . With y we
associate the real number ŷ with the decimal expansion of 0’s and 1’s given by
ŷ = 0.η1 η2 η3 . . . . . . . By the previous problem, there are an uncountable number
of such y ∈ `∞ . Furthermore, the metric on `∞ restricted the subset of such y is
the discrete metric, so by one of the examples from class `∞ cannot be separable.
(2) Let A denote the set of all sequences of the form y = (η1 , η2 , . . . . . . , ηn , 0, 0, . . . . . . )
where ηi ∈ Q for each i. By the proposition that Cartesian product of countable
sets is countable we have that A is countable. We want to show that A is dense
in `p . Suppose x = (ξi ) be as sequence in `p . Therefore ∞ p
P
i=1 |ξi | is finite. A
series converges if the sequence of the partial sum sn = nj=1 |ξj |p converges to
P

the sum of the series. Therefore the sequence ∞ p


P Pn
i=n+1 |ξi | = s − i=1 |ξi | P → 0.
P∞ p
Therefore for any given  > 0 there is N ∈ N such that i=n+1 |ξi |p < 2 for all
n ≥ N.
Consider a sequence y ∈ A such that y = (η1 , η2 , . . . . . . , ηN , 0, 0, . . . . . . ) where

|ηi − ξi | < √ p √
2pN
for all i = 1 . . . . . . N . Such a sequence exists because Q is dense
in R. Observe that,

!1/p N ∞
!1/p  1/p
X p
X p
X p p p
d(x, y) = |ξi − ηi | = |ξi − ηi | + |ξi − ηi | < + =
i=1 i=1 i=N +1
2 2

Therefore for any given , any ball B (x) contains a point y of A and hence A is
dense in `p .
We conclude that `p is separable.
(3) We consider two cases. First we will prove completeness of `p for 1 ≤ p < ∞ and
then we will prove for `∞ .

Let (xn ) be a Cauchy sequence in `p where xn = (ξn,i ). Let  > 0 be given.


There exists an N ∈ N such that d(xn , xm ) <  for all n, m ≥ N . That is

!1/p
X
|ξn,i − ξm,i |p <
i=1

Which implies,

X
(5.1) |ξn,i − ξm,i |p < p
i=1

Since |ξn,i − ξm,i | ≥ 0 we must have,

|ξn,i − ξm,i |p < p ⇒ |ξn,i − ξm,i | < 

for all i whenever n, m ≥ N . Therefore for a fixed i, (ξn,i ) is a Cauchy sequence in


R and hence convergence. Let xn,i → ηi for all i. Construct a sequence y = (ηi ).
30
Claim: y ∈ `p . The equation (5.1) implies that for any k
k
X
|ξn,i − ξm,i |p < p
i=1

Since the sum is finite and true for all m, n ≥ N we can take the limit m → ∞
to get,
k
X
(5.2) |ξn,i − ηi |p < p
i=1

Since the above inequality is true for every k we can take k → ∞ and that
P∞ p p
implies the sequence i=1 |ξn,i − ηi | < ∞ and hence xn − y ∈ ` . Writing
y = (y − xn ) + xn and using Minkowski’s inequality we can write

X
|ηi |p < ∞
i=1
p
and hence y ∈ ` .
Now we want to prove that xn → y. To show that we use the equation (5.2).

!1/p
X
d(xn , y) = |ξn,i − ηi |p <
i=1

for all n ≥ N . Hence xn → y in `p showing that `p space is complete.

Now we will prove `∞ is complete. Let (xn ) be any Cauchy sequence in `∞ where
xn = (ξn,j ). Consider any  > 0. There exists N such that n, m > N implies

d`∞ (xm , xn ) = sup |ξm,j − ξn,j | < .


j∈N

Thus, for each fixed j ∈ N, and n, m > N , |ξm,j − ξn,j | < , so for fixed j, (ξn,j )
is a Cauchy sequence of real numbers. By the completeness of R, these sequences
converge for each j. Let (ξj ) denote the sequence x defined by the term-wise limit
of each of these Cauchy sequences of real numbers. We show that x ∈ `∞ and
xn → x. For each j, we have the following inequalities

|ξj | − |ξn,j | ≤ |ξj − ξn,j | ≤ |ξj − ξm,j | + |ξj − ξn,j | .

There exists an N such that for n, m > N we have |xij − ξm,j | < 2 and |ξj − ξn,j | <

2
, so |ξj | − |ξn,j | ≤ . Fix any n > N , and observe that xn ∈ `∞ implies there
exists real number cn such that |ξn,j | < cn for each j, so

|ξj | ≤  + cn .

The bound does not depend on j, so x = (ξj ) ∈ `∞ . We also have that d(x, xn ) < 
for n > N from the previous inequality since  does not depend on j, which shows
xn → x. Since (xn ) was an arbitrary Cauchy sequence, `∞ is complete.

31
Problem 5.16. Let E 0 be the set of all limit points of a set E ⊂ X where (X, d) is a
metric space. Prove that E 0 is closed. Prove that E and Ē have the same limit points.
Do E and E 0 always have the same limit points?

Solution. (1) If the set of limit point of E 0 is empty then we have nothing to prove.
Let x be a limit point of E 0 . For any given  > 0 the open ball B (x) contains a
point y 6= x of E 0 . So y is a limit point of E. d(x, y) <  and so  − d(x, y) > 0.
The ball centered at y and radius  − d(x, y) contains a point z ∈ E and so
d(y, z) <  − d(x, y). So,

d(x, z) ≤ d(x, y) + d(y, z) < d(x, y) +  − d(x, y) = .

Which implies that for any  > 0 the open ball B (x) contains a point of E and
hence x is a limit point of E. Therefore we conclude that x ∈ E 0 and hence E 0 is
closed.
(2) Suppose x is a limit point of E. Then there is a sequence (xn ) ⊂ E such that
xn → x. Since E ⊂ E we have (xn ) ⊂ E and together with the fact that xn → x
in E we conclude that x is a limit point of E.

Now assume x is a limit point of E. For any given  > 0 the open ball B (x)
contains a point of y ∈ E. If y ∈ E then we have nothing to prove if not then
y ∈ E 0 . Similarly above we can show that there is a point zıE such that d(x, z) < 
and hence B (x) contains a point of E other than itself. Hence x is a limit point
of E. Therefore we conclude that E and E have the same limit points.
(3) NO! Consider X = R with usual metric and E = 1, 21 , . . . . . . n1 . . . then E 0 = {0}.
The limit only limit point of E is 0 and E 0 does not have any limit point.

32
Problem 5.17. Let A1 , A2 , A3 , . . . be subsets of a metric space.
(a) If Bn = ∪ni=1 Ai , prove that B̄n = ∪ni=1 Āi for n = 1, 2, 3, . . ..
(b) If B = ∪∞ ∞
i=1 Ai , prove that ∪i=1 Āi ⊂ B̄. Show, by an example, that this inclusion can
be proper.

Solution.

33
Problem 5.18. Is every point of every open set E ⊂ R2 a limit point of E? Answer the
same question for closed sets in R2 .

Solution: Every point of an open set is a limit point of E because for each x ∈ E and
there exists r > 0 such that Br (x) ⊂ E and each of these balls contains an infinite number
of points on E ∩ R2 .
It is not true for closed set. Consider the singleton set E = {(0, 0)}. It does not have
any limit point that is E 0 = ∅ and clearly E 6= ∅

34
Problem 5.19. Let E o denote the set of all interior points of a set E.
(a) Prove that E o is always open.
(b) Prove that E is open if and only if E o = E.
(c) If G ⊂ E and G is open, prove that G ⊂ E o .
(d) Prove that the complement of E o is the closure of the complement of E.
(e) Do E and Ē always have the same interiors?

Solution. (1) If x ∈ E o then, by definition of interior point, there is an neighborhood


of x that is contained entirely in E. Assume that the  ball B (x) ⊂ E. Take
any point y ∈ B (x) and x 6= y. The open ball centered at y and radius β =  −
d(x, y) > 0, written as Bβ (y) is entirely contained in B (x) and hence Bβ (y) ⊂ E.
Since y is an arbitrary point, we conclude that all the point in B (x) are interior
points of E, so B (x) ⊂ E o . This implies that x is an interior point of E o . Since
x was chosen to be arbitrary, we conclude that E o is an open set.
(2) First assume that E is open. By definition, for every point there is an open neigh-
borhood that is entirely contained in E. Then all the points of E are interior
points that is if x ∈ E then x ∈ E o or in other words E ⊂ E o and by definition
of a set of interior point E o ⊂ E and hence E = E o .

Now assume that E = E o . By proof (a) E o is open and hence E.


(3) Take any x ∈ G. Since G is open there is a ball B (x) contained in G that
is B (x) ⊂ G ⊂ E. Which tell us that for every point of G there is a open
neighborhood that is entirely contained in E and hence all the points of G are
interior points of E and hence G ⊂ E o .
(4) We have proved that E o is open. So in the topological space induced by the
metric, we can write E o as arbitrary union of open intervals Gα where α ∈ A
where A is an index set. So, E o = α∈A Gα by De-Morgan’s law,
S
\
(E o )c = Gcα .
α∈A

Since Gα is open by definition Gcα is closed. Observe that Gα ⊂ E 0 ⊂ E for all


α ∈ A and so, E c ⊂ Gcα for all α ∈ A. Therefore Gcα is a closed set that contains
E c and α∈A is the smallest closed set containing E c and by definition is the
T

closure of Ec . Hence we conclude that the complement of E o is the closure of the


complement of E.
(5) NO! Consider the set E = (−1, 0) ∪ (0, 1) in R with usual metric. The closure,
E = [−1, 1]. Observe that 0 is not an interior point of E because 0 ∈/ E. But 0 is
an interior point of E.

35
Problem 5.20. Show that Rk is separable.

Solution. We want to show that there is a countable dense subset of Rk . Consider the
subset Qk .

Claim1: Qk is countable.
Since finite Cartesian product of countable subset is countable we immediately have Qk
is countable.
Claim 2: Qk is dense in Rk .
Let x ∈ Rk be any point of the form x = (x1 , x2 , . . . . . . , xk ). We want to show that every
ball centered at x contains a point of Qk . For any  > 0 consider the ball B (x). Since Q
2 2 2
is dense in ≈x≈ there is a rational number ξj in (xi − k , xi + k ) and so |xi − ξi | < k
for all 1 ≤ i ≤ k. Let y = (ξ1 , ξ2 , . . . . . . , ξk ) ∈ Qk . Then,
v
u k r
uX 2
d(x, y) = t (xi − ξi )2 ≤ k = .
i=1
k

This implies there any  ball around x contains a point of Qk and hence Qk is dense in
Rk

36
Problem 5.21. Prove that every separable metric space has a countable base.

Solution. Let A = {xi }∞ i=1 denote a countable dense subset of X. For each n ∈ N and
r ∈ Q+ let Vn,r = Br (xn ). Note that {Vn,r } is countable.

Consider any nonempty open set G ⊂ X. For each x ∈ G, there exists x > 0 such
that Bx ⊂ G. The open ball B 2x (x) contains a point xm for some m, and there exists
r ∈ Q+ such that r < 2x implying Br (xm ) ⊂ Bx ⊂ G. Thus, G can be written as the
union of balls of rational radii centered at the points of a countable dense subset. This
implies {Vn,r } is a countable base for the metric-topology.

Problem 13.5 A metric space (X, d) is separable if and only if it is second countable.

Solution: Since a metric space is also a topological space, the above shows that separa-
ble implies second countable. Now we will prove that if X is second countable then it is
separable.
Since X is second countable, by definition, the metric topology has a countable base
S
B = Gα with Gα 6= ∅. Now form a subset E taking one element from each Gα . So E is
countable. Let x ∈ X be any point and for any  > 0, B be a neighborhood of x. Since
B (x) is open, by the definition of countable base there is a collection aα ⊂ B such that
S
B (x) = α Gα . Therefore B (x) contains a point of E and hence E is a countable dense
subset of X and therefore X is separable.

37
Problem 5.22. Prove that any uncountable set with the discrete metric is not second
countable.

Solution. Let (X, d) be an uncountable metric space with the discrete metric. Every
point of X is also an open set, so every point is a neighborhood. So the metric topology
is uncountable and hence does not have a countable base implying that it is not second
countable.

38
Problem 5.23. Show that a Schauder basis (fn )∞
n=0 of C([0, 1]) may be constructed from
”tent” functions.

39
Problem 5.24. Let E be the set of all x ∈ [0, 1] whose decimal expansion contains only
the digits 4 and 7. Is E countable? Is E dense in [0, 1]? Is E compact? Is E perfect?

Solution. (1) E is not countable.


To see this we use same argument as cantor diagonalize argument. Assume that
E is countable. Then we have an ordering of the elements of E : {xn }n∈N . Write
out the decimal expansion of xn as y1n y2n . . . . . . yjn . . . where yjn represents the jth
digit of xn . Let z have the decimal expansion given by zj = 4 if yjj = 4 and zj = 7
if yii = 4. Then z ∈ E but z 6= xn for any n, contradiction.
(2) E is not dense in [0, 1]
0 ∈ [0, 1] however B0.1 (0) does not contain any point of E because B0.1 (0) =
(0, 0.1) and all the point inside the ball has decimal expansions beginning with a
0 and thus are not in E. Therefore 0 is not a limit point of E so E is not dense
in [0, 1].
(3) E is compact:
If x ∈ / [0, 1] then r = min{d(x, 0), d(x, 1)} > 0 then Nr (x) ∩ E = ∅. Now suppose
x ∈ [0, 1] \ E. Then the decimal expansion of x has some digit which is not a 4 or
a 7. Let d1 d2 . . . . . . denote the decimal expansion of x. Let k be the lowest index
such that dk 6= 4 or 7.
1
Now let r = 10k+2 . Then Br (x) only contains elements which either have dk as
their k digit or they have 9 as their k + 1th digit. Both of these conditions ensure
th

that the element of Br (x) is not an element of E. Therefore x is not a limit point
of E. Therefore E contains all of its limit points and is closed.
Since E ⊂ [0, 1], it is bounded. By Heine-Borel theorem E is compact.
(4) E is not perfect:
The answer to this question kinda depends on how one interpret E. If it is allowed
to have points like 0.4 = 0.4000000 . . . which can be written with or without 0’s
then E is not perfect because 0.4 is not a limit point because B0.1 (0.4) does not
contain any points of E except for 0.4 because every point in (0.39, 0.41) either
has a 3 as its first digit or has a 0 as its second digit and has non-zero digits
following that 0 if it is not exactly 0.4. Therefore E is not perfect.

On the other hand, if you interpret this question such that 0.4000 . . . is not in-
cluded in E and the only elements in E are infinite expansions where all of the
infinite decimal places are either 4 or 7 then every point in E is a limit point
because:

For every  > 0 there exists an N such that 101N < . Then for a given x ∈ E we
want to find a y ∈ E such that y ∈ B (x) and y 6= x. We construct such a y by
letting its first N digits be the same in x. Then let the N + 1st digit be different
from x(if x has a 4 then y has a 7 and vice versa) and let all the other digits be
40
same. Then
3 1
|x − y| = < <
10N +1 10N
so y ∈ B (x) so since  was arbitrary, x is a limit point of E. So this proves that
all the points of E which are infinite sequence of 4’s and 7’s are limit points of E
so if this is all of E then E is perfect.

41
Problem 5.25. Below, A and B are sets in metric space (X, d).
(a) If A and B are disjoint closed sets prove that they are separated.
(b) Fix p ∈ X, δ > 0, define A = {q ∈ X : d(p, q) < δ} and B = {q ∈ X : d(p, q) > δ}.
Prove that A and B are separated.
(c) Prove that every connected space with at least two points is uncountable.

Solution. (1) Since A and B are closed, A = A and B = B. Also given that
A ∩ B = ∅. So, A ∩ B = A ∩ B = ∅ and A ∩ B = A ∩ B = ∅. Hence A and B are
separated.
(2) Note that A and B are disjoint open sets. Let E = B c = X \ B. Since E is closed
and A ⊂ E, by the definition that closure is the smallest closed subset containing
the set we have A ⊂ E. Since E ∩ B = ∅ we have A ∩ B = ∅. Similarly we can
show that A ∩ B = ∅. Therefore, A and B are separated.
(3) Consider x, y ∈ X with x 6= y, and d(x, y) > 0. For δ ∈ (0, α), since X is
connected there exists z ∈ X such that d(x, z) = δ or else X is separated by (a)
and (b) with point p = x or y. Thus there exists some subset of X that can be put
with one -one correspondent with [0, α] ⊂ R. Which implies X is uncountable.

42
Problem 5.26. Let A and B be separated subsets in Rn , suppose a ∈ A and b ∈ B and
define p(t) = (1 − t)a + tb for t ∈ R. Put A0 = p−1 (A) and B0 = p−1 (B) (thus t ∈ A0 if
and only if p(t) ∈ A).
(1) Prove that A0 and B0 are separated subsets of R.
(2) Prove that there exists t0 ∈ (0, 1) such that p(t0 ) ∈
/ A ∪ B.
(3) Prove that every convex subset of Rk is connected.

Solution. 6 ∅ because 0 ∈ A0
(1) We prove by contradiction. First observe that A0 =
as a ∈ A and also 1 ∈ B0 for b ∈ B implies B0 6= ∅. Assume that A0 and B0 are
not separated then one of the followings is true

A0 ∩ B0 6= ∅

A0 ∩ B0 6= ∅
A0 ∩ B0 6= ∅
First we take that A0 ∩ B0 6= ∅. Let t0 ∈ A0 ∩ B0 which implies t0 ∈ A0 and
t0 ∈ B0 . By the definition of the given function t0 ∈ A0 ⇒ p(t0 ) ∈ A and
t0 ∈ B0 ⇒ p(t0 ) ∈ B. So A ∩ B 6= ∅ which contradicts the fact that A and B are
separated. So, we can draw a conclusion that A0 and B0 are disjoint.

Now assume that A0 ∩ B0 6= ∅. Let t0 ∈ A0 ∩ B0 . Then t0 ∈ B0 and t0 is a limit


point of A0 and so there is a sequence (tn ) in A0 that converges to t0 . That is for
given  > 0 there exists an N ∈ N such that |tn − t0 | < 2c where c = max(kak, kbk)
whenever n ≥ N . Now construct a sequence p(tn ) in A. We want to show that
p(tn ) converges to p(t0 ).

For given  > 0, n ≥ N implies,


kp(tn ) − p(t0 )k =k(1 − tn )a + tn b − (1 − t0 )a − t0 bk
≤k(t0 − tn )ak + k(tn − t0 )bk
=kak |tn − t0 | + kbk |tn − t0 |
 
≤kak + kbk
2c 2c
 
≤ +
2 2
=

Therefore p(tn ) → p(t0 ) and so p(t0 ) ∈ A. Together with p(t0 ) ∈ B we conclude


that p(t0 ) ∈ A ∩ B and hence A ∩ B 6= ∅. Which is a contradiction to the fact
that A and B are separated. A0 ∩ B0 = ∅
Similarly, we can show that A0 ∩ B0 = ∅. Hence A0 and B0 are separated.
(2) We prove by contradiction. Assume p(t) ∈ A ∪ B for all t ∈ [0, 1], then [0, 1] ⊂
A0 ∪B0 . Let E = [0, 1]∩A0 and F = [0, 1]∩B0 . None that E and F are nonempty
43
since 0 ∈ A0 and 1 ∈ B0 . Also, [0, 1] = E ∪ F , but [0, 1] is connected whereas E
and F are separated by part (a), a contradiction.
(3) By the definition of convex set if E is convex then for all t ∈ [0, 1], p(t) ∈ E. If E
is not connected then there are non-empty subsets A, B ⊂ E such that E = A ∪ B
and A and B are separated set. According part (b) there exists t0 ∈ (0, 1) such
that p(t0 ) ∈
/ A ∪ B = E which is contradiction to the definition of convex set.

44
Problem 5.27. Give an example of matrix space that is separable but not second count-
able.

Solution. The space R` defined by the topology to be the collection of set of the form
[a, b). R` is first countable because for any element x ∈ R consider the basis [x, n1 ) which
is countable. Also the set of rational number is dense in R` which is countable and hence
R` is separable. But we will show that R` is not second countable.
Let B be a basis for R` . Choose for each x, an element Bx ∈ B such that x ∈ Bx ⊂
[x, x + 1). If x 6= y, then Bx 6= By , since x = inf Bx and y = inf By . Therefore, B must
be uncountable.

Problem 5.28. Any open set in R can be written as countable union of open intervals.

Solution. Suppose O be an open subset of real numbers. For each rational number r,
define the interval Ir to be the largest open interval containing r that is a subset of O (it is
possible of course that if r is not in O then Ir is empty, so in this case we simply ignore this
r). More particularly, if we define ar = inf{x : [x, r] ⊆ O} and br = sup{x : [r, x] ⊆ O},
then Ir = (ar , br ). Since the rationals are a countable set, the collection {Ir : r ∈ Q} is a
countable collection of intervals.
Of course they need not be disjoint. However it is true that if s is rational and x ∈ Ir
then Is = Ir . This follows by noting that since Is is the largest open interval containing
s that is a subset of O, and since s ∈ Ir , an open interval that is a subset of O, Ir ⊂ Is .
Since then we have that r ∈ Is the same argument shows that Is ⊆ Ir . It is also true that
if s and r are rational and s ∈ / Ir then Is ∩ Ir = ∅. To see this suppose that x ∈ Is ∩ Ir .
Since Is and Ir are both open intervals, so is Ir ∪ Is . Since Ir is the largest open interval
containing r and a subset of O, Ir ∪ Is ⊆ Ir . But this implies that s ∈ Ir contradicting
our hypothesis. Therefore, we conclude that Is ∩ Ir = ∅. Together, this implies that given
two intervals Is and Ir that they are either disjoint or identical.
By removing duplicates in {Ir : r ∈ A}, we arrive at a sequence of disjoint intervals
S∞ S∞
{Ik }∞
S
k=1 such that r∈Q Ir = k=1 Ik . It remains only to show that O = k=1 Ik . Since
S∞
by construction, each Ik ⊆ O, k=1 Ik ⊆ O. Now let x ∈ O. Since O is open there is a
natural number n such that (x − n1 , 1 + n1 ) ⊆ O. By the density or the rationals in the
reals, there is a rational number r ∈ (x− n1 , x+ n1 ), and so by construction the largest open
interval containing r that is a subset of O will have to include the interval (x − n1 , x + n1 ).
Hence x ∈ Ir for this r, and so x ∈ ∞
S S∞
k=1 Ik , and we conclude that O = k=1 Ik .

45
6. Numerical Sequences and Series

6.1. Sequences

(1) Suppose {an } and {bn } are sequences of real numbers such that an ≤ bn for all
n ≥ N and some V ∈ N. If an → a and bn → b then a ≤ b
(2) Let {an } be a sequence such that an → a. If a > 0, then an > 0 for all n > N
and some N ∈ N
(3) Suppose an ≤ bn ≤ cn ∀n ≥ N and some n ∈ N. If an → l and cn → l then
bn → l

Theorem 6.1. Let {sn } and {tn } be two sequences of complex number such that sn → s
and tn → t for some complex number s and t. Let z be any complex number. Then,
(1) sn + tn → s + t
(2) zsn → zs
(3) sn tn → st
(4) s1n → 1s

Theorem 6.2. Let {xn } be a monotonic sequence. {xn } is convergent if and only if it is
bounded.

Definition 6.1. Let {xn } be a sequence in R.


We say {xn } converges to ∞ and we write xn → ∞ if and only if given any M > 0 there
is n ∈ N such that xn > M for al n > N .
We say {xn } converges to −∞ and we write xn → −∞ if and only if given any M > 0
there is n ∈ N such that xn < M for al n > N .

Definition 6.2. {xn } be a sequence of real numbers, bn = sup{xk |k ≥ n}if {xn } is


bounded above and an = inf{xk |k ≥ n} if {xn } is bounded bellow. Note that bn and an
are non increasing and non decreasing sequences and lim bn and lim an exist in R.
n→∞ n→∞

 lim bn if {xn }isboundedabove
lim sup xn = n→∞
n ∞ otherwise

Theorem 6.3. Let {xn } be a sequence of real numbers. Let E be the set of all y in R
such that y is a limit of a subsequence of {xn }. Then,

lim sup xn = sup Eand lim inf xn = inf E

Here inf E and sup E are in the extended metric R

Theorem 6.4. Let {xn } be a sequence of real numbers. xn → x for some x ∈ R if


lim sup xn = x = lim inf xn

Theorem 6.5. If {xn } and {yn } are sequences in R such that xn ≤ yn for all n ≥ N
and some N ∈ N, then,
46
(1) lim sup xn ≤ lim inf yn
(2) lim inf xn ≤ lim inf yn

Theorem 6.6. (1) If p ≥ 0 then lim n1p = 0


√ n→∞
(2) If p > 0 then lim n p = 1
√ n→∞
(3) lim n n = 1
n→∞

(4) If p > 0 and α is a real number, then lim (1+p)n =0
n→∞
n
(5) If |x| < 1, then lim |x| = 0
n→∞
(6) If |x| > 0 then lim |x|n = ∞ if x > 1 and does not exists if x ≤ −1
n→∞

6.2. Series

Definition 6.3. Let {an } be a sequence of complex numbers. The formal sum a1 + a2 +
a3 + . . . · · · + an + . . . . . . which can be written as ∞
P
k=1 ak is called an infinite series or
simply a series.

With the above infinite series we associate another sequence called the sequence of
partial sums of the series denoted by {sn } and defined by sn = nk=1 .
P

We say the series ∞


P
k=1 ak converges to a real number s or has the sums iff the associated
sequence {sn } of partial sum converges to s. Otherwise that is if {sn } diverges we say
that the series ∞
P
k=1 ak diverges.

Theorem 6.7. The series ∞


P
n=1 an converges if and only if for every  there is a positive
integer N such that if m > n > N then | m
P
j=n aj | < .
P
Theorem 6.8. If an converges then an → 0

Note that this theorem doesn’t day if an → 0 then the sum converges rather the contra-
positive is true that is if an doesn’t tends to zero then the sum doesn’t converge.

Theorem 6.9. A series of non negative terms converges if and only if its sequence of
partial sums is bounded.

Theorem 6.10 (Comperison Test). (1) Let |an | < cn , ∀n ≥ N0 , where N0 is some
P P
fixed integer, and if cn converges, then an converges.
P P
(2) Let an ≥ dn ≥ 0, ∀n ≥ N0 , and, if dn diverges, then an diverges.

Theorem 6.11. The series ∞ k


P
k=0 x is convergent if and only if |x| < 1. If |x| < 1 then
P∞ k 1
k=0 x = 1−x

Theorem 6.12 (Cauchy Condensation Test). Suppose {an } is a non increasing sequence
of nonnegative terms then ∞ n
P
n=1 an converges if and only if 2 a2n converges.
P 1
Theorem 6.13. np
converges if p > 1 and diverges if p ≤ 1
47
Theorem 6.14. If P > 1,

X 1
n=2
n (log n)p
converges; if p ≤ 1, the series diverges.
P √
Theorem 6.15 (Root Test). Given an , put α = lim sup n an . Then
n→∞
P
(a) if α < 1, an converges;
P
(b) if α > 1, an diverges;
(c) if α = 1, the test gives no information.
P
Theorem 6.16 (Ratio Test). The series an

an+1
(a) converges if lim sup an < 1,
n→∞
(b) diverges if an+1 ≥ 1 for n ≥ n0 , where n0 is some fixed integer.

an

Definition 6.4. Given a sequence {cn } of complex numbers, the series



X
cn z n ,
n=0

is called a power series. The numbers cn are called the coefficients of the series; z is a
complex number.

cn z n , put
P
Theorem 6.17 (Radius of convergence). Given the power series
p 1
α = lim sup n |cn |, R = ,
n→∞ α
cn z n converges if |z| < R and
P
(If α = 0, R = +∞; if α = +∞, R = o.) Then
diverges if |z| > R

Theorem 6.18 (Summation by Parts). Given two sequences {an } and {bn }, put
n
X
An = ak
k=0

if n ≥ 0; put A−1 = 0. Then if 0 ≤ p ≤ q, we have


q q−1
X X
an b n = An (bn − bn+1 ) + Aq bq − Ap−1 bp
n=p n=p

Theorem 6.19. Suppose


P
(a) the partial sums An of an form a bounded sequence;
(b) b0 ≥ b1 ≥ b2 ≥ . . . . . . i.e. ({bn } is a decreasing sequence);
(c) lim bn = 0.
Pn→∞
Then an bn converges.

Theorem 6.20 (Alternating series test). Suppose


(a) |c1 | ≥ |c2 | ≥ |c3 | ≥ . . . i.e. {|cn |} is a decreasing sequence
48
(b) c2m−1 ≥ 0, c2m ≥ 0, m = 1, 2, 3 . . . . . . , i.e. alternating series
(c) lim cn = 0.
n→∞
P
Then cn converges.

7. Rudin’s Solution:

Problem 7.1. Prove that convergence of {sn } implies convergence of {|sn |}. Is the
converse true?

Solution. Let  > 0 be given. Since {sn } converges, it is a Cauchy sequence and so there
exists natural number N such that n, m ≥ N implies |sn − sm | < . We have

||sn | − |sm || ≤ |sn − sm | < 

for all n, m ≥ N . Hence the sequence {|sn |} is also a Cauchy sequence and therefore
converges in the complete metric R.
√ 
Problem 7.2. Calculate lim n2 + n − n
n→∞

Solution.
√ n 1
n2 + n − n = √ =q
n2 + n + n 1 + n1 + 1
1
It follows that the limit is 2

Problem 7.3. If s1 = 2, and

q
sn+1 = 2+ sn , (n = 1, 2, 3, . . . . . . )

prove that {sn } converges, and that sn < 2 for n = 1, 2, 3, . . . . . .


√ √
Solution. S1 = 2 < 2. Assume that sn < 2 then sn+1 < 2 + 2 = 2. Therefore by
mathematical induction sn < 2 for all n = 1, 2, 3, . . . . . . .
√ p √
It is obvious that s1 = 2 < 2 + s1 = s2 . Now assume that s1 < s2 < . . . · · · < sn < 2
then,

√ √
q q
sn = 2 + sn−1 < 2 + sn = sn+1
So by mathematical induction {sn } is an increasing sequence.
{sn } being increasing and bounded sequence of nonnegative terms it converges.

Problem 7.4. Find the upper and lower limits of the sequence {sn } defined by
s2m−1 1
s1 = 0, s2m = , s2m+1 = + s2m .
2 2
Solution. Construction: We compute some of the terms of the sequence
1 3 3 7 7
s1 = 0, s2 = 0, s3 = , s5 = , s6 = , s7 = , s8 = .
4 4 8 8 16
49
We see the sequence have two subsequences corresponding to the even and odd numbered
terms
1 3 7 15 2m + 1 1
0, , , , , . . . · · · ≡ s2m−1 = m
=1− m
2 4 8 16 2 2
1 3 7 15 2m − 1 1 1
0, , , , ≡ s2m = m+1 = − m
4 8 16 32 2 2 2
Solution: We shall prove by induction that
1 1 1
(7.1) s2m = − m, s2m+1 = 1 −
2 2 2m
for m = 1, 2, . . . . . . . Immediate computation shows that s2 = 0 and s3 = 12 . Now assume
that the both formulas hold for m ≤ r. Then
 
1 1 1 1 1
s2r+2 = s2r+1 = 1 − r = − r+1
2 2 2 2 2
Therefore the first of (7.1) relations is true for all m and to see the second we use the
given equation
1 1 1 1 1
s2m+1 = + s( 2m) = + − m = 1 − m
2 2 2 2 2
1
So the subsequence of even terms converges to 2 and the subsequence of the odd terms
converge to 1.
Now we want to show that there are no other sub-sequential limits. For any x other than
1 and 21 , choose r = 12 min{|x − 1| , |−1/2|}. Note that sn ∈ Br (x) for at most finitely
many n ∈ N. Thus 21 and 1 are the only subsequential limits of {sn }, so therefore,
1
lim sup sn = 1, lim inf sn = .
n→∞ n→∞ 2
Problem 7.5. For any tow real sequences {an } and {bn } prove that

lim sup(an + bn ) ≤ lim sup an + lim sup bn


n→∞ n→∞ n→∞

provided the sum of the right is not the form ∞ − ∞

Solution. Since the um of the right hand side is not of the form ∞−∞, if one term is −∞
then both of the term have to be bounded above by M1 (say). Let M be any real number.
Without loss of generality assume that lim sup an = −∞ which implies that there is a
n→∞
subsequence {ank } such that ank → −∞ that is for every M ∈ R there is an integer N1
such that nk ≥ N1 implies ank ≤ M − M1 . Therefore ank + bnk ≤ M − M1 + M1 = M .
Which implies that ank + bnk → −∞ and so lim sup(an + bn ) = −∞.
n→∞
Now suppose one of the term in the right side is ∞ then other term has to be bounded bel-
low by M1 (say). Without loss of generality assume that lim sup an = ∞. So, there exists
n→∞
a subsequence {ank } of {an } such that ank → ∞. Let M be any given real number. There
exists n ∈ N such that ank ≥ M −M1 for all nk ≥ N . Hence ank +bnk ≥ M −M1 +M1 = M .
So there is a subsequence of ank + bnk → ∞ and hence lim sup(an + bn ) = ∞.
n→∞
Now assume that both terms in the right are finite. Suppose lim sup an = α and
n→∞
50
lim sup bn = β. Fix any  > 0. There exist natural numbers N1 and N2 such that
n→∞

n ≥ N1 implies an < α + 2
and n ≥ N2 implies bn < β + 2 . Let N := max{N1 , N2 }.
Then n ≥ N implies
an + b n < α + β + 
Since  > 0 is arbitrary, therefore,

an + bn ≤ α + β = lim sup an + lim sup bn


n→∞ n→∞

for all but finitely many n. Thus every sub-sequential limit of {an + bn } is bounded above
by lim sup an + lim sup bn , and in particular,
n→∞ n→∞

lim sup(an + bn ) = lim sup an + lim sup bn


n→∞ n→∞ n→∞
P
Problem 7.6. Investigate the behavior of an if
√ √
(a) an = √ n + 1 − n;

(b) an = n+1− n
n
;
√ n
(c) an = ( n − 1) ;
n

1
(d) an = 1+z n , for complex values of z.

Solution. (a)
√ √ √ √
( n + 1 − n)( n + 1 + n) 1 1
an = √ √ = √ √ ≥ √
( n + 1 + n) ( n + 1 + n) 2 n+1
P 1
√ diverges and so 1 √ 1 . By the comparison test
P
We know that the series n 2 n+1
P
the given series an diverges.
(b)
1 1 1
|an | = √ √ ≤ √ ≤ 3/2
n( n + 1 + n) n2 n n
P
By comparison test an converges.
√ √ √
(c) We use the root test. n an = n n − 1 and thus lim sup n an = 0 < 1. So by the
n→∞
root test the series converges.
(d) If |z| ≤ 1, then |an | ≥ 12 , so that an does not tend to zero and hence
P
an
1 1 1 1
diverges. If |z| > 1 then |an | = 1+zn ≤ zn = ( z )n . Note that |z| < 1 and so
P 1 n P
z
converges and by the comparison test an converges.
P P √an
Problem 7.7. Prove that the convergence of an implies the convergence of n
, if
an ≥ 0.
√ 2
Solution. Since an − n1 ≥ 0, it follows that
√    
an 1 1 1 1

n 2 an + n 2 = 2 an + n 2

P P 1
Since an and n2
both converges we conclude by the comparison test that the series
P √an
n
converges.

51
P
Problem 7.8. If an converges, and if {bn } is a monotonic and bounded, prove that
P
an bn converges.

Solution. Since {bn } is a bounded monotonic sequence it converges to b(say). Define,



b − b if {b }ismonotonically increasing.
n n
cn =
bn − b if {bn }ismonotonically decreasing.
P
Then {cn } is monotonically decreasing sequence and lim cn = 0. Therefore an cn con-
verges.
If {bn } is monotonically decreasing, we have, for all N ∈ N
N
X N
X N
X
an b n = an c n + b an .
n=1 n=1 n=1

Similarly if {bn } is increasing we have, for all N ∈ N,


N
X N
X N
X
an b n = an c n − b an .
n=1 n=1 n=1
P
In both cases, both the term in the right side converge and hence an bn converges.

an z n are integers,
P
Problem 7.9. Suppose that the coefficients of the power series
infinitely many of which are distinct form zero. Prove that the radius of convergence is
at most 1.

Solution. Since cn ≥ 1 which implies n cn ≥ 1 for all but finitely many n. So, α =
lim sup ≥ 1 and therefore the radius of convergence R = α1 ≤ 1
n→∞
P
Problem 7.10. Suppose an > o, sn = a + 1 + · · · + an , and an diverges,
P an
(a) Prove that 1+an
diverges.
(b) Prove that sN +1 +. . . · · ·+ asNN+k
aN +1
≥ 1− sNsN+k and hence deduce that
+k
P an
s2n
converges.
an 1 1
P an
(c) Prove that s2 ≤ sn−1 − sn and deduce that s2n
converges.
n
(d) What can be said about
X an X an
, and
1 + nan 1 + n 2 an
P an
Solution. (a) We assume to the contrary that 1+an
converges. That means that
an
1+an
→ 0 which implies 1 +1 → 0 which is possible only if a1n → ∞ ⇔ an → 0.
1
an
So there exists an integer N such that an < 1 for all n ≥ N . Then,
an a2n an
an − = <
1 + an 1 + an 1 + an
which implies
an
0 < an <
.
1 + an
P P an
So, by comparison test an converges which is a contradiction and hence 1+an
does not converge.
52
(b) Since an > 0 we must have sN +k ≥ sN +i for all 1 ≤ i ≤ k and therefore
aN +1 aN +k aN +1 + . . . . . . aN +k
+ ...··· + ≥
sN +1 sN +k sN +k
sN +k − sN
=
sN +k
sN
=1 −
sN +k
Since sn does not converge for a fixed N we can choose k large enough so that
NP+k
sN 1 ak
sN +k
≥ 2
and so the partial sum sk
≥ 12 which is true for any choice of N .
k=N +1
Therefore the sequence of partial sum of { asnn } does not form a Cauchy sequence
and so does not converge.
(c) We observe that for n ≥ 2,
1 1 sn − sn−1 an an
− = = ≥ 2
sn−1 sn sn sn−1 sn sn−1 sn
Also,
∞  
X 1 1 1 1 1
− = − = .
n=2
sn−1 sn s1 s∞ a1
By the comparison test since

an
≤ 1 − 1
s2 sn−1 sn
n
P an
and the series of terms in the right side converges s2n
also converges.
P an
(d) The series 1+nan
may be either divergent or convergent.
If the sequence {nan } is bounded above that is if there exists M ∈ R such that
nan ≤ M for all n then

an 1
1 + nan ≥ 1 + M an

P P an
Since an diverges, by comparison test 1+nan
also diverges.
If {nan } is bounded bellow that is if there exists  > 0 such that an ≥  for all n,
then,
nan 
nan + nan ≥  + nan ⇒ ≥
1 + nan 1+
which implies,
an  1
1 + nan ≥ 1 +  n .

Again, by comparison test the series does not converge.


P an
However the series 1+nan
converges. For example let an = n12 if n is not a
perfect square and an = √1n if n is a perfect square. For a nonsquare term,
an 1
= 2
1 + nan n +n

53
also for a square integer n,
an 1
=√
1 + nan n + n2

an
in both of the cases 1+na n
≤ n12 and hence by comparison test the series con-
verges.

For the second series,



an 1 1
1 + n2 an =

2
≤ 2.
1
+n an
n
P an
Hence by comparison test, the series 1+n2 an
converges.

P
Problem 7.11. Suppose an > 0 and an converges. Put

X
rn = am .
m=n

(a) Prove that


am an rn
+ ...··· + >1−
rm rn rm
P an
if m < n, and deduce that rn
diverges.
(b) Prove that
an √ √
√ < 2( rn − rn+1 )
rn
P an
and deduce that √
rn
converges.

ak ak
Solution. (a) Obviously rm > rk for all k > m since an > 0. Therefore rk
> rm
.
am an am + · · · + an rm − rn rn
+ ...··· + > = =1− .
rm rn rm rm rm
P P
Since the series an converges and suppose an = s, then the sequence of partial
sum sn converges to s. Fix an m ∈ N, there exists N such that |s − sn−1 | < r2m
for all n > N . But
rm rn 1
|s − sn−1 | < ⇒ < .
2 rm 2
n
an
> 21 . Hence
P P an
Therefore, for any m we can find an n such that rn rn
is not
k=m
convergent.
(b) √
an √ √ rn+1
√ ( rn + rn+1 ) = an + an √ < 2an = 2(rn − rn+1 )
rn rn
Therefore,
an √ √
√ < 2( rn − rn+1 ).
rn
54
P
Since an convergent the sequence of partial sum sn → s and so rn → 0 as
∞ √ √
P √ P an
n → ∞. Therefore ( rn − rn+1 ) = r1 . Hence the series √
rn
converges
n=1
by the comparison test.

Problem 7.12. Suppose {pn } is a Cauchy sequence in a metric space X, and some
subsequence {pn } converges to a point p ∈ X. Prove that the full sequence {pn } converges
to p.

7.1. Problems from other sources

Problem 7.13. Prove that every convergent sequence has a monotone subsequence.

Solution. Consider the convergent sequence an that converges to a. So there is an


integer N1 such that |an − a| < 1 for all n ≥ N1 . Select the b1 = max{a1 , . . . aN1 −1 }.
By induction there is an integer Nj such that |an − a| < 1j for all n ≥ Nj . Select
bj = max{aNj−1 , . . . . . . aNj−1 }. By construction {bn } is a decreasing subsequence of {an }.

Theorem 7.1 (Leibniz’s Rule for Alternating Series). If {an } is a monotonic decreasing

(−1)n+1 an converges. If S denotes
P
sequence with limit 0, then the alternating series
n=1
its sum and sn denotes its nth partial sum, we also have the inequalities

0 < (−1)n (S − sn ) < an+1

Proof 2. The subsequence of partial sum s2k is increasing because

s2k+2 − s2k = a2k+1 − a2k+2 > 0.

Similarly the subsequence of partial sums s2k−1 is decreasing, because,

s2k+1 − s2k−1 = −a( 2k) + a2k+1 < 0.

Observe that,

s2k = a1 − (a2 − a3 ) − . . . . . . (a2k−2 − a2k−1 ) − a2k < a1

and so the sequence {s2k } is also bounded above by a1 . Therefore s2k converges. Now

s2k−1 = s2k + a2k

and it follows that


lim s2k−1 = lim s2k + lim ak = lim s2k .
k→∞ k→∞ k→∞ k→∞
Since {s2n } and {s2n−1 } converges tot he same limit, sn converges.
Since s2k is increasing and s2k−1 is decreasing, we have

s2k < s2k+2 ≤ S, and S ≤ s2k+1 < s2k−1 , ∀k ≥ 1.

We have,
0 < S − s2k ≤ s2k+1 − s2k = a2k+1 ⇒ |S − s2k | ≤ a2k+1
55
and,
0 < s2k−1 − S ≤ s2k−1 − s2k = a2 k ⇒ |S − s2k−1 | ≤ a2k .
Putting these last two equations together gives the result

|S − sn | ≤ an+1 , ∀n ∈ N.

Problem 7.14. Give an example of a sequence of positive numbers so that an → ∞, but



(−1)n+1 an diverges.
P
the series
n=1

Solution. Consider the sequence



1 if n is odd
n
an =
0 if n is even
Obviously,
X X1
an =
n
which does not converge.
Another example could be, 
1 if n is odd
n
an =
 12 if n is even
n

56
8. Compactness

In this section we define compactness of a subset of a metric space. For subsets of real
number compactness is simply being closed and bounded. We start with definitions and
theorems.
Sequentially Compact: A subset K of a metric space (X, d) is sequentially compact if
every sequence in K has a convergent subsequence whose limit belongs to K. If K = X,
then X is said to be sequentially compact and every sequence in X has convergent sub-
sequence.

Theorem 8.1 (Heine-Borel Theorem). A subset of Rn is sequentially compact if and only


if it is closed and bounded.

Theorem 8.2 (BolzanoWeierstrass theorem). Every bounded sequence in Rn has a con-


vergent subsequence.

Definition 8.1. Let K be a subset of a topological space (X, T ). Let {Gα }α∈Λ denote
S
an arbitrary set of open sets in T . If K ⊂ α∈Λ Gα , then we call {Gα }α∈Λ is an open
cover of A.

Definition 8.2 (Compact Set). If every open cover of K has a finite subcover, then K
is compact.

Definition 8.3 ( − net). Let K be a subset of a metric space (X, d). For a fixed  > 0,
we call {xα }α∈A ⊂ X an -net of K if the family of open balls {B (xα )}α∈A is an open
cover of A. If {xα } is finite, then we call {xα } a finite -net of A.

Definition 8.4. The subset K of a metric space (X, d) is totally bounded if it has a finite
-net for every  > 0.

Theorem 8.3. A subset of a metric space is sequentially compact if and only if it is


complete and totally bounded.

Theorem 8.4. A subset of a metric space is compact if and only if it is sequentially


compact.

Lemma 1. A sequentially compact metric space is separable.

Definition 8.5. A subset K of a topological space (X, T ) is precompact if its closure is


compact.

Theorem 8.5. A closed subset of a compact space is compact.

Corollary 4. Every subset of a compact space is precompact.


57
8.1. Problems

Problem 8.1. Let f : A ⊂ R → R have the property that for every x ∈ A there is an
 > 0 such that
f (t) >  if t ∈ (x − , x + ) ∩ A.
Show that if the set A is compact, then there is some c > 0 such that f (x) > c for all
x ∈ A.
S
Solution. Observe that x∈A (Bx (x)) covers A. Since A is compact there is a finite
subcover, say, ni=1 Bi (xi ). (Here we write xi as i for notational simplicity). In each
S

ball Bi = Bi (xi ) that for each 1 ≤ i ≤ n, we have, f (t) > i if t ∈ (xi − i , xi + i ).
Take c = min{i }ni=1 . Let x be any point in A. x ∈ Bi for some 1 ≤ i ≤ n. Therefore,
f (x) > i ≥ c. So, we conclude that f (x) > c for all x ∈ A.

58
Problem 8.2. Use compactness to show that if a function f : R → R is locally increasing
at every point in R, then f must be increasing. Here, locally increasing at a point x means
there exists δ > 0 such that f (s) < f (x) < f (t) whenever x − δ < s < x < t < x + δ.

Solution.

59
Problem 8.3. Let (X, d) be a metric space. Let {K1 , K2 , . . . , Kn } be a set of compact
subsets of X. Prove that ∪ni=1 Ki is a compact subset of X. Let {Kα } be a set of compact
subsets of X. Prove that ∩α Kα is a compact subset of X.

Solution. Let G = α∈Λ Gα be a cover for Y = ni=1 Ki . Since Ki ⊂ Y , for each


S S
S
1 ≤ i ≤ n, the set α∈Λ Gα ∩ Ki is a cover for Ki . Since each Ki is compact there is a
finite subcover Gi = kj=1 Ki ∩ Gj of Ki for each i. Take Gf = ni=1 Gi . We claim that
Si S

Gf is the finite subcover of Y corresponding to the cover G.

Let x ∈ Y , then x ∈ Ki for some 1 ≤ i ≤ n consequently x ∈ Gi and hence x ∈ Gf .


Therefore Y ⊂ Gf showing that Gf is a cover.

Since each of Gi is a finite subcover, Gf being a finite union of them is also finite collec-
tion of sets that covers Y .

Sn
Hence Y = i=1 Ki is compact.

60
Problem 8.4. Let (X, d) be a metric space with the discrete metric d. What subsets of
X are compact?

Solution. Only the finite subsets in a discrete metric are compact. A finite subset of
discrete metric is complete and also totally bounded because for any  > 0 the  − net is
finite and hence compact.

Now we want to prove that a compact set A in discrete metric is finite. If not, let (xn )
be a sequence such that xn 6= xk for any n 6= k, infiniteness of the set A make it possible
to form such sequence. The sequence (xn ) has no convergent subsequence since only con-
vergence subsequence in discrete metric is the constant sequence which is a contradiction.

Therefore we conclude that only a finite subset in a discrete metric is compact.

61
Problem 8.5. Describe an open cover of the open unit square {(x, y) : 0 < x < 1, 0 < y < 1}
that has no finite subcover.

Solution. The open cover [


G= B1− 1 ((0, 0))
n+1
n∈N
S
has no finite subcover. For an instant suppose it has a finite subcover Gf = n∈A B1− 1 ((0, 0)).
n+1
1
Since A is finite there is a maximum say m. Any point in R2 , 1 − m+2 distance away
from the origin does not contained in Gf and hence Gf can not be a cover.

62
Problem 8.6. Let (xn ) be a sequence in a metric space (X, d) that converges to a limit
x. Show the set {x1 , x2 , x3 , · · · } is compact.

Solution.

63
Problem 8.7. Show that the product of two compact metric spaces with the product
metric is a compact metric space.

64
Problem 8.8. Let a, b ∈ R and a < b. Show that any closed ball in C([a, b]) is not com-
pact with the metric d(f, g) = supx∈[a,b] |f (x) − g(x)| for f, g ∈ C([a, b]). Hint: Sequential
compactness and compactness are the same on a metric space (theorem from lecture 2)
and there are some really nice examples of sequences with no convergent subsequence
within the unit ball centered at zero from which we can easily extend these examples to
any closed ball centered at any point (i.e., continuous function) by simply translating and
scaling these examples as necessary. The so-called “tent” functions of Example 2.13 are
one such sequence. The sequence {sin(2n πx/(b − a))} is another (easy) one (you can also
use {sin(nπx)}). Also, the sequence {([x − a]/b)n } works. None of these sequences con-
verge with respect to this metric even though you can find the pointwise limits. Can you
prove this for each one? What this is saying is that a sequence of continuous functions
taken from a closed and bounded subset of continuous functions on a compact space is
not enough to guarantee that this subset is itself compact. Additionally, we can show that
C([a, b]) is complete (lecture 4), so the issue here is that the space is not totally bounded.
Once we show that C([a, b]) is complete, then any of the sequence examples shows the
space is not totally bounded (why? see a specific theorem in lecture 2).

65
Problem 8.9. Prove that every compact metric space K has a countable base, and that
K is therefore separable. Hint: For every positive integer n, there are finitely many
neighborhood of radius 1/n whose union covers K.

66
Problem 8.10. Theorem 2.36 in Rudin
Prove that if {Kα } is a collection of compact subsets of a metric space X such that the
intersection of every finite subcollection of {Kα } is nonempty, then ∩Kα is nonempty.
(Hint: Try a proof by contradiction by assuming there is some member of {Kα }, say
K1 , such that no point in K1 belongs to every Kα , then create an open cover of K1
using complements of Kα . Show that this becomes false (in R, for example) if the word
“compact” is replaced by either “closed” or by “bounded.”

Solution. Fix a member K1 of {Kα } and put Gα = Kαc . Assume that no point of
K1 belongs to every Kα . Then, there are finitely many indices α1 , α2 , . . . , αn such that
K1 ⊂ Gα1 ∪ . . . · · · ∪ Gαn . So for any x ∈ K1 we have x ∈ ∪ni=1 Gαi and therefore
x∈/ ∩ni=1 Gcαi = ∩ni=1 Kαi . But this means that

K 1 ∩ K α1 ∩ · · · ∩ K αn

is empty, in contradiction to our hypothesis.

Consider Kα = [α, ∞). Let Kα1 , . . . . . . , Kαn be a finite sub-collection and let β =
max(α1 , . . . . . . αn ) then [β, ∞) ⊂ Kαi for all i = 1, . . . , n and hence the intersection
of any subcollection is not empty. But ∩Kα = ∅. If not let β ∈ ∩Kα , and let r > β where
r ∈ R. Observe that β ∈ / Kr which is a contradiction. Hence ∩Kα is empty.

Consider the bounded Kn = (0, n1 ). Any finite intersection Kn1 ∩ · · · ∩ Knm is not empty
because it contains Km+1 where m = max{n1 , . . . . . . nm }. Let 0 < x < 1 is in the in-
tersection ∩Kn . There is m such that m1 < x and hence Km does not contain x. Hence
∩Kn = ∅.

Corollary 5. If {Kn } is a sequence of nonempty compact sets such that Kn ⊃ Kn+1 (n =


1, 2, 3 . . . ), then ∩∞
1 Kn is not empty.

67
Problem 8.11. Let E be the set of all x ∈ [0, 1] whose decimal expansion contains only
the digits 4 and 7. Is E compact?

Solution. E is not countable:


Assume E is countable. Then we have an ordering of the elements of E: {xn }n∈N . Write
out the decimal expansion xn as dn1 dn2 . . . . . . . Then let z have decimal expansion given
by zi = 4 if dii = 7 and zi = 7 if dii = 4. Then z ∈ E but z 6= zn for any n, contradiction.

E is not dense in [0, 1]:


0 ∈ [0, 1] however B0.1 (0) ∩ E = ∅ because B[ 0.1](0) = (0, 0.1) and all of these have
decimal expansions beginning with a 0 and thus are not in E. Therefore 0 is not a limit
point of E so E is not dense in [0, 1].

E is compact:
First we claim that E is closed. If x 6= [0, 1] then let r = min{d(x, 0), d(x, 1)} > 0 then
Br (x) ∩ E = ∅. Now suppose x ∈ [0, 1] \ E. Then the decimal expansion of x has some
digit which is not a 4 or a 7. Let d1 , d2 , . . . . . . denote the decimal expansion of x. Let
1
k be the lowest index such that dk 6= 4 or 7. Now let r = 10k+2 . Then Br (x) only
contains elements which either have dk as their k th digit or they have 9 as their n + 1st
digit. Both of these conditions ensure that the element of Br (x) is not an element of E.
Therefore x is not a limit point of E. Therefore E contains all of its limit points and is
closed.
Since E ⊂ [0, 1], it is bounded. By Heine-Borel theorem E is compact.

E is not perfect because B0.01 (0.4) does not contain any point of E other than 0.4 and
hence 0.4 is not a limit point.

68
Problem 8.12. Let X be a metric space in which every infinite subset has a limit point.
Prove that X is separable and compact. Hint: Once we have that X is separable, then
from the lecture 1 examples, we have the existence of a countable base, so any open cover
of X has a countable subcover. Now prove by contradiction assuming that the countable
subcover does not have a finite subcover.

69
Problem 8.13. (a) If X is a compact metric space and if {pn } is a Cauchy sequence
in X, then {pn } converges to some point of X
(b) In Rk every Cauchy sequence converges.

Solution. Supporting Materials


(a) If E is the closure of a set E in a metric space X, then

diam E = diam E.

(b) If Kn is a sequence of compact sets in X such that Kn ⊃ Kn+1 (n = 1, 2, . . . . . . )


and if
lim diam Kn = 0,
n→∞
T∞
then 1 Kn consists of exactly one point.
If {pn } is a sequence in X and if EN consists of the points pN , pN +! , pN +2 , . . . . . . , it is
clear from the definition of Cauchy sequence and diameter of a set that {pn } is a Cauchy
sequence if and only if
lim diam EN = 0
n→∞
Proof
(a) Let {pn } be a Cauchy sequence in the compact space X. For N = 1, 2, 3, . . . . . . ,
let EN be the set consisting of pN , pN +1 , . . . . . . . Then

(8.1) lim diam E N = 0.


n→∞

Being a closed subset of the compact space X, each E N is compact. Also EN ⊃


EN +1 , so that E N ⊂ E N +1 .
Therefore there is a unique p ∈ X which lies in every E N .
Let  > 0 be given. By (8.1) there is an integer N0 such that diam E n <  if
N ≥ N0 . Since p ∈ E N , it follows that d(p, q) <  for every q ∈ E N , hence for
every q ∈ EN . In other words, d(p, pn ) <  if n ≥ N0 . Hence pn → p.
(b) Let {xn } be a Cauchy sequence in Rk . Define EN as above, with xi in place of pi .
For some N , diam EN < 1. The range of {xn } is the union of EN and the finite
set {x1 , . . . . . . xN −1 }. Hence {xn } is bounded. Since every bounded set of Rk has
compact closure in Rk the result follows from (a).

70
Problem 8.14. Let Q be a metric space with metric d(p, q) = |p − q|. Let E be the set
of all p ∈ Q such that 2 < p2 < 3. Show E is closed and bounded in Q but not compact.
Is E open in Q?

Solution. Let x ∈ Q \ E. We need to show that x is not a limit point of E. Since


√ √
x ∈ Q \ E either x2 ≤ 2 or x2 ≥ 3. Since ± 2 and ± 3 are not rational x2 6= 2 and
x2 6= 3 in Q. Considering the space R take,
√ √ √ √
r = min{d(x, 2), d(x, 3), d(x, − 2), d(x, − 3)} > 0.

Then Br (x) ⊂ Q \ E and therefore Br (x) ∩ E = ∅ and hence x is not a limit point of E.
This proves that E is closed.


Fix a point p ∈ E then d(x, p) ≤ 2 3 for all x ∈ E and hence E is bounded.


Construct a sequence {xn } in E so that xn → 2. Construction of such sequence is
possible since Q is dense in R. Since any subsequence of a convergent sequence conver-
√ √
gence to the same point, all the convergent subsequence converges in 2 but 2 ∈ / E
and hence E is not sequentially compact and hence not compact.


Another way to show that E is not compact could be as follows: 3 is a limit point

of E because any neighborhood of it contains a rational less than 3 since Q is dense in

R. However 3 ∈ / E since it is irrational. Therefore E as a subset of R is not closed.
Then by the Heine-Borel theorem E is not compact as a subset of R. Then E is not
compact relative to Q ⊂ R by Rudin theorem 2.33.

√ √ √ √
Finally we prove that E is open. For any q ∈ E, either q ∈ ( 2, 3) or (− 3, − 2).
√ √ √ √
Without loss of generality we assume q ∈ ( 2, 3). So, 2 < q < 3 therefore
√ √
r = min{ 3 − q , 2 − q } > 0. Then Br (q) ⊂ E and hence q is an interior point.
Since q was arbitrary, E is open.

71
Problem 8.15. Rudin Theorem 2.33
Suppose K ⊂ Y ⊂ X. Then K is compact reltive to X if and only if K is compact
relative to Y .

Solution. Suppose K is compact relative to X, and let {Vα } be a collection of sets, open
S
relative to Y , such that K ⊂ α Vα . There are sets Gα open relative to X, such that
Vα = Y ∩ Gα , for all α; and since K is compact relative to X, we have
n
[
(8.2) K⊂ Gαi
i=1

for some choice of finitely many indices α1 , . . . , αn . Since K ⊂ Y , from (8.2) we have
n
[
(8.3) K⊂ V αi .
i=1

This proves that K is compact relative to Y .

Conversely, suppose K is compact relative to Y , let {Gα } be a collection of open subsets


of X which covers K, and put Vα = Y ∩ Gα . Then (8.3) will hold for some choice of
α1 , . . . , αn ; and since Vα ⊂ Gα , (8.3) implies (8.2)

72
9. Continuity

Continuous Function: Let (X, dX ) and (Y, dY ) be a metric spaces. Any function
f : X → Y is continuous at x0 ∈ X if for every  > 0 there is a δ > 0 such that
dX (x, x0 ) < δ implies dY (f (x), f (x0 )) < . The function f is continuous on X if it is
continuous at every point in X. If f is not continuous at x, then we say f is discontinuous
at x.
Continuity in Topological Space: Let (X, T ) and (Y, S) be topological spaces. A
function f : X → Y is continuous at x ∈ X if for each neighborhood W of f (x) there
exists a neighborhood V of x such that f (V ) ⊂ W . We say that f is continuous on X if
it is continuous at every x ∈ X
Sequential continuity in Topology: Let (X, T ) and (Y, S) be topological spaces. A
function f : X → Y is sequentially continuous at x ∈ X if for every sequence (xn ) that
converges to x ∈ X, the sequence (f (xn )) converges to f (x) ∈ Y .
Continuity and Sequential Continuity are same:
Theorem Let (X, dX ) and (Y, dY ) be metric spaces. A function f : X → Y is continuous
at x ∈ X if and only if it is sequentially continuous at x
Another way to state this theorem is same as Theorem 4.2 of Baby Rudin
Definition: Let (X, dX ) and (Y, dY ) be metric spaces; suppose E ⊂ X, f maps E into
Y , and p is a limit point of E. We write f (x) → q as x → p, or

lim f (x) = q
x→p

if there is a point q ∈ Y with the following property: For each  > 0 there exists a δ > 0
such that dY (f (x), q) <  for all points x ∈ E for which 0 < dX (d, p) < δ.
Theorem: Let (X, dx ) and (Y, dY ) be metric spaces; suppose E ⊂ X, f maps E into Y ,
and p is a limit point of E. Then lim f (x) = q if and only if lim f (pn ) = q for every
x→p n→∞
sequence {pn } in E such that lim pn = p where pn 6= p.
n→∞
Proof: Suppose lim f (x) = q holds. Now choose any sequence {pn } so that pn 6= p
n→∞
that is {pn } is not a constant sequence and pn → p as n → ∞. We want to show that
lim f (pn ) = q.
n→∞
Let  > 0 be given. Since lim f (x) = q, there exists a δ > 0 such that dY (f (x), q) < 
x→p
if x ∈ E and 0 < dX (x, p) < δ. Also since pn → p, there exists an N ∈ N such that
dX (pn , p) < δ for all n > N . Thus for all n > N , we have dY (f (pn ), q) < , which implies
lim f (pn ) = q.
n→∞
Conversely suppose lim f (fn ) = q for every sequence {pn } in E such that lim pn = p
n→∞ n→∞
where pn 6= p but lim f (x) 6= q. Then there exists some  such that for every δ > 0 there
x→p
exists a point x ∈ E (depending on δ), for which dY (f (x), q) ≥  but 0 < dX (x, p) < δ.
Let δn = n1 . Take a point p1 ∈ E such that 0 < dX (p1, p) < δ1 but dY (f (p1 ), q) > 1 and
similarly choose p2 ∈ E such that 0 < d(p2 , p) < δ but dY (f (p1 ), q) > 2 . We construct a
sequence in this way. Note that we guarantee existence of such points pj because p is a
73
limit point of E. Now consider the sequence {pn } and note that pn → p by construction
but d( f (pn ), q) > n for all n which is a contradiction to the fact that lim f (pn ) = q for
n→∞
all {pn }.
Theorem: Let (X, T ) and (Y, S) be two topological spaces and f : X → Y . Then f is
continuous on X if and only if f −1 (G) ∈ T for every G ∈ S.
Corollary: A function f is continuous if and only if the inverse image of closed sets is
closed.
Uniform Continuity: Let (X, dX ) and (Y, dY ) be metric spaces. A function f : X → Y
is uniformly continuous on X if for every  > 0 there exists δ > 0 such that, dX (x, y) < δ
implies dY (f (x), f (y)) <  for all x, y ∈ X.

9.1. Completing a Metric Space with an Isometry

Definition: Let (X, dX ) and (Y, dY ) be metric spaces. A map ` : X → Y which satisfies

dY (`(x1 ), `(x2 )) = dX (x1 , x2 ), ∀x1 , x2 ∈ X

is called an isometry or isometric embedding of X into Y . An isometry which is onto is


called a metric space isomorphism. We say that X and Y are isometric if there exists an
isomorphism ` : X → Y .
Definition: A metric space (X, d) is the completion of (X, d) if the following conditions
are satisfied:

(a) there exists an isometric embedding ` : X → X;


(b) the image space `(X) is dense in X;
(1) the space (X, d) is complete.

Theorem: Every metric space has a completion and the completion is unique up to
isomorphism

9.2. Continuous Function on Compact Sets

Theorem: Let f : K → Y be continuous on K, where K is a compact metric space and


Y is any metric space. Then f (K) is compact.

Theorem: Let f : K → Y be a continuous function on a compact set K. Then f is


uniformly continuous.
Theorem: Let K be a compact metric space and f : K → R a continuous, real-valued
function. Then f is bounded on K and attains its maximum and minimum. That is,
there are points x, y ∈ K such that

f (x) = inf ∈ Kf (z), f (y) = sup f (z)


z z∈K
74
Theorem: A function f : X → R on a metric space (X, d) is Lipschitz continuous on X
if there is a constant C ≥ 0 such that

|f (x) − f (y)| ≤ Cd(x, y), ∀x, y ∈ X

75
9.3. Proofs of the Theorems

Theorem: Let f : K → Y be continuous on K, where K is a compact metric space and


Y is any metric space. Then f (K) is compact.
Proof: Consider any open cover {Gα }α∈Λ ∈ Y of f (K). Since inverse image of a open
set is open under continuous map f −1 (Gα ) is open in K for all α ∈ Λ. For any x ∈ K,
f (x) ∈ Gα for some α ∈ Λ and so x ∈ f −1 (Gα ). Therefore, {f −1 (Gα )}α∈Λ is an open
cover of K. Since K is compact, there exists a finite subcover, which we denote by
{f −1 (Gi )}ni=1 . We want to show that {Gi }ni=1 is a finite subcover of f (K). Take y ∈ f (K)
be any arbitrary point and so there is x ∈ K such that f (x) = y and let x ∈ f −1 (Gi )
for any 1 ≤ i ≤ n and therefore y = f (x) ∈ Gi . Since y was taken to be arbitrary we
conclude that f (K) ⊆ ni=1 Gi and hence Gα has a finite subcover. Therefore f (K) is
S

compact.
Theorem: Let f : K → Y be a continuous function on a compact set K. Then f is
uniformly continuous.
Proof: Suppose f is not uniformly continuous. Then there is an  > 0 such that for all
δ > 0, there are x, y ∈ K with dK (x, y) < δ and dY (f (x), f (y)) ≥ . Taking δ = n1 for
n ∈ N, we find that there are sequence (xn ) and (yn ) in K such that
1
dK (xn , yn ) < , dY (f (xn ), f (yn )) ≥ 
n
Since K is compact there are convergent subsequences of (xn ) and (yn ) which we denote
by (xni ) and (ynj ). Since dK (xn , yn ) < n1 the subsequences converges to the same limit
(taking n → ∞, d(x, y) → 0). But the sequence (f (xn )) and (f (yn )) either diverge or
converge to different limits. This contradicts the continuity of f .

76
9.4. Problems:

Problem 9.1. Let X and Y be metric spaces and A ⊂ X. Suppose f maps A into
Y and p is a limit point of A. We say that f has a limit at p and write f (x) → q as
x → p, or limx→p f (x) = q if there is a point q ∈ Y with the property that for every
 > 0 there exists a δ > 0 such that for all points x ∈ E with 0 < dX (x, p) < δ implies
dY (f (x), q) < . Prove the following theorems.
(a) limx→p f (x) = q if and only if limn→∞ f (pn ) = q for every sequence (pn ) ⊂ E such
that pn 6= p for all n and pn → p.
(b) If f has a limit at p, then the limit is unique.
(c) The function f is continuous at p if and only if limx→p f (x) = f (p).

Solution. Solution:(a) First assume that lim f (x) = q holds. Now choose any sequence
x→p
{pn } so that pn 6= p that is {pn } is not a constant sequence and pn → p as n → ∞. We
want to show that lim f (pn ) = q.
n→∞

Let  > 0 be given. Since lim f (x) = q, there exists a δ > 0 such that dY (f (x), q) < 
x→p
if x ∈ E and 0 < dX (x, p) < δ. Also since pn → p, there exists an N ∈ N such that
dX (pn , p) < δ for all n > N . Thus for all n > N , we have dY (f (pn ), q) < , which implies
lim f (pn ) = q.
n→∞
Conversely suppose lim f (fn ) = q for every sequence {pn } in E such that lim pn = p
n→∞ n→∞
where pn 6= p but lim f (x) 6= q. Then there exists some  such that for every δ > 0 there
x→p
exists a point x ∈ E (depending on δ), for which dY (f (x), q) ≥  but 0 < dX (x, p) < δ.
Let δn = n1 . Take a point p1 ∈ E such that 0 < dX (p1, p) < δ1 but dY (f (p1 ), q) > 1 and
similarly choose p2 ∈ E such that 0 < d(p2 , p) < δ but dY (f (p1 ), q) > 2 . We construct a
sequence in this way. Note that we guarantee existence of such points pj because p is a
limit point of E. Now consider the sequence {pn } and note that pn → p by construction
but d( f (pn ), q) > n for all n which is a contradiction to the fact that lim f (pn ) = q for
n→∞
all {pn }.

Solution (b): Since f has a limit at p is same as saying every sequence {pn } in X that
converges to p the sequence {f (pn )} converges to some point q ∈ Y . From elementary
analysis we know that limit of a convergent sequence is unique and hence limit of {f (pn )}
is unique and so limit of f at p is unique.
Solution (c): lim f (x) = f (p) holds if and only if for any  > 0 there exists a δ > 0 such
x→p
that dX (x, p) < δ implies dY (f (x), f (p)) <  and this implies continuity of f at p ∈ E.

77
Problem 9.2. Let f : R2 → R be defined
 2
 x41 x22 , (x , x ) 6= (0, 0),
1 2
f (x1 , x2 ) = x1 +x2
0, (x1 , x2 ) = (0, 0).

Show that limx→0 f (x, mx) = 0 for every m ∈ R but f is discontinuous at (0, 0). Interpret
this result.
mx 3 mx
Solution. If x 6= 0 then f (x, mx) = x4 +m 2 x2 = x2 +m2 . If m = 0 then f(x,mx)=0. If

m 6= 0 then x2 + m2 ≥ m2 ∀ x, but mx → 0 as x → 0, which shows

lim f (x, mx) = 0, ∀m ∈ R


x→0

However f (x, mx2 ) tells a different story. Assume m 6= 0 and x 6= 0, then (x, mx2 ) →
(0, 0) as x → 0 but
m
f (x, mx2 ) =
1 + m2
and therefore
lim f (x, mx2 ) 6= 0.
x→0
Hence f is not continuous at (0, 0)

78
Problem 9.3. Let C 1 ([a, b]) be the metric space of continuously differentiable functions
on [a, b] with metric

d(f, g) = sup |f (x) − g(x)| + sup |f 0 (x) − g 0 (x)| .


a≤x≤b a≤x≤b

Let D : C 1 ([a, b]) → C([a, b]) be defined by D(f ) = f 0 . Prove that D is continuous.

Solution. Let  > 0 be given. Fix a function f ∈ C 1 ([a, b]).


d0 (D(f ), D(g)) =d0 (f 0 , g 0 )
= sup |f 0 (x) − g 0 (x)|
a≤x≤b

≤ sup |f (x) − g(x)| + sup |f 0 (x) − g 0 (x)| = d(f, g)


a≤x≤b a≤x≤b

Therefore taking δ =  we can conclude that, for given  > 0, there exists a δ > 0 such
that d(f, g) < δ implies d0 (D(f ), D(g)) < . Since f was arbitrary it infers that D is
continuous.

79
Problem 9.4. Let C([a, b]) be the metric space of continuous functions on [a, b] with
metric Z b
d(f, g) = |f (t) − g(t)| dt.
a
Define T : C([a, b]) → R by
Z b
T (f ) = f (t) dt.
a
(a) Prove that T is continuous using sequential definition of continuity.
(b) Prove that T is continuous using δ −  definition.

Solution. Solution (a): Let {fn } be any sequence in C([a, b]) so that fn → f and
fn 6= f for all n. Since fn → f , for given  > 0 there is N ∈ N such that d(fn , f ) <  for
all n > N . Observe that, for all n > N ,
Z b Z b

|T (fn ) − T (f )| =
fn (t) − f (t) ≤ |fn (t) − f (t)| = d(fn , f ) < 
a a

Which implies that the sequence T (fn ) converges to T (f ) and by definition, T is sequen-
tially continuous and hence continuous by Theorem 1.
Solution (b): Let f be any continuous function in C([a, b]).Let  > 0 be given. Since
|T (f ), T (g)| ≤ d(f, g) taking δ =  we can conclude that d(f, g) < δ implies |T (f ), T (g)| <
. So, T is continuous at f ∈ C([a, b]). Since f is arbitrary we conclude that T is contin-
uous in C([a, b])

80
Problem 9.5. Let f and g be continuous maps of a metric space X into a metric space
Y and let E be a dense subset of X.
(a) Prove that f (E) is dense in f (X).
(b) If g(x) = f (x) for all x in a dense subset E of X, prove that g(x) = f (x) for all
x ∈ X.
Solution. Solution (a): Take any y ∈ f (X). For any  > 0 the ball B (y) is open in
Y and f −1 (B (y)) is open in X since f is continuous in X. Since E is dense in X there
exists x ∈ f −1 (B (y)) ∩ E. So f (x) ∈ B (y). So f (E) ∩ B (y) 6= ∅ and contains a point
of f (E), for any y ∈ f (X) and hence f (E) is dense in f (X).

Alternative: Lemma: For all A ⊂ X, f (A) ⊂ f (A)


Proof of Lemma: We only need to show that f (A0 ) ⊂ f (A). Consider x ∈ A0 and we want
to show that f (x) ∈ f (A). Consider any neighborhood W of f (x). Since f is continuous
there exists a neighborhood of V of x in X such that V ⊂ f −1 (W ). Since x is a limit
point of A there is a point y ∈ A ∩ V such that y 6= x which implies f (y) ∈ W ∩ f (A).
Since W was arbitrary we can conclude that every neighborhood of f (x) contains a point
of f (A) and hence f (x) ∈ f (A). Its prove the lemma.
Since E is dense in X, we have E = X and hence f (X) = f (E) ⊂ f (E) showing that
f (E) is dense in f (X).

Solution(b): Consider the function h : X → R defined by h(x) = dY (f (x), g(x)). First


we show that h is continuous. Let x0 ∈ X be any point and  > 0 be given,
|h(x) − h(x0 )| = |dY (f (x), g(x)) − dY (f (x0 ), g(x0 ))|
= |dY (f (x), g(x)) + dY (f (x0 ), g(x)) − dY (f (x0 ), g(x)) − dY (f (x0 ), g(x0 ))|
≤ |dY (f (x), g(x)) + dY (f (x0 ), g(x))| + |dY (f (x0 ), g(x)) + dY (f (x0 ), g(x0 ))|
≤ |dY (f (x), f (x0 ))| + |dY (g(x), g(x0 ))|
Since f : X → Y is continuous, for given  there exists a δ1 > 0 such that dX (x, x0 ) < δ1
implies dY (f (x), f (x0 )) < 2 . Again since g : X → Y is continuous there exists a δ2 > 0
such that dX (x, x0 ) < δ2 implies dY (g(x), g(x0 )) < 2 . Therefore, taking δ = min(δ1 , δ2 )
we get, d( x, x0 ) < δ implies
 
|h(x) − h(x0 )| < + = .
2 2
Therefore h is continuous.

Since f (x) = g(x) in E we have h(x) = 0 in E. Take any point y ∈ X \ E. For any
δ > 0, Bδ (y) contains a point z ∈ E since E is dense in X. Assume that h(y) 6= 0 that
is h(y) ≥ y . So, ∀δ we have |h(y) − h(z)| > y which contradicts the fact that h is
continuous. So h(y) = 0 and since y was arbitrary we can conclude that h(x) = 0 for all
x ∈ X and therefore f (x) = g(x) for all x ∈ X.
81
Problem 9.6. Let A be a set and x a point in a metric space (X, d). Define the “distance
from x to A” to be
d(x, A) = inf d(x, y).
y∈A

(a) Show that for fixed A, the function f : X → R (with the usual metric on R) given by
f (x) = d(x, A) is continuous. (b) Show {x : d(x, A) = 0} = Ā.

82
Problem 9.7. Suppose f is a mapping from metric space X to metric space Y . Prove
that a function is continuous if and only if f −1 (A) is closed in X for every closed set
A⊂Y.

Solution. First assume that f is continuous and we want to show that if A is closed
then f −1 (A) is closed. Since A is closed Ac is open and since f is continuous, f −1 (Ac ) is
open. But
c
f −1 (Ac ) = {x ∈ X|f (x) ∈ Ac } = {x ∈ X|f (x) ∈ / A} = {x ∈ X|x ∈ / f −1 (A)} = f −1 (A)
c
shows that (f −1 (A)) is open and hence f −1 (A) is closed.

Now we take that f −1 (A) is closed whenever A is closed in Y and want to show that
f is continuous. Let B be any open set in Y . So B c is closed. By the hypothesis f −1 (B c )
c c
is closed. But we have shown that f −1 (B c ) = (f −1 (B)) and hence (f −1 (B)) is closed
which implies f −1 (B) is open. Since B was arbitrary we conclude that for every open set
B the preimage f −1 (B) is open and hence f is continuous.

83
Problem 9.8. Assume f : R → R is continuous and define A = {x : f (x) = 0}. Show
A is closed.

Solution.
A = {x : f (x) = 0} = f −1 ({0})
But the set {0} being a singleton set is closed and by the above problem A = f −1 ({0})
is closed.

84
Problem 9.9. Suppose f is a continuous bijection of a compact metric space X onto a
metric space Y . Prove that the inverse map f −1 defined on Y by f −1 (f (x)) = x (where
x ∈ X) is a continuous mapping of Y onto X.

Solution. Let A ⊂ X be any closed subset. Since X is compact being a closed subset
A is compact. Since f is continuous it maps a compact set to a compact set and hence
f (A) is compact and therefore is closed. But
−1
f −1 (A) = {y ∈ Y : f −1 (y) ∈ A} = {y ∈ Y : y ∈ f (A)} = f (A)

Since A is arbitrary we conclude that the inverse image of a closed set A under the
function f −1 is closed and hence f −1 is continuous.

85
Problem 9.10. Let (X, d) be a metric space and F, G ⊂ X with F closed, G open,
and F ⊂ G. Show that there is a continuous function f : X → R such that 0 ≤
f (x) ≤ 1, f (x) = 1 for x ∈ F , and f (x) = 0 for x ∈ Gc . Hint: Consider the function
f (x) = d(x, Gc )/[d(x, Gc ) + d(x, F )]. This result is called Urysohn’s lemma and is useful
in showing the existence of partitions of unity.

Solution. First we would see the construction. f (x) = 1 for x ∈ F and f (x) = 0 for
x ∈ Gc . So we can see this as a parallel line to x axis over F and the x axis itself over Gc .
To make it continuous we have to draw a line form (Gc , 0) (thinking Gc to be the corner
c c)
point and) to (F, 1). The equation of the line would look like y = Fx−G−Gc
= d(x,Gd(x,G
c )+d(x,F ) .

Take
d(x, Gc )
f (x) =
d(x, Gc ) + d(x, F )
For all x ∈ F , d(x, F ) = 0 and we claim that d(x, Gc ) > 0. Since x ∈ F ⊂ G and G
is open, there is  such that B (x) ⊂ G and so d(x, y) ≥  for all y ∈ Gc and hence
d(x, Gx ) ≥  > 0. Therefore we have
d(x, Gc ) d(x, Gc )
f (x) = = = 1, ∀x ∈ F
d(x, Gc ) + d(x, F ) d(x, Gc )
Now take x ∈ Gc . For any y ∈ F since y ∈ G and G is open using same argument above
we conclude that d(x, y) ≥  and hence d(x, F ) ≥  > 0. Therefore,
0
f (x) = =0
d(x, F )
Now we want to show that f is continuous. (The idea is to use the fact that distance
metric is continuous and composition is also continuous but have to fill up with details
to show the denominator is not equal to zero)

86
Problem 9.11. Let f and g be continuous maps of a metric space X into a metric space
Y and let E be a dense subset of X. (a) Prove that f (E) is dense in f (X). (b) If
g(x) = f (x) for all x in a dense subset E of X, prove that g(x) = f (x) for all x ∈ X.
Explain why these results are important for our everyday experience with functions and
numbers.

87
Problem 9.12. (a) Show that the requirement in the definition of uniform continuity
can be rephrased as follows, in terms of diameters of sets: For every  > 0 there exists a
δ > 0 such that diam(f (E)) <  for all E ⊂ X with diam(E) < δ. (b) Let E be a dense
subset of a metric space X, and let f be a uniformly continuous function into a complete
metric space Y defined on E. Prove that f has a continuous extension from E to X in
two ways using part (a) and then using Cauchy sequences.

88
Problem 9.13. Let f : R → R be a continuous function that is periodic with period
T > 0. (a) Show that f has a finite maximum and minimum values on R. (b) Show that
f is uniformly continuous on R.

89
Problem 9.14. Let X be a metric space.
(a) Call two Cauchy sequences (pn ), (qn ) in X equivalent if

lim d(pn , qn ) = 0.
n→∞

Prove that this is an equivalence relation.


(b) Let X ∗ be the set of all equivalence class so obtained. If P ∈ X̃ and Q ∈ X̃, (pn ) ∈ P
and (qn ) ∈ Q, define
∆(P, Q) = lim d(pn , qn ).
n→∞
Note that by an exercise from lecture 1 that this limit exists. Show that the number
∆(P, Q) is unchanged if (pn ) and (qn ) are replaced by equivalent sequences, and hence
that ∆ is a distance function in X̃.
(c) Prove that the resulting metric space X̃ is complete.
(d) For each p ∈ X, there is a Cauchy sequence all of whose terms are p; let Pp be the
element of X̃ which contains this sequence. Prove that ∆(Pp , Pq ) = d(p, q) for all
p, q ∈ X. In other words, the mapping φ defined by φ(p) = Pp is an isometry of X
into X̃.
(e) Prove that φ(X) is dense in X, and that φ(X) = X̃ if X is complete. By part (d),
we may identify X and φ(X) and thus regard X as embedded in the complete space
X̃. We call X̃ the completion of X.

90
Problem 9.15. Let (X, d) be a metric space and suppose f is uniformly continuous on
the compact subsets E1 , . . . , En of X. (a) Prove that f is uniformly continuous on ∪ni=1 Ei .
(b) Give an example of a function f that is continuous on R and a sequence of compact
intervals E1 , E2 , E3 , . . . on each of which f is uniformly continuous, yet f is not uniformly
continuous on ∪∞ i=1 Ei .

91
Problem 9.16. Prove that if f is a continuous mapping of a metric space X into a
metric space Y and E ⊂ X is connected, then f (E) is connected.

92
10. Differentiation of Single Variable

Definition 10.1. Let f be defined on [a, b]. For any x ∈ [a, b] form the quotient
f (t) − f (x)
φ(t) = (a < t < b, t 6= x)
t−x
and define
f 0 (x) = lim φ(t),
t→x
provided this limit exists.
We thus associate with this function f a function f 0 whose domain is the set of points x
at which the limit exists; f 0 is called the derivative of f .
If f 0 is defined as a point x, we say that f is differentiable at x. If f 0 is defined at every
point of a set E ⊂ [a, b], we say that f is differentiable on E.

Theorem 10.1. Let f is defined on [a, b]. If f is differentiable at a point x ∈ [a, b], then
f is continuous at x.

Theorem 10.2. Suppose f and g are defined on [a, b] and are differentiable at a point
x ∈ [a, b]. Then f + g, f g and f /g are differentiable at x, and
(a) (f + g)0 (x) = f 0 (x) + g 0 (x);
0 0
(b) 
(f g)
0 (x) = f (x)g(x) + f (x)g 0 (x);
0 0 (x)f (x)
(c) fg (x) = g(x)f (x)−gg 2 (x)

In (c) we assume of course that g(x) 6= 0.

Theorem 10.3 (Chain Rule). Suppose f is continuous on [a, b], f 0 (x) exists at some
point x ∈ [a, b], g is defined on an interval I which contains the range of f , and g is
differentiable at the point f (x). If

h(t) = g (f (t)) (a ≤ t ≤ b),

then h is differentiable at x, and

h0 (x) = g 0 (f (x))f 0 (x)

Definition 10.2. Let f be a real function defined on a metric space X. We say that f
has a local maximum at a point p ∈ X if there exists δ > 0 such that f (q) ≤ f (p) for all
q ∈ X with d(p, q) < δ

Theorem 10.4. Let f be defined on [a, b]; if f has a local maximum at a point x ∈ (a, b),
and if f 0 (x) exists, then f 0 (x) = 0.

Theorem 10.5 (Generalized Mean Value Theorem). If f and g are continuous real
functions on [a, b] which are differentiable in (a, b), then there is a point x ∈ (a, b) at
which
[f (b) − f (a)]g 0 (x) = [g(b) − g(a)]f 0 (x).
93
Theorem 10.6. If f is a real continuous function on [a, b] which is differentiable in
(a, b), then there is a point x ∈ (a, b) at which

f (b) − f (a) = (b − a)f 0 (x)

Theorem 10.7. Suppose f is differentiable in (a, b).


(a) If f 0 (x)] ≥ 0 for all x ∈ (a, b) , then f is monotonically increasing.
(b) If f 0 (x) = 0 for all x ∈ (a, b), then f is constant.
(c) If f 0 (x)] ≤ 0 for all x ∈ (a, b) , then f is monotonically decreasing.

Theorem 10.8 (Similar to IVT). Suppose f is a real differentiable function on [a, b] and
suppose f 0 (a) < λ < f 0 (b). Then there is a point x ∈ (a, b) such that f 0 (x) = λ

Theorem 10.9 (L’Hospital’s Rule). Suppose f and g are real and differentiable in (a, b),
and g 0 (x) 6= 0 for all x ∈ (a, b), where −∞ ≤ a ≤ b ≤ +∞. Suppose
f 0 (x)
→ A, as x→a
g 0 (x)
If f (x) → 0 and g(x) → 0 as x → a or if g(x) → +∞ as x → a, then
f (x)
→A as x → a.
g(x)
The consequence of the mean value theorem which is also valid for vector valued func-
tion is as follows:

Theorem 10.10. Suppose f is a continuous mapping of [a, b] into Rk and f is differen-


tiable in (a, b). Then there exists x ∈ (a, b) such that

|f (b) − f (a)| ≤ (b − a) |f 0 (x)| .

10.1. Some Problems

Problem 10.1. content...

94
11. Sequences and Spaces of Continuous Functions

Definition 11.1. A sequence of bounded, real-valued functions (fn ) on a metric space


(X, d) converges uniformly to a function f if

lim kfn − f k∞ = 0.
n→∞

Another representation can be

lim sup |fn (x) − f (x)| = 0.


n→∞ x∈X

Theorem 11.1. Let (fn ) be a sequence of bounded, continuous, real-valued functions on


a metric space (X, d). If fn → f uniformly, then f is continuous.

11.1. Space of Continuous Functions

Definition 11.2. Let (X, d) be a metric space. We define the set of continuous, real-
valued functions f : X → R by C(X). We let Cb (X) denote the space of bounded contin-
uous functions on X.

Theorem 11.2. Let K be a compact subset of a metric space (X, d). The space C(K) is
complete.

Definition 11.3. The support, supp(f ) of a function f : X → R on a metric space (X, d)


is the closure of the set on which f is nonzero,

supp(f ) = {x ∈ X : f (x) 6= 0}.

We say that f has compact support if supp(f ) is a compact subset of X, and denote
the space of continuous functions on X with compact support by Cc (X). We denote the
closure of Cc (X) by C0 (X).

Theorem 11.3 (Weierstrass Approximation Theorem). Let a < b be real numbers. The
set of polynomials on [a, b] is dense in C([a, b]).

11.2. Compact Subset of Continuous Function Space

Definition 11.4 (Equicontinuity). Let F be a family of functions from a metric space


(X, dX ) to a metric space (Y, dY ). The family F is equicontinuous if for every x ∈ X and
 > 0 there is a δ > 0 such that d(x, y) < δ implies d(f (x), f (y)) <  for all f ∈ F. If δ
can be chosen independently of x, then F is called uniformly equicontinuous.

Theorem 11.4. An equicontinuous family of functions from a compact metric space to


a metric space is uniformly equicontinuous.
95
Theorem 11.5 (Arzela-Ascoli). Let K be a compact subset of metric space (X, d). A
subset of C(K) is compact if and only if it is closed, bounded, and equicontinuous.

Theorem 11.6. If X is a compact metric space and (fn ) is a sequence of continuous


functions from X to metric space Y that converges uniformly to f , then {fn } is equicon-
tinuous on X.

96
11.3. Proof of the Theorems:

Theorem: Let (fn ) be a sequence of bounded, continuous, real-valued functions on a


metric space (X, d). If fn → f uniformly, then f is continuous.

Proof: Let  > 0 be given. Since fn → f uniformly, there exists N ∈ N such


that |fn (x) − f (x)| < 3 for all n ≥ N and for all x ∈ X. Let x0 be an arbitrary
point in X. Since fN is continuous, there exists δ > 0 so that d(x, x0 ) < δ implies
|fN (x) − fN (x0 )| < 3 . Therefore, d(x, x0 ) ≥ δ implies,
  
|f (x) − f (x0 )| ≥ |f (x) − fN (x)| + |fN (x) − fN (x0 )| + |fN (x0 ) − f (x)| < + + = 
3 3 3
Therefore f is continuous in X.

Theorem: An equicontinuous family of functions from a compact metric space to a


metric space is uniformly equicontinuous.

Proof: Suppose that K is a compact metric space, and F is a family of functions


f : K → Y that is not uniformly equicontinuous. We will prove that F is not equicon-
tinuous.

Since F is not uniformly equicontinuous, there is an  > 0, such that for every n ∈ N
there are points xn , yn ∈ K and a function fn ∈ F with
1
(11.1) d(xn , yn ) < , and d(fn (yn ), fn (xn )) ≥ 2.
n
Since K is compact, the sequence (xn ) has a convergent subsequence, which are also
denoted by (xn ). Suppose that xn → x as n → ∞. Then d(xn , yn ) < n1 implies yn → x as
well. Hence, for all δ > 0, there are points xn , yn such that d(xn , x) < δ and d(yn , x) < δ.
But from (11.1), we must have either d(fn (xn ), fn (x)) ≥  or d(fn (yn ), fn (x)) ≥ , so F
is not equicontinuous at x.

97
11.4. Problems on Sequence of Functions
x
Problem 11.1. Prove the sequence ( n+x2 ) converges uniformly on R with the usual

metric.
x x
Solution. Fix x ∈ R then lim 2 = 0 and so fn = n+x2
converges pointwise to 0.
n→∞ n+x

To see uniform convergence we let



x
Mn = sup

2
x∈R n + x
x 0 n−x2 √
Let g(x) = n+x 2 then g (x) = (n+x2 )2
which gives the critical value x = n which gives
the maximum value 2√1 n . So,

x 1
n + x2 ≤ 2√n

For given  we can find N ∈ N such that 2√1N <  by Archimedean property to conclude
x
that n ≥ N implies n+x 2
<  for all x ∈ R. Hence the given sequence is uniformly
convergence.

98
Problem 11.2. (a) Define what it means for a sequence of functions to be uniformly
Cauchy and prove that a uniformly convergent sequence of functions is uniformly Cauchy.
(b) Prove that every uniformly convergent sequence of bounded functions is uniformly
bounded. Here, we say that a sequence of real-valued functions (fn ) is uniformly bounded
if there exists M > 0 such that kfn k∞ ≤ M . (c) Show there exists a sequence of functions
that converges pointwise to the function that is identically 0, but each function in the
sequence is unbounded Hint: Consider the sequence {x/n} on [0, ∞).

Solution. (a) A sequence of function (fn (x)) in a subset A of a metric space (X, d)
is called uniformly Cauchy if for any  > 0 there exists N ∈ N such that n, m ≥ N
implies d(fn (x), fm (x)) <  for all x ∈ A.

The another way of defining the uniform Cauchy criteria is using the natural
norm i.e. the infinity norm. A sequence of real valued function (fn ) on a metric
space (X, d) is said to be uniformly Cauchy if for any  > 0 there exists an N ∈ N
such that for all n ≥ N
kfn − f k∞ < .
Let the sequence (fn ) of functions converges to f uniformly. Let  > 0 be given.
There exists an N ∈ N such that d(fn (x), f (x)) < 2 for all x ∈ A whenever
n ≥ N . Now take n, m ≥ N , then
 
d(fn (x), fm (x)) ≤ d(fn (x), f (x)) + d(fm (x), f (x)) < + = 
2 2
for all x ∈ A. Therefore (fn ) is uniformly Cauchy.
(b) Since (fn ) is a sequence of bounded functions for each n there is Mn such that
kfn k∞ ≤ M . Since the sequence is uniformly convergence it is uniformly Cauchy
and so for  = 1 there is N ∈ N such that d(fm (x), fN (x)) < 1 for all m ≥ N .
Take M = 1 + max{M1 , . . . . . . MN }.

99
Problem 11.3.
(a) Prove that if the sequences of real-valued functions (fn ) and (gn ) converge uniformly
on A ⊂ X, then (fn + gn ) converges uniformly on A. (b) If fn and gn are also bounded for
each n ∈ N, prove that (fn gn ) converges uniformly on A. (c) Show that the boundedness
is necessary in (b) by constructing an example where (fn gn ) does not converge uniformly
(although it must converge pointwise).

Solution. Let  > 0 be given. Since (fn ) converges uniformly to f (say), there exists an
N1 ∈ N such that |fn (x) − f (x)| < 2 for all n ≥ N1 and for all x ∈ A.

Also, since (gn ) converges uniformly to g(say), there exists an N2 ∈ N such that |gn (x) − g(x)| <

2
for all n ≥ N2 and for all x ∈ A. Take N = max{N1 , N2 }, then n ≥ N implies
 
|(fn + gn ) (x) − (f + g)(x)| ≤ |fn (x) − f (x)| + |gn (x) − g(x)| < + = 
2 2
for all x ∈ A. Hence (fn + gn ) converges uniformly.

Since the function gn is bounded, there is Mn ∈ R such that |gn | ≤ Mn for all x ∈ A.
Then, |gN2 (x) − g(x)| <  implies |g(x)| ≤ |gN2 (x)| +  ≤ MN2 +  = Q(say). Since (fn ) is
a uniformly convergent sequence of bounded function, the sequence is uniformly bounded
(Rudin Problem 1) by P(say). Now, since fn → f uniformly, there exists N1 ∈ N such

that n ≥ N1 implies |fn (x) − f (x)| < 2Q for all x ∈ A. Similarly, for gn → g uniformly,
there exists N2 ∈ N such that |gn (x) − g(x)| < 2P for all x ∈ A.
|(fn gn )(x) − (f g)(x)| = |fn (x)gn (x) − fn (x)g(x) + fn (x)g(x) − f (x)g(x)|
≤ |fn (x)| |gn (x) − g(x)| + |g(x)| |fn (x) − f (x)|
 
<P +Q = ,
2P 2Q
for all x ∈ A. Therefore, fn gn converges uniformly.
Let fn (x) = x1 and gn (x) = n1 for each n on the interval (0, 1). Then the sequences (fn )
converges to f (x) = 1/x uniformly because it is a constant sequence. Also (gn ) converges
uniformly to g(x) = 0 uniformly because the functions are independent of x. We claim
1
the the product hn = (fn gn )(x) = xn converges pointwise but not uniformly.
(hn ) converge point wise because for a fixed x and for given  > 0 by Archimedean
1
Property we can pick N ≥ x so that for all n ≥ N we have

1 1
|hn (x) − 0| = < <
xn Nx
So the sequence converges to 0 pointwise. However for any given N we can choose
0 < x < N1 so that
1 1
N x − 0 = N x > 1,

so the convergence can not be uniform.


100
Problem 11.4. Let (X, dX ) and (Y, dY ) be metric spaces, A ⊂ X, and (fn ) a sequence
of continuous functions from A to Y that converges to a function f : A → Y uniformly on
A. Given any x ∈ A, prove that limn→∞ fn (xn ) = f (x) for every sequence (xn ) in A with
xn → x. Construct an example showing the converse is not true, but if A is compact,
then the converse is true.

101
Problem 11.5. (General Arzelà Ascoli Theorem) If X is a compact metric space, (fn ) ⊂
C(K), (fn ) is pointwise bounded and equicontinuous on X, prove that
(a) (fn ) is uniformly bounded on X, and
(b) f (n) contains a uniformly convergent subsequence.
Hint: For (a) use compactness to get the uniform bound. For (b), use the fact that
a compact set is separable to extract a countable dense subset. Then prove that on the
countable set the sequence has a convergent subsequence. Prove this subsequence converges
uniformly on X.

Solution. (a) Since (fn ) is an equicontinuous family of function on a compact metric


space it is uniformly equicontinuous. That is for given  > 0 there is a δ > 0 such
that d(x, y) < δ implies dY (f (x), f (y)) <  for all x ∈ X. Since X is compact
there is a finite δ − net, say xi : 1 ≤ i ≤ m. Since (fn ) is pointwise bounded
f n(xi ) < Mi for 1 ≤ i ≤ m. Now take M = max(M1 , M2 , . . . . . . , Mm ) + . We
claim that M is a bound of fn (x) independent of x. Take any x ∈ X, there is xi
for some i so that d(x, xi ) < δ which implies dY (fn (x), fn (xi )) <  and therefore
|fn (x)| ≤ |fn (xi )| +  < M
(b) Since X is compact it is sequentially compact and hence is separable. Also X being
separable has a countable dense subset, say, E. Let the elements of the countable
set E are {x1 , x2 , . . . . . . }. The numerical sequence {fn (x1 )}∞
n=1 is bounded, so by
Bolzano-Weierstrass it has a convergent subsequence, say,

102
Problem 11.6. Let (X, d) be a metric space and suppose K ⊂ X is compact, fn → f on
K (and all functions are real-valued and continuous), and fn (x) ≥ fn+1 (x) for all x ∈ K
and n ∈ N. Prove that fn → f uniformly on K. Construct an example showing the
necessity of compactness.

Solution. Set gn = fn − f . Then gn is continuous, gn → 0 pointwise, and gn ≥ gn+1 . We


have to prove that gn → 0 uniformly on K.

Let  > 0 be given. Let Kn be the set of all x ∈ K with gn (x) ≥ . The set
Yn = {gn (x) ∈ gn (K) : g(x < )} is open. Take any point g(x0 ) ∈ Yn then there is
a neighborhood Bδ (g(x0 )) with δ <  − g(x0 ) > 0 is contained entirely in Yn and hence
the set {gn (x) ∈ gn (K) : g(x ≥ )} is closed. By continuity the inverse image Kn is also
closed. Since Kn is a closed subset of a compact set K, Kn is itself compact.

Since gn ≥ gn+1 , we have Kn+1 ⊂ Kn . Fix x ∈ K. Since gn (x) → 0, we see that


T
x∈/ Kn if n is sufficiently large. Thus x ∈ Kn . Since x was arbitrary it implies that
T
Kn = ∅ Hence KN is empty for some N . It follows that 0 ≤ gn (x) <  for all x ∈ K
and for all n ≥ N . This proves the theorem.

Compactness is necessary. For example if


1
fn (x) = (0 < x < 1; n = 1, 2, 3, . . . . . . )
nx + 1
then fn (x) → 0 monotonically in (0, 1), but the convergence is not uniform. To see that
take x = n1 and so fn (x) = 12 .

103
 n

Problem 11.7. Is the set of functions x2 + 1+n
x n=1 taking R to R (with the usual
metric) equicontinuous on [0, 1]?

Solution. .
n

Process 1 The set [0, 1] is compact. The sequence of function (fn ) = x2 + n+1 x is point-
wise convergent to f = x2 + x as n → 0. Also for a fixed x, fn is monotonically
decreasing. Hence according to above problem the sequence of function is uni-
formly convergent and hence equicontinuous.
Process 2
2 n 2 n ≤ x − y 2 + n |x − y|
2
x + x − y − y
n+1 n+1 n+1
≤ |x − y| |x + y| + |x − y|
= (|x + y| + 1) |x − y|
≤ 3 |x − y|
Let  > 0 be given. Taking δ < 3 we can conclude that, for any x ∈ [0, 1],
|x − y| < δ implies |fn (x) − fn (y)| <  for all n. Hence (fn ) is equicontinuous.

104
Problem 11.8. Compute and plot the Bernstein polynomials for 1/(1 + x) on [0, 1] of
degrees 1, 2, and 3.

105
Problem 11.9. Assume that (fn ) is a sequence of functions with fn : [a, b] → R for all
n where we take the usual metric on [a, b] and R.
(a) Assume that {fn } is uniformly Lipschitz continuous on [a, b] with constant L. Prove
that {fn } is equicontinuous on [a, b].
(b) Assume that {fn } is uniformly Lipschitz continuous on [a, b] with constant L and
further that (fn (x)) is a uniformly bounded sequence of numbers for some a ≤ x ≤ b.
(1) Prove that (fn ) has a subsequence that converges uniformly on [a, b]. (2) Prove
the second assumption is needed with an example.
(c) Assume that (fn ) is a bounded subset of C([a, b]). Show that the sequence of functions
(Fn ) from [a, b] into R defined by
Z x
Fn (x) = fn (x) ds
a

contains a subsequence that converges uniformly on [a, b]. (This is quite a remark-
able result when considering the sequence of functions (sin(nx)) as an example of a
bounded sequence in C([a, b]) that fails to have any convergent subsequence.)
Solution. Definition: A function f : X → R on a metric space (X, d) is Lipschitz
continuous on X if there is a constant C ≥ 0 such that

|f (x) − f (y)| ≤ Cd(x, y), ∀x, y ∈ X

(a) Since (fn ) is uniformly Lipschitz continuous there is a constant L ≥ 0 such that

|fn (x) − fn (y)| ≤ Ld(x, y), ∀x, y ∈ X, ∀n

So, if L = 0 then the functions are the constant functions and there is nothing to
prove. So we consider more interesting case L > 0.

Let  > 0 be given. Taking 0 < δ < L we can conclude that, for any x ∈ [a, b],
d(x, y) < δ implies |fn (x) − fn (y)| ≤ L L =  for all n. Therefore (fn ) is equicon-
tinuous.
(b) Since (fn ) is bounded equicontinuous family of functions, (fn ) is precompact and
hence has a convergent subsequence. The simple constant sequence (fn ) = (n)
has uniform Lipschitz constant 0, but clearly fails to converge even pointwise.
(c) Let M be the uniform bound for the set {fn }.
Z x Z x

|Fn (x)| = fn (s)ds ≤ |fn (s)| ds ≤ M (x − a) ≤ M (b − a)
a a

Therefore, (Fn ) is uniformly bounded. Also for any x, y ∈ [a, b]


Z x Z y Z x Z x

|Fn (x) − Fn (y)| =
fn (s)ds − fn (s)ds ≤
|f (s)| ds ≤ M ds ≤ M |x − y| .
a a y y

So, {Fn } is equicontinuous. Now by part (b), (Fn ) has a subsequence that con-
verges uniformly.
106
Problem 11.10. Suppose K = {x1 , . . . , xn } is a finite space with the discrete metric d.
Show C(K) is linearly isomorphic to the finite-dimensional space Rn with the ∞-norm
(which is just the maximum norm in finite dimensions). Hint: Any function f : K → R
is just a mapping of each xi to some point yi ∈ R, so the function can be identified with
a point y = (y1 , . . . , yn ) ∈ Rn where f (xi ) = yi . This is a linear isometry that is also a
metric space isomorphism with the metric induced by the ∞-norm.

107
Problem 11.11. Suppose f is a real-valued continuous function with dom(f ) = R. Let
fn (x) = f (nx) for n ∈ N and any x ∈ R. If {fn } is equicontinuous, prove that f must be
a constant function on [0, ∞).

Solution. Suppose f is not constant then there are x, y ∈ [0, ∞) with x 6= y and f (x) 6=
f (y). Let  = |f (x) − f (y)| > 0. Then |fn (x/n) − fn (y/n)| =  for all n ∈ N. When n
is large enough x/n, y/n ∈ [0, 1] and |x/n − y/n| → 0. So for any δ > 0 there is N such
that |x/n − y/n| < δ for all n ≥ N but |fn (x/n) − fn (y/n)| =  which is contradiction
to the fact that {fn } is equicontinuous.

108
Problem 11.12. Suppose that (fn ) is an equicontinuous sequence of functions on a
compact set K and (fn ) converges pointwise on K. Prove that (fn ) converges uniformly
on K.

Solution. Let  > 0 be given.


Since (fn ) is equicontinuous, for a fixed x ∈ K, there exists a δ > 0 such that dK (x, y) < δ
implies dY (fn (x), fn (y)) < 3 for all n ∈ N. Since K is compact, it is totally bounded
and there is a finite δ − net, that covers K. For any x ∈ K there exists xi such that
x ∈ Bδ (xi ) where xi ∈ δ − net. Therefore, dK (x, xi ) < δ and hence by equicontinuity
dY (fn (x), fn (xi )) < 3 .

Since fn → f is pointwise convergent for a fixed xi ∈ K, fn (xi ) converges to f (xi )


and so (fn (xi )) is Cauchy sequence. There exists an Ni such that n, m ≥ Ni implies
dY (fn (xi ), fm (xi )) < /3.
Now take N = max Ni then for all xi in the set of δ − net. So for any xi ,

n, m > N ⇒ dY (fn (xi ), fm (xi )) < /3

For any n, m > M we have,


  
dY (fn (x), fm (x)) ≤ dY (fn (x), fn (xi ))+dY (fn (xi ), fm (xi ))+dY (fm (xi ), fm (x)) ≤ + + = 
3 3 3
for all x ∈ K, showing that (fn ) is uniformly Cauchy and since Y is complete, (fn ) is
uniformly convergent.

109
Problem 11.13. Let X and Y be two metric spaces and A ⊂ X. Let (fn ) be a sequence of
functions from A to Y that converges to f pointwise on A. Prove the following theorems.
(a) Let Mn = supx∈A dY (fn (x), f (x)). Prove that fn → f uniformly if and only if Mn →
0.
(b) If fn is continuous on A for each n, then f is continuous on A. This is a generalization
of the theorem in the notes.
(c) Suppose Y is complete, then fn converges uniformly on A if and only if for every  > 0
there exists an integer N such that m ≥ N , n ≥ N , x ∈ E implies dY (fn (x), fm (x)) <
. Construct an example showing the necessity of Y being complete.
(d) Suppose Y is complete, and fn converges uniformly on A. Let x be a limit point of A
and suppose that limt→x fn (t) = yn for each n. Then (yn ) converges and limt→x f (t) =
limn→∞ yn . In other words, prove limt→x limn→∞ fn (t) = limn→∞ limt→x fn (t). Con-
struct an example showing the necessity of Y being complete.

110
R1
Problem 11.14. Prove that if f ∈ C([0, 1]) and 0 f (x)xn dx = 0 for all n ∈ N, then
R1
f (x) = 0 on [0, 1]. Hint: If 0 f 2 (x) dx = 0, then a result from elementary real analysis
states that f ≡ 0. Consider a sequence of polynomials that converge uniformly to f and
take the product with f to get a uniformly convergent sequence to f 2 .

111
Problem 11.15. Let (X, d) be a metric space and pick any fixed but arbitrary a ∈ X.
Define the real-valued function fp for each p ∈ X by

fp (x) = d(x, p) − d(x, a),

where x ∈ X.
(a) Prove that kfp k∞ ≤ d(a, p) and fp ∈ C(X).
(b) Prove that
kfp − fq k∞ = d(p, q)
for all p, q ∈ X.
(c) Let Φ(p) = f (p), so that Φ is an isometry of X onto Φ(X) ⊂ C(X). Let Y be the
closure of Φ(X) in C(X). Show that Y is complete which implies X is isometric to a
dense subset of a complete metric space Y . Note: This is a different proof of what we
previously showed in lecture 3 examples about defining the closure of a metric space.

112
n
Problem 11.16. Let fn (x) = (−1)n xn .

P
(a) Show that fn (x) converges uniformly on [0, 1].
n=1
P∞
(b) Show that |fn (x)| converges uniformly on [0, 1).
n=1
P∞
(1) Show that |fn (x)| does not converge uniformly on [0, 1).
n=1

Solution.

Theorem 11.7 (Leibniz’s Rule for Alternating Series). If {an } is a monotonic decreasing

(−1)n+1 an converges. If S denotes
P
sequence with limit 0, then the alternating series
n=1
its sum and sn denotes its nth partial sum, we also have the inequalities

0 < (−1)n (S − sn ) < an+1


∞ n
(−1)n xn converges pointwise since it is an alternat-
P
(a) For each x ∈ [0, 1] the series
n=1

P
ing series of decreasing positive terms approaching to 0. Define F (x) = fn (x)
n=1
for each x ∈ [0, 1]. To show that the series converges uniformly we need to show
that the uniform norm
n
X xj
k (−1)j − F (x)k∞ → 0
j=1
j

as n → ∞. By the Leibniz’s rule for alternating series we have,



n j
X
j x
(−1) − F (x) < |fn (x)|

j


j=1

for each x ∈ [0, 1] and there fore,


n
X xj 1
k (−1)j − F (x)k∞ < kfn (x)k∞ = → ∞, as n → ∞
j=1
j n

Therefore the series converges uniformly.


(b) Fix x ∈ [0, 1). We use ratio test.

|fn+1 (x)| nx
|fn (x)| = n + 1 → x, as n → ∞

, so for x ∈ [0, 1), the series converges pointwise.


(c) Fix an m ∈ N. Choose x ∈ [0, 1) close enough to 1 so that x2m > 12 . So, xj > 1
2
for all m + 1 ≤ j ≤ 2m. Then
∞ 2m 2m 2m
X xn X xn 1 X 1 1 X 1 1 1 1
> > > = m= .
n=m+1
n n=m+1
n 2 n=m+1 n 2 n=m+1 2m 2 2m 4

xn 1
P
Therefore, however large m we choose there is an x ∈ [0, 1) for which n
> 4
n=m+1
and hence the series does not converge uniformly.
113
Problem 11.17 (Rudin 1). Prove that every uniformly convergent sequence of bounded
functions is uniformly bounded.

Solution. Let (fn ) be a uniformly convergent sequence of bounded functions on a metric


space X. So, by Cauchy Criteria, there exists an N ∈ N such that for all m, n ≥ N we
have
|fn (x) − fm (x)| < 1
for all x ∈ X. Since each of the functions (fn ) are bounded there is Mn ∈ R such that
|fn (x)| < Mn for all x ∈ X. Thus if n ≥ N , then

|fn (x)| = |fn (x) − fN (x) + fN (x)| ≤ |fn (x) − fN (x)| + |fN (x)| < 1 + MN .

for all x ∈ X. Let M = max{1 + MN , M1 , . . . . . . MN −1 }. Then |fn (x)| ≤ M for all n ∈ N


and all x ∈ X.

114
Problem 11.18 (Rudin 2). If {fn } and {gn } converge uniformly on a set E, prove that
{fn + gn } converges uniformly on E. If in addition, {fn } and {gn } are sequences of
bounded functions, prove that {fn gn } converges uniformly on E.

Solution. See This Problem

Problem 11.19. Construct sequences {fn } and {gn } which converges uniformly on some
set E, but such that {fn gn } does not converge uniformly on E.

Solution. See This Problem

115
Problem 11.20. Consider ∞
X 1
f (x) =
n=1
1 + n2 x
For what values of x does the series converge absolutely? On what intervals does it
converge uniformly? On what intervals does it fail to converge uniformly? Is f continuous
whatever the series converges? Is f bounded?

Solution. • Where it converges: This series converges for all x except 0 and

x = − n12 , n = 1, 2, . . . . . . . For x = 0, f (x) = 1 = ∞. For x = − n12 the nth
P
n=1
term is not defined. For all other values of x the series converges.

By Weierstrass M-test the series converges uniformly on the interval [δ, ∞) if


δ > 0, since on that interval
1 1
2
≤ 2 .
1+n x nδ
Likewise, the seriesqconverges uniformly on (−∞, −δ] except at the points x =
− n12 , since for n ≥ 2δ we have,

116
Problem 11.21. Rudin Chapter 7, Problem 5:
Let 
1
0 x < n+1


fn (x) = sin2 πx n+1 1
≤x≤ 1
n

 1
0 < x.

n
P
Show that {fn } converges to a continuous function, but not uniformly. Use the series fn
to show that absolute convergence, even for all x, does not imply uniform convergence.

Solution. The limit of fn (x) is zero. If x ≤ 0 or x ≥ 1, then fn (x) = 0 for all n, and so
this assertion is obvious. If 0 < x < 1, then fn (x) = 0 for all n ≥ x1 , and so once again
the assertion is obvious.
The convergence is not uniform, since, no matter how large n is taken, there is a point
1
x, namely x = 2n+ 1 , for which fn (x) = 1.
2
The series converges to 0 for x ≤ 0 and x ≥ 1 and to sin2 πx for 0 < x < 1. Since the
terms are nonnegative, the series obviously converges absolutely. Since the sum is not
continuous at 0, the series does not converge uniformly on any interval containing 0.

117
Problem 11.22. Rudin Chapter 7, Problem 6:
Prove that the series ∞
X x2 + n
(−1)n
n=1
n2
converges uniformly in every bounded interval, but does not converge absolutely for any
value of x.

Solution. The series is the sum of two series


∞ n
X (−1)n X (−1)n
x2 +
n=1
n2 n=1
n

The first of these converges both uniformly and absolutely on any bounded interval [a, b]
because by Weierstrass M test
2 (−1)n M 2

x ≤
n2 n2

M2
P
where M = max(|a| , |b|) and n2
converges.
n=1

The second sum is independent of x and converges, hence it converges uniformly in


∞ 2
(−1)n x n+n
P
x. Being a sum of two uniformly convergence sequence the resulted sum 2
n=1
also uniformly
converges
(Rudin Exercise 7.2).
n x2 +n 1
P1
Since (−1) n2 ≥ n for any x, and does not converge we conclude that the series

n
does not converge absolutely.

118
Problem 11.23. Rudin Chapter 7, Problem 7:
For n = 1, 2, 3, . . . . . . , x real, put
x
fn (x) =
1 + nx2
Show that {fn } converges uniformly to a function f , and that the equation f 0 (x) =
lim fn0 (x) is correct if x 6= 0, but false if x = 0.
n→∞
√ √
Solution. Since (1 − nx)2 ≥ 0 implies 1 + nx2 ≥ 2 nx (Schwarz inequality) we have

≤ √|x| = √
x 1
|fn (x)| ≤ 2
1 + nx 2 n |x| 2 n
if x 6= 0. Also note that fn (0) = 0 for all n. Therefore fn (x) tends to uniformly to 0.
1−nx2
Now fn0 (x) = (1+nx 0
2 )2 , which tends to 0 if x 6= 0, though fn (0) = 1 for all n

119
Problem 11.24. Rudin Chapter 7, Problem 8: If

0 x ≤ 0
I(x) =
1 x > 0
P
if {xn } is a sequence of distinct points of (a, b), and if |cn | converges, prove that the
series ∞
X
f (x) = cn I(x − xn ) (a ≤ x ≤ b)
n=1
converges uniformly, and that f is continuous for every x 6= xn .
P
Solution. If |cn | converges then for any m ≥ n,
n

Xm X m
cj I(x − xj ) ≤ |cj | , ∀x ∈ [a, b]



j=n j=n

By Weierstrass M test, the series converges uniformly on [a, b].


Let  > 0 be given. There exists N such that

X 
cj I(x − xj ) < , ∀y ∈ [a, b].

3


j≥N
P
If x 6= xn for any n then each of the functions and hence the sum cj I(y−xj ) is continu-
j<N
P P
ous at x, so there exists δ > 0 such that |x − y| < δ ⇒ cj I(x − xj ) − cj I(x − xj ) <

j<N j<N

3
. Each of the functions cj I(x − xj ) are continuous because the only discontinuity occurs
at xj .

Then we see that , if |x − y| < δ,



X X X X
|f (x) − f (y)| ≤ cj I(x − xj ) − cj (y − xj ) + cj I(x − xj ) + cj I(y − xj ) < .


j<N j<N j≥N j≥N

Thus, f is continuous at x.

120
Problem 11.25. Rudin Chapter 7, Problem 9
Let {fn } be a sequence of continuous functions which converges uniformly to a function
f on a set E. Prove that
lim fn (xn ) = f (x)
n→∞
for every sequence of point xn ∈ E such that xn → x and x ∈ E. Is the converse of this
true?

Solution. Let  > 0 be given. Since fn converges uniformly on E, there exists an N1 ∈ N


such that for all n ≥ N1 , |fn (x) − f (x)| < 2 for all x ∈ E. Also since fn is continuous for
all n, there exits δ > 0 such that |fn (x) − fn (y)| < 2 whenever |x − y| < δ. Now since
xn → x, there exists an N2 ∈ N such that |xn − x| < δ whenever n ≥ N2 . Now taking
N = max(N1 , N2 ) we get,

|fn (xn ) − f (x)| ≤ |fn (xn ) − fn (x)| + |fn (x) − f (x)| < 

for all n ≥ N and therefore lim fn (xn ) = f (x).


n→∞

121
1
Problem 11.26. Let fn (x) = (1 + xn ) n for x ≥ 0. Does the sequence converge pointwise
on E = [0, ∞)? If so, find the limit function. Check whether or not the convergence is
uniform on E.

Solution. If 0 ≤ x ≤ 1 then
1  n1
(1 + xn+1 ) 1+n 1 + xn+1

fn+1 (x)
= ≤ ≤ 11/n = 1
fn (x) (1 + xn )1/n 1 + xn
Therefore {fn (x)} is monotonically decreasing for 0 ≤ x ≤ 1. Now if x > 1 then we write
n 1
fn (x) = x(1 + n1 ) n . Since 0 < x1 < 1 we conclude that
1
1
fn+1 x(1 + xn+1 ) n+1
= 1 ≤1
fn x(1 + x1n ) n
Therefore, the for x ∈ [0, ∞) fn is monotonically decreasing and bounded bellow by x
because 1 + xn ≥ xn ⇒ (1 + xn )1/n ≥ x for all x ≥ 0. Hence the sequence converges
pointwise for each x ∈ [0, ∞).
For x > 1,
1
lim (1 + xn )1/n = x lim (1 + n )1/n = x.
n→∞ n→∞ x
For x < 1,
lim (1 + xn )1/n = 1.
n→∞
Now to determine uniform convergence. First consider the case x > 1

Mn = sup |(1 + xn )1/n − x|


x∈[1,∞)

d (1 + xn )1/n xn
(1 + xn )1/n − x = (1 + xn )1/n−1 xn−1 − 1 = −1
dx x(1 + xn )
Observe that,  
n n n2 n
(1 + x ) = x + (xn )n−1 + . . . · · · + 1
1
2
(1 + xn )n ≥ xn + (xn )n−1
2
xn (1 + xn )n ≥ xn (xn + (xn )n−1 )
2
xn (1 + xn )n ≥ xn (xn + 1)
x(1 + xn ) ≥ xn (1 + xn )1/n
xn (1 + xn )1/n
≤1
x(1 + xn )
xn (1 + xn )1/n
−1<0
x(1 + xn )
Therefore the function is decreasing and hence its maximum occurs at x = 1. Therefore
Mn = 21/n − 1 → 0 and so fn → x uniformly.
122
Now consider the case when x ≤ 1

Mn = sup |(1 + xn )1/n − 1|.


x∈[0,1]

Since derivative of (1 + xn )1/n − 1 is (1 + xn )1/n−1 xn−1 which is positive for 0 ≤ x ≤ 1 and


hence the increasing. So maximum occurs at x = 1. So, Mn = 21/n − 1 → 0. Therefore
for x < 1 we get fn → 1 uniformly. We conclude that the given sequence of function
uniformly convergent to, 
x if x ≥ 1
f (x) =
1 if 0 ≤ x ≤ 1

123
Problem 11.27. Let {fn } be a sequence of continuous real functions on R, and that
fn → f uniformly on R. Let gn (x) = fn (x + n1 ) for all n = 1, 2, . . . . . . , and x ∈ R. Show
that {gn } converges uniformly to f on any bounded interval [a, b].

Solution. Since fn is continuous on a compact set [a, b], we can assert that fn is uniformly
continuous on [a, b]. Hence for given  > 0 there is δ > 0 such that

|x − y| < δ ⇒ |fn (x) − fn (y)| < /2


1
By Archimedean property for any δ > 0 there is N1 ∈ N such that n
< δ for all n ≥ N1 .
So,
1 1
|(x + ) − x| < < δ ∀n ≥ N1
n n
Hence for given  > 0 there is N1 ∈ N such that,
1 1
n ≥ N1 ⇒ |(x + ) − x| < δ ⇒ |fn (x + ) − f (x)| < /2
n n
Since fn → f uniformly there is N2 ∈ N such that

|fn (x) − f (x)| < /2 ∀n ≥ N2 , ∀x ∈ [a, b]

Take N = max(N1 , N2 ), then we get,


1
|gn (x) − f (x)| = |fn (x + ) − f (x)|
n
1
≤ |fn (x + ) − fn (x)| + |fn (x) − f (x)|
n
 
≤ + =
2 2
∀n ≥ N and for all x ∈ [a, b].
Therefore {gn } is uniformly convergent to f on [a, b]

124
Problem 11.28. Consider the series

X 1
f (x) = .
n=1
1 + n2 |x|

(a) For what values of x does the series converge?


(b) Characterize the subsets of R on which the series converges uniformly.
(c) Ia f continuous on its domain?
(d) Is f bounded on its domain?

Solution.
(a) For x = 0 the given series diverges.
For x 6= 0 we get
∞ ∞
X 1 1 X 1
2
= 1
n=1
1 + n |x| |x| n=1 n +
2
|x|
1 1
Since 1
n2 + |x|
< n2
by comparison test we can say that the given series converges.
(b) For any intervals of the form [a, b] with either a > 0 or b < 0 we get,
1 1 1
| 2
|≤ 2
≤ 2 forc = max(|a|, |b|)
1 + n |x| 1+n c nc
Since ∞ 1
P
n=1 n2 c < ∞ by Weierstrass M test the series convergence uniformly.
Now consider the interval (0, b]
Suppose the series converges uniformly in (0, b]. Then the sequence of partial sum sn =
Pn 1
n=1 1+n2 |x| converges uniformly. By Cauchy criterion of uniform convergence there
exists N ∈ N such that
n
1 X 1 1
|sn − sm | < ⇒ | 2
|< for ∀n > m ≥ N, ∀x ∈ (0, b]
3 k=m+1
1 + n |x| 3

But this is not true for x = N12 . Hence the series is not uniformly convergent on the
interval of the form (0, b].
(c) 1+n12 |x| is a sequence of continuous functions. So (by the corollary after the theorem
of uniform convergence and continuity) f is continuous on the intervals where the series
uniformly converges to f .
(d) The function f is not bounded. For example lim f (x) = ∞.
x→0

125
n
Problem 11.29. For each positive integer n, let fn (x) = n2 +xn
. Show that the sequence
{fn } converges uniformly to f (x) ≡ 0 on E = [0, ∞)

126
12. Differential Calculus and Banach Space

Definition 12.1. A linear map or linear operator T between real (or complex) linear
spaces X, Y is a function T : X → Y such that

T (λx + µy) = λT x + µT y, ∀ λ, µ ∈ R (or C) and x, y ∈ X.

A linear map T : X → X is called a linear transformation of X, or a linear operator on


X. If T : X → Y is one-to-one and onto, then we say that T is nonsingular or invertible,
and define the inverse map T −1 : Y → X by T −1 y = x if and only if T x = y, so that
T T −1 = IY , T −1 T = IX , where IY and IX are identity maps.

Definition 12.2. Let X and Y be two normed linear spaces with norms k·kX and k·kY ,
respectively. A linear map T : X → Y is bounded if there exists a constant M ≥ 0 such
that
kT xkY ≤ M kxkX , ∀ x ∈ X.
If no such constant exists, then we say that T is unbounded. If T : X → Y is a bounded
linear map, then we define the operator norm kT k of T by

kT k = inf {M : kT xkY ≤ M kxkX ∀ x ∈ X} .

We denote the set of all linear maps T : X → Y by L(X, Y ), and the set of all bounded
linear maps T : X → Y by B(X, Y ). When the domain and range spaces are the same,
we write L(X) and B(X).

Theorem 12.1. A linear map is bounded if and only if it is continuous.

Theorem 12.2 (Bounded linear transformation). Let X be a normed linear space and Y
a Banach space. If M is a dense linear subspace of X and T : M ⊂ X → Y is a bounded
linear map, then there exists a unique bounded linear map T̄ : X → Y such that T̄ x = T x

for all x ∈ M and T̄ = kT k.

Definition 12.3. Two linear spaces X and Y are linearly isomorphic if there is a one-
to-one, onto linear map T : X → Y . If X and Y are normed linear spaces and T, T −1
are bounded linear maps, then X and Y are topologically isomorphic. If T also preserves
norms, meaning kT xkY = kxkX for all x ∈ X, then X and Y are isometrically isomorphic.

Theorem 12.3. If X is a normed linear space and Y is a Banach space, then B(X, Y )
is a Banach space with respect to the operator norm.

Definition 12.4. A scalar-valued linear map from a linear space X to R is called a


linear functional or linear form on X. The space of linear functionals on X is called the
algebraic dual space of X, and the space of continuous linear functionals on X is called
the topological dual space of X.

Theorem 12.4 (Hahn-Banach). If Y is a linear subspace of a normed linear space X


and ψ : Y → R is a bounded linear functional on Y with kψk = M , then there exists
127
a bounded linear functional φ : X → R such that φ restricted to Y is equal to ψ and
kφk = M .

Definition 12.5. A function f : (a, b) → X from an open interval (a, b) of real numbers
into a Banach space X is differentiable at t ∈ (a, b) with derivative f 0 (t) ∈ X if the
following limit exists in X:
f (t + h) − f (t)
f 0 (t) = lim .
h→0 h
The function f is continuously differentiable in (a, b) if f 0 : (a, b) → X is continuous.

Theorem 12.5. If f : (a, b) → X is differentiable in (a, b) and f 0 = 0, then f is a


constant function.

Theorem 12.6 (Mean value). If f is continuously differentiable in an open interval that


contains the closed, bounded interval [a, b], with values in a Banach space, then

kf (b) − f (a)k ≤ M (b − a) where M = sup kf 0 (t)k .


a≤t≤b

Definition 12.6. A map f : U ⊂ X → Y whose domain U is an open subset of a


Banach space X and whose range is a Banach space Y is differentiable at x ∈ U if there
is a bounded linear map A : X → Y such that
kf (x + h) − f (x) − AhkY
lim = 0.
h→0 khkX
We call the map A the Fréchet derivative and write A = f 0 (x). If f is differentiable at
each point of U , then f 0 : U → B(X, Y ) is the map that assigns to each point x ∈ U
the bounded linear map f 0 (x) : X → Y that approximates f near x. We say that f is
continuously differentiable at x if the map f 0 is continuous at x, where the domain U
is equipped with the norm on X and the range B(X, Y ) is equipped with the operator
norm.

Definition 12.7. Let X and Y be Banach spaces, and f : U ⊂ X → Y , where U is an


open subset of X. The directional derivative of f at x ∈ U in the direction h ∈ X is
given by
f (x + th) − f (x)
δf (x; h) = lim .
t→0 t
If this limit exists for every h ∈ X, and fG0 (x) : X → Y defined by fG0 (x)h = δf (x; h) is a
linear map, then we say that f is Gâteaux differentiable at x, and we call fG0 the Gâteaux
derivative of f at x.

Theorem 12.7. Suppose that f : U ⊂ X → Y is a Gâteaux differentiable function from


an open subset U of a Banach space X to a Banach space Y . If x, y ∈ U , and the line
segment {tx + (1 − t)y : 0 ≤ t ≤ 1} connecting x and y is contained in U , then

kf (x) − f (y)kY ≤ M kx − ykX where M = sup kfG0 (tx + (1 − t)y)k .


0≤t≤1
128
Theorem 12.8. Suppose that f : U ⊂ X → Y is a Gâteaux differentiable function from
an open subset U of a Banach space X to a Banach space Y . If the Gâteaux derivative
fG0 : U ⊂ X → B(X, Y ) is continuous at x ∈ U , then f is Fréchet differentiable at x and
f 0 (x) = fG0 (x).

Theorem 12.9 (Chain Rule). Suppose that X, Y, and Z are Banach spaces, and f : U ⊂
X → Y and g : V ⊂ Y → Z, where U and V are open subsets of X and Y , respectively. If
f is differentiable at x ∈ U and g is differentiable at f (x) ∈ V , then g ◦ f is differentiable
at x and
(g ◦ f )0 (x) = g 0 (f (x))f 0 (x).

Definition 12.8. If f : X × Y → Z is a differentiable map on the product of two Banach


spaces, then we have

f (x + h, y + k) = f (x, y) + Ah + Bk + o(h, k)

for suitable linear maps A : X → Z and B : Y → Z. We call A and B the partial


derivatives of f with respect to x and y, respectively, and denote them by A = Dx f (x, y)
and B = Dy f (x, y).

Theorem 12.10 (Inverse function). Suppose that f : U ⊂ X → Y is a differentiable


map from an open subset U of a Banach space X to a Banach space Y . If f is continu-
ously differentiable in U and f 0 (x) has a bounded inverse at x ∈ U , then there are open
neighborhoods V ⊂ U of x and W ⊂ Y of f (x) such that f : V → W is a one-to-one,
onto continuous map with continuous inverse f −1 : W → V . Moreover, the local inverse
is continuously differentiable at f (x) and

(f −1 )0 (f (x)) = [f 0 (x)]−1 .

Theorem 12.11 (Implicit function theorem). Suppose that X, Y , and Z are Banach
spaces, and F : U ⊂ X × Y → Z is a continuously differentiable map defined on an open
subset U of X × Y . If (x0 , y0 ) ∈ U is a point such that F (x0 , y0 ) = 0, and Dy F (x0 , y0 ) :
Y → Z is a one-to-one, onto, bounded linear map, then there is an open neighborhood
V ⊂ X of x0 , an open neighborhood W ⊂ Y of y0 , and a unique function f : V → W
such that
F (x, f (x)) = 0 ∀ x ∈ V.
The function f is continuously differentiable, and

f 0 (x) = −[Dy F (x, f (x))]−1 Dx F (x, f (x)).

129
13. Problems on Differentiation

Problem 13.1. Define the discontinuous real-valued function f : R2 → R by



 xy , if(x, y) 6= (0, 0),
2 2
f (x, y) = x +y
0, else.

prove that (the Gateaux derivatives) Dx f (x, y) and Dy f (x, y) exist everywhere in R2 even
though f is discontinuous at (0, 0).

Solution. For (x, y) 6= (0, 0), the existence of the derivative follows from the standard
rules of differentiation. By symmetry, we can easily determine both partial derivatives
with one computation.
f (t, 0) − f (0, 0) 0
Dx f (0, 0) = lim = lim = Dy f (0, 0).
t→0 t t→0 t

Thuse Gateaux derivatives Dx f and Dy f exists everywhere even though f is clearly not
continuous at (0, 0) because the limit lim f (x, y) along x axis is 0 but the same limit
(x,y)→(0,0)
1
along x = y line is 2
.

130
Problem 13.2. Suppose that f : A ⊂ Rn → R where A is an open set and the partial
derivatives D1 f, . . . , Dn f are bounded in A. Prove that f is continuous in A.

Solution. Consider any  > 0 and x = (x1 , x2 , . . . . . . , xn ) ∈ A ⊂ Rn .

Since the derivatives are bounded, let

M= max |Di f (y)| .


y∈A,1≤i≤n

Choose δ = (M +1)n
. Consider any y ∈ Bδ (x) ∩ A, and for 0 ≤ i ≤ n let

y (i) = (y1 , . . . , yi , xi+1 , . . . , xn )

So, y (0) = x and y (n) = y



|f (y) − f (x)| = f (y (n) ) − f (y (0) )

≤ f (y (n) ) − f (y (n−1) ) + f (y (n−1) ) − f (y (n−1) ) + dotsdots + f (y (1) ) − f (y (0) )
n
X (i)
= f (y ) − f (y (i−1) )
i=1

By the Mean Value Theorem for 1-D functions, we have that there exists ci in the
interval defined by xi and yi such that

f y (i) − f (y i−1 ) = |f (y1 , . . . , yi−1 , xi , . . . , xn ) − f (y1 , . . . , yi , xi+1 , . . . , xn )| = |Di f (ci )(xi − yi )| < M δ


Since the bound is independent of i we get,


M
|f (y) − f (x)| <  < .
M +1
Therefore f is continuous.

131
Problem 13.3. Suppose that f : A ⊂ Rn → R is differentiable, where A is an open set
and that f has a local maximum at a point x ∈ A. Prove f 0 (x) = 0.

Solution. Define g(t) = x + ty, from R to Rn , where y ∈ Rn and t ∈ R. Note that


g is nothing more than a parameterized line in Rn that passes through the local maxi-
mum point x ∈ A of the function f . Clearly the function g is differentiable on R. Now
g(0) = x ∈ A. Since A is open and g is continuous there exists a δ > 0 such that g(t) ∈ A
for any t ∈ (−δ, δ).

We define h(t) = f (g(t)). Note that h : R → R has a local maximum at t = 0 since f


has a local maximum at g(0) = x. If t is sufficiently close to 0 so that g(t) ∈ A, then h
is differentiable function of t by the Chain Rule, and

h0 (t) = f 0 (g(t))g 0 (t) = f 0 (x + ty)y

Hence h has a local maximum at t = 0 and is a differentiable real-valued function of one


variable, we have that
0 = h0 (0) = f 0 (x)y
Since y ∈ Rn is arbitrary, we have that f 0 (x) = 0.

132
Problem 13.4. Suppose that f : A ⊂ Rn → R is differentiable, where A is an open
connected set and f 0 (x) = 0 for every x ∈ A. Prove that f is constant in A.

Solution. Let x ∈ A be any point. Define y (i) = (y1 , . . . , yi , xi+1 , . . . , xn ) where y =


(y1 , y2 , . . . , yn ) ∈ A ⊂ Rn . Define the function hi : R → R as

hi (z) = f (y1 , . . . , yi−1 , z, xi+1 , . . . , xn )

So, hi (yi ) = f (y (i) ) and hi (xi ) = f (y (i−1) ) Since f is differentiable in A it is obvious that
hi is differentiable in the interval defined by xi and yi .

133
Problem 13.5. Rudin Exercise 9.17
Let f : R2 → R2 be defined by
! !
f1 (x, y) ex cos y
f (x, y) = = .
f2 (x, y) ex sin y
(a) Show that the range of f is R2 \ {(0, 0)}.
(b) Show that the Jacobian of f is non-zero on R2 . Thus, every point of R2 has a
neighborhood in which f is 1-1, but f is not 1-1 on R2 .
(c) Let (x̄, ȳ) = (0, π/3) and (ū, v̄) = f (x̄, ȳ). Let g = f −1 denote the continuous inverse
of f defined in a neighborhood of (ū, v̄) such that g(ū, v̄) = (x̄, ȳ). Find an explicit
formula for g, compute f 0 (x̄, ȳ), g 0 (ū, v̄) and show that g 0 (f (x))f 0 (x) = IR2 .
(d) What are the images under f of lines parallel to the coordinate axes?


Solution. (a) If (u, v) 6= (0, 0), chose x = ln u2 + v 2 . Notice that √u2u+v2 ≤ 1 and

so it is possible to choose y so that
u
cos y = √
u + v2
2

then
v
sin y = √ .
u2
+ v2
It is obvious from the equations defining x and y that u = ex cos y and v = ex sin y.
Hence every point except (0, 0) is the range of f . The point (0, 0) is not in the
range since
u2 + v 2 = e2x > 0
for any point (u, v) = f (x, y).
(b) The Jacobian of f (x, y) is
" #
x x
e cos y −e sin y
x x
= e2x 6= 0.
e sin y e cos y
However, since f (x, y) = f (x, y + 2π), follows that f is not one-one in R2 .
 √it 
(c) By our definition, b = f (0, π/3) = 12 , 23 . We can therefore take y = arctan v

u
near b, the arctangent being between −π/2 to π/2. Thus we have
 √  v 
g(u, v) = ln u2 + v 2 , arctan .
u
We then have, " #
u v
g 0 (u, v) = u2 +v 2
−v
u2 +v 2
u
u2 +v 2 u2 +v 2
When we take u = ex cos y and v = e sin y, we find that
x
" #
−x −x
e cos y e sin y
g 0 (f (x, y)) =
−e−x sin y e−x cos y
134
Therefore " #
1 0
g 0 (f (x, y))f 0 (x, y) =
0 1
(d) The family of lines x = c parallel to y-axis maps to the family of concentric circles
centered at origin with radius ec . The lines y = c parallel to the x axis maps to
rays starting from origin with the angle c − 2nπ for n = 0, 1, . . . . . . with positive
horizontal axis.

135
Problem 13.6. If X = Y = Z = R in the implicit function theorem, interpret the
theorem and its proof graphically.

136
Problem 13.7. Define f : R3 → R by f (x, y, z) = x2 y+ex +z. Show that f (0, 1, −1) = 0,
Dx f (0, 1, −1) 6= 0, and that there exists a differentiable function g in some neighborhood
of (1, −1) in R2 such that g(1, −1) = 0 and f (g(y, z), y, z) = 0. Find Dy g(1, −1) and
Dz g(1, −1).

137
Rb
Problem 13.8. Suppose f ≥ 0, f is continuous on [a, b], and a
f (x) dx = 0. Prove that
f (x) = 0 for all x ∈ [a, b].

138
Problem 13.9. Let p and q be positive real numbers such that 1/p + 1/q = 1. Prove
the following statements.
(a) If u ≥ 0 and v ≥ 0, then
up v q
uv ≤ + ,
p q
p q
and equality holds if and only if u = v .
Rb Rb Rb
(b) If f, g ∈ R([a, b]), f, g ≥ 0, and a f p = 1 = a g q , then a f g ≤ 1.
(c) If f, g, ∈ R([a, b]), then
Z b Z b 1/p Z b 1/q
p q


f g ≤
|f | |g| .
a a a

139
hR i1/2
b 2
(1) For f ∈ R([a, b]) define kf k2 = a
|f | . Suppose f, g, h ∈ R([a, b]). Prove the
triangle inequality

kf − hk2 ≤ kf − gk2 + kg − hk2 .

140
(2) Show that for any f ∈ R([a, b]) and  > 0 that there exists g ∈ C([a, b]) such that
kf − gk2 < .

141
Problem 13.10. Rudin Exercise 9.7
Suppose that f is a real valued function defined in an open set E ⊂ Rn , and that the
partial derivatives D1 f, . . . . . . , Dn f are bounded in E. Prove that f is continuous in E.

Solution. Fix x ∈ E and  > 0. Since E is open, there is an open ball S0 ⊂ E, with
center at x and radius r. Let M = max{D1 f (x), . . . . . . , Dn f (x)}. Choose
x∈E

δ = min{r, }.
(n + 1)M
P
The open ball S ⊂ E centered at x and radius δ. Suppose y − x = hj ej with |h| < δ.
Put v0 = 0 and vk = h1 e1 + . . . · · · + hk ek , for 1 ≤ k ≤ n. Then
n
X
f (y) − f (x) = [f (x + vj ) − f (x + vj−1 )] .
j=1

Since vk < δ for 1 ≤ k ≤ n and since S is convex, the segments with end points x + vj−1
and x + vj lie in S. Since vj = vj−1 + ej hj , the mean value theorem shows that the jth
summand is equal to
jj (Dj f )(x + vj−1 + θj hj ej )
for some θj ∈ (0, 1).
n
X n
X
|f (y) − f (x)| ≤ M |hj | ≤ M δ < .
j=1 j=1

Which shows that |y − x| < δ implies |f (y) − f (x)| <  and hence f is continuous.

142
Problem 13.11. Rudin Exercise 9.8
Suppose f is differentiable real function in an open set E ⊂ Rn , and that f has a local
maximum at a point x ∈ E. Prove that f 0 (x) = 0.

Solution. We will first prove for single variable case.

Definition 13.1. Let f be a real function defined on a metric space X. We say that f
has a local maximum at a point p ∈ X if there exists δ > 0 such that f (q) ≤ f (p) for all
q ∈ X with d(p, q) < δ.

Choose δ in accordance with above definition of local maximum, so that

a < x − δ < x < x + δ < b.

If x − δ < t < x, then


f (t) − f (x)
≥ 0.
t−x
Letting t → x, we see that f 0 (x) ≥ 0.
If x < t < x + δ, then
f (t) − f (x)
≤ 0,
t−x
which show that f 0 (x) ≤ 0. Hence f 0 (x) = 0

Let u be any vector in Rn . Consider the real valued function φ(t) = f (x + tu) defined
near t = 0. The function φ(t) is differentiable by the Chain rule, and has a maximum
at t = 0. Therefore φ0 (0) = 0 which implies, by chain rule, f 0 (x)(u) = 0. Since u is
arbitrary, it follows by the definition of zero linear transformation that f 0 (x) = 0.

143
Problem 13.12. Rudin Exercise 9.9
If f is differentiable mapping of a connected open set E ∈ Rn into Rm , and if f 0 (x) = 0
for every x ∈ E, prove that f is constant in E.

Solution. Pick some point x0 ∈ E and define,

C = {x ∈ E : f (x) = f (x0 )}.

C is open.
Let x ∈ C. Then f (x) = f (x0 ). Since x ∈ E and E is an open set, there is a neighborhood
B (x) ⊂ E for some  > 0. Let y ∈ B (x). So, [x, y] ⊂ E and hence by Mean Value
Theorem,
|f (x) − f (y)| ≤ kf 0 (z)k |x − y|
for some z ∈ [x, y]. By the given hypothesis, |f (x) − f (y)| ≤ 0 ⇒ f (x) = f (y) = f (x0 )
and hence y ∈ C. Therefore B (x) ⊂ C and hence C is open.
C is closed.
Let x ∈ E \ C. Therefore f (x) 6= f (x0 ). Since E is open B (x) ⊂ C for some  > 0.
Let y ∈ B (x). So, [x, e] ⊂ E and using Mean Value Theorem we can show that f (x) =
f (y) 6= f (x0 ) and therefore y ∈ E \ C. Hence B (x) ∩ C = ∅ which implies that x can
not be a limit point of C. Since x is arbitrarily chosen we conclude that C is closed.
Since E is connected the only subset of E that is open and closed is the set E itself.
Hence C = E and therefore f (x) = f (x0 ) for all x ∈ E and therefore f is constant.

Problem 13.13. Rudin Exercise 9.13


Suppose f is a differentiable mapping of R1 into R3 such that |f (t)| = 1 for every t. Prove
that f 0 (t).f (t) = 0

Solution.
|f (t)| = 1 ⇒ f (t).f (t) = 1
Differentiating and using product rule

f 0 (t).f (t) = 0

144
Problem 13.14. Rudin Exercise 9.14
Define f (0, 0) = 0 and
x3
f (x, y) = 2 , if (x, y) 6= (0, 0).
x + y2
(a) Prove that D1 f and D2 f are bounded function in R2 .
(b) Let u be any unit vector in R2 . Show that the directional derivative (Du f )(0, 0)
exists, and that its absolute value is at most 1.
(c) Let γ be a differentiable mapping of R1 into R2 , with γ(0) = (0, 0) and |γ 0 (0)| > 0.
Put g(t) = f (γ(t)) and prove that g is differentiable for every t ∈ R1 . If γ ∈ C 1 ,
prove that g ∈ C 1 .
(1) In spite of this, prove that f is not differentiable at (0, 0).

Solution. For (x, y) 6= (0, 0),


x4 − 3x2 y 2 3x2 (x2 + y 2 ) 3x2
D1 f = ≤ ≤ ≤3
(x2 + y 2 )2 (x2 + y 2 )2 x2 + y 2
−2x3 y 2

|D2 f | = 2
≤ x ≤ 1.
(x + y 2 )2 x2 + y 2
Also,
f (t, 0) − f (0, 0) t−0
D1 f (0, 0) = lim = lim =1
t→0 t t→0 t
and
f (0, t) − f (0, 0)
D2 f (0, 0) = lim = 0.
t→0 t
Hence, as asserted, f (x, y) is continuous.
Let u = (a, b) be an unit vector. Then,
f (ta, tb) − f (0, 0) t 3 a3 a3
Du f (0, 0) = lim = lim 2 2 = .
t→0 t t→0 t(t a + t2 b2 ) a2 + b 2
Since |u| = a2 + b2 = 1 this implies |a| ≤ 1. Hence we have

Du f (0, 0) = a3

and

|Du f (0, 0)| = a3 ≤ 1
Since both the partial derivatives are continuous in R2 possibly at origin, the function
f is differentiable every where except possibly at origin. and So by the chain rule g is
differentiable everywhere except possib

145
Problem 13.15. Rudin Exercise 9.19
Show that the system of equations
3x + y − z + u2 = 0
x − y + 2z + u = 0
2x + 2y − 3z + 2u = 0
can be solved for x, y, u in terms of z; for x, z, u in terms of y; for y, z, u in terms of x;
but not for x, y, z in terms of u.

Solution. We note if (f, g, h)(x, y, z, u) = (0, 0, 0), then f − g − h = 0. Expand-


ing this gives u2 − 3u = 0, so we must have u = 0 or u = 3. Hence, there is no
nonempty open interval I ⊂ R with a γ : I → R3 , γ(u) = (x(u), y(u), z(u)) satisfying
(f, g, h)(x(u), y(u), z(u), u) = (0, 0, 0) as we would have I ⊂ {0, 3}. That is, we cannot
solve for x, y, z in terms of u. For the others, by the implicit function theorem the claim
reduces to checking that various 3×3 submatrices of F 0 are invertible, where F : R4 → R3
by F = (f, g, h). We have
 
3 1 −1 2u
F 0 (x, y, z, u) = 1 −1 2 1
 

2 2 −3 2
And the matrices to consider are, for x, y, u in terms of z,
 
3 1 2u
1 −1 1 
 

2 2 2
which has determinant 8u − 12. As points we invert around have values u = 0 or u = 3,
this is invertible on both cases and so we may solve for x, y, u in terms of z. For solving
for x, z, u in terms of y, the matrix to consider is
 
3 −1 2u
1 2 1
 

2 −3 2
which has determinant 21 − 14u. As before, this is invertible for u = 0 or u = 3. Lastly,
for solving for y, z, u in terms of x, the matrix to consider is
 
1 −1 2u
−1 2 1
 

2 −3 2
with determinant 3 − 2u. As before, this is invertible for u = 0 or u = 3.

146
14. List of Problems

14.1. Problems in Norm and Metric

Problem 1Show that d(x, y) = kx − yk is a metric on Rn where k·k is the standard


Euclidean norm on Rn .
Problem 2Determine which of the following define metrics on R.

(a) d1 (x, y) = (x − y)2 ,


p
(b) d2 (x, y) = |x − y|,
(c) d3 (x, y) = |x2 − y 2 |,
(d) d4 (x, y) = |x − 2y|,
(e) d5 (x, y) = |x − y| /[1 + |x − y|].

Problem 3 For f, g : [0, 1] → R, let d1 (f, g) = supx∈[0,1] {x2 |f (x) − g(x)|}, d2 (f, g) =
R1
supx∈[0,1] |f (x) − g(x)|, and d3 (f, g) = 0 |f (x) − g(x)| dx. Prove that d1 , d2 , and d3 de-
fine metrics on the linear space of C([0, 1]) of continuous functions f : [0, 1] → R.

Problem 4 Show that Rn with d(x, y) = kx − yk∞ is a metric space, where kxk∞ =
supxi |xi | for i = 1, 2, . . . , n.

Problem 5 Consider the convergence of sequences (xn ) = (ξn,1 , ξn,2 , ξn,3 , · · · ) to a point
y = (η1 , η2 , η3 , · · · ) in `2 . Show that if xn → y, then ξn,i → ηi for i = 1, 2, 3, · · · . But,
construct an example that shows that ξn,i → ηi for i = 1, 2, 3, · · · does not imply that
xn → y. (This is similar to the idea of pointwise convergence not guaranteeing metric-
convergence for sequences of functions.) Consider the Hilbert Cube subspace of `2 defined
by,
H = x = (ξ1 , ξ2 , ξ3 , · · · ) : x ∈ `2 and |ξi | ≤ 1/i, i = 1, 2, 3, · · · .


Show that a sequence (xn ) in H converges to a point y = (η1 , η2 , η3 , · · · ) if and only if


ξn,i converges to ηi for i = 1, 2, 3, · · · .

Problem 6 Let p ≥ 1 be a fixed real number. The real space `p is defined such that
x ∈ `p is a sequence of real numbers x = (ξj ) = (ξ1 , ξ2 , ξ3 , · · · ) such that ∞ p
P
j=1 |ξj | < ∞.
For x, y ∈ `p , where x = (ξj ) and y = (ηj ), let d(x, y) be defined by

!1/p
X
d(x, y) = |ξj − ηj |p .
j=1

Show that `p with d defined above is a metric space.

Problem 7 Let `∞ be the space defined by real-valued bounded sequences, i.e., x ∈ `∞


implies x = (ξi ) = (ξ1 , ξ2 , · · · ) such that |zetai | ≤ cx for all j ∈ N where cx ≥ 0 is some
real number that may depend on x but not on j. For x, y ∈ `∞ , where x = (ξi ) and
147
y = (ηi ), show that
d(x, y) = sup |ξj − ηj |
j∈N

defines a metric on ` .

Problem 8 Clearly `1 ⊂ `∞ . Is `1 dense in `∞ ? Is `1 closed in `∞ ? Justify your an-


swers.

14.2. Cauchy and Compactness

Problem 9 Prove that if a subsequence of a Cauchy sequence in a metric space converges,


then the full sequence converges to the same limit.

Problem 10 Let (xn ) and (yn ) be Cauchy sequences in a metric space (X, d). Show
that (d(xn , yn )) converges whether or not (xn ) and (yn ) converge.

Problem 11 Let (X1 , d1 ) and (X2 , d2 ) be two complete metric spaces. Is the product
metric space X1 × X2 complete?

Problem 12 Prove that a metric space X is complete if and only if the intersection
of every descending sequence of closed balls whose radii approach zero consists of a single
point.

Problem 13 For f, g : [0, 1] → R, let d1 (f, g) = supx∈[0,1] {x2 |f (x) − g(x)|}, d2 (f, g) =
R1
supx∈[0,1] |f (x) − g(x)|, and d3 (f, g) = 0 |f (x) − g(x)| dx. Prove that (C([0, 1]), d1 ) is
not complete but (C([0, 1], d2 ) is complete (this requires recalling some useful results from
elementary analysis that we will revisit in more generality in lecture 4). Since these met-
rics can be defined by norms, one choice of norm produces a Banach space, while the
other does not (this happens on infinite dimensional spaces where norms are not always
equivalent). If we use d3 is the resulting metric space complete?

Problem 14 Prove that a discrete metric space X is separable if and only if X is count-
able. (A discrete metric space is any set X with the discrete metric d(x, y) = 0 if x = y
and d(x, y) = 1 if x 6= y.)

Problem 15 Prove that `∞ is not separable, but `p with 1 ≤ p < +∞ is separable.


Prove that `p with 1 ≤ p ≤ +∞ is complete.

Problem 16 Let E 0 be the set of all limit points of a set E ⊂ X where (X, d) is a
metric space. Prove that E 0 is closed. Prove that E and Ē have the same limit points.
Do E and E 0 always have the same limit points?
148
Problem 17 Let A1 , A2 , A3 , . . . be subsets of a metric space.
(a) If Bn = ∪ni=1 Ai , prove that B̄n = ∪ni=1 Āi for n = 1, 2, 3, . . ..
(b) If B = ∪∞ ∞
i=1 Ai , prove that ∪i=1 Āi ⊂ B̄. Show, by an example, that this inclusion can
be proper.
Problem 18 Is every point of every open set E ⊂ R2 a limit point of E? Answer the same
question for closed sets in R2 .

Problem 19 Let E o denote the set of all interior points of a set E.


(a) Prove that E o is always open.
(b) Prove that E is open if and only if E o = E.
(c) If G ⊂ E and G is open, prove that G ⊂ E o .
(d) Prove that the complement of E o is the closure of the complement of E.
(e) Do E and Ē always have the same interiors?
Problem 20 Show that Rk is separable.

Problem 21 Prove that every separable metric space has a countable base.

Problem 22 Prove that any uncountable set with the discrete metric is not second count-
able.

Problem 23 Show that a Schauder basis (fn )∞


n=0 of C([0, 1]) may be constructed from
”tent” functions.

Problem 24 Let E be the set of all x ∈ [0, 1] whose decimal expansion contains only
the digits 4 and 7. Is E countable? Is E dense in [0, 1]? Is E compact? Is E perfect?

Problem 25 Below, A and B are sets in metric space (X, d).


(a) If A and B are disjoint closed sets prove that they are separated.
(b) Fix p ∈ X, δ > 0, define A = {q ∈ X : d(p, q) < δ} and B = {q ∈ X : d(p, q) > δ}.
Prove that A and B are separated.
(c) Prove that every connected space with at least two points is uncountable.
Problem 26 Let A and B be separated subsets in Rn , suppose a ∈ A and b ∈ B and
define p(t) = (1 − t)a + tb for t ∈ R. Put A0 = p−1 (A) and B0 = p−1 (B) (thus t ∈ A0 if
and only if p(t) ∈ A).
(1) Prove that A0 and B0 are separated subsets of R.
(2) Prove that there exists t0 ∈ (0, 1) such that p(t0 ) ∈
/ A ∪ B.
(3) Prove that every convex subset of Rk is connected.
Problem 27 Give an example of matrix space that is separable but not second countable.
149
Problem 28 Any open set in R can be written as countable union of open intervals.

14.3. Compactness

Problem 1 Let f : A ⊂ R → R have the property that for every x ∈ A there is an  > 0
such that
f (t) >  if t ∈ (x − , x + ) ∩ A.
Show that if the set A is compact, then there is some c > 0 such that f (x) > c for all
x ∈ A.

Problem 2
Use compactness to show that if a function f : R → R is locally increasing at every
point in R, then f must be increasing. Here, locally increasing at a point x means there
exists δ > 0 such that f (s) < f (x) < f (t) whenever x − δ < s < x < t < x + δ.

Problem 3 Let (X, d) be a metric space. Let {K1 , K2 , . . . , Kn } be a set of compact


subsets of X. Prove that ∪ni=1 Ki is a compact subset of X. Let {Kα } be a set of compact
subsets of X. Prove that ∩α Kα is a compact subset of X.

Problem 4 Let (X, d) be a metric space with the discrete metric d. What subsets of
X are compact?

Problem 5 Describe an open cover of the open unit square {(x, y) : 0 < x < 1, 0 < y < 1}
that has no finite subcover.

Problem 6 Let (xn ) be a sequence in a metric space (X, d) that converges to a limit
x. Show the set {x1 , x2 , x3 , · · · } is compact.

Problem 7 Show that the product of two compact metric spaces with the product metric
is a compact metric space.

Problem 8 Let a, b ∈ R and a < b. Show that any closed ball in C([a, b]) is not compact
with the metric d(f, g) = supx∈[a,b] |f (x) − g(x)| for f, g ∈ C([a, b]).

Problem 9 Prove that every compact metric space K has a countable base, and that
K is therefore separable.

Problem 10 Prove that if {Kα } is a collection of compact subsets of a metric space


X such that the intersection of every finite subcollection of {Kα } is nonempty, then ∩Kα
is nonempty. (Hint: Try a proof by contradiction by assuming there is some member of
{Kα }, say K1 , such that no point in K1 belongs to every Kα , then create an open cover
150
of K1 using complements of Kα . Show that this becomes false (in R, for example) if the
word “compact” is replaced by either “closed” or by “bounded.”

Problem 11 Let E be the set of all x ∈ [0, 1] whose decimal expansion contains only
the digits 4 and 7. Is E compact?

Problem 12 Let X be a metric space in which every infinite subset has a limit point.
Prove that X is separable and compact.

Problem 13

14.4. Continuity
Problem 1 Let X and Y be metric spaces and A ⊂ X. Suppose f maps A into Y and p
is a limit point of A. We say that f has a limit at p and write f (x) → q as x → p, or
limx→p f (x) = q if there is a point q ∈ Y with the property that for every  > 0 there
exists a δ > 0 such that for all points x ∈ E with 0 < dX (x, p) < δ implies dY (f (x), q) < .
Prove the following theorems.
(a) limx→p f (x) = q if and only if limn→∞ f (pn ) = q for every sequence (pn ) ⊂ E such
that pn 6= p for all n and pn → p.
(b) If f has a limit at p, then the limit is unique.
(c) The function f is continuous at p if and only if limx→p f (x) = f (p).
Problem 2 Let f : R2 → R be defined

 x421 x22 , (x , x ) 6= (0, 0),
1 2
f (x1 , x2 ) = x1 +x2
0, (x1 , x2 ) = (0, 0).

Show that limx→0 f (x, mx) = 0 for every m ∈ R but f is discontinuous at (0, 0). Interpret
this result.

Problem 3 Let C 1 ([a, b]) be the metric space of continuously differentiable functions on
[a, b] with metric

d(f, g) = sup |f (x) − g(x)| + sup |f 0 (x) − g 0 (x)| .


a≤x≤b a≤x≤b

Let D : C 1 ([a, b]) → C([a, b]) be defined by D(f ) = f 0 . Prove that D is continuous.

Problem 4 Let C([a, b]) be the metric space of continuous functions on [a, b] with metric
Z b
d(f, g) = |f (t) − g(t)| dt.
a

Define T : C([a, b]) → R by


Z b
T (f ) = f (t) dt.
a
151
(a) Prove that T is continuous using sequential definition of continuity.
(b) Prove that T is continuous using δ −  definition.

Problem 5 Let f and g be continuous maps of a metric space X into a metric space Y
and let E be a dense subset of X.

(a) Prove that f (E) is dense in f (X).


(b) If g(x) = f (x) for all x in a dense subset E of X, prove that g(x) = f (x) for all
x ∈ X.

Problem 6 Let A be a set and x a point in a metric space (X, d). Define the “distance
from x to A” to be
d(x, A) = inf d(x, y).
y∈A

(a) Show that for fixed A, the function f : X → R (with the usual metric on R) given by
f (x) = d(x, A) is continuous. (b) Show {x : d(x, A) = 0} = Ā.

Problem 7 Suppose f is a mapping from metric space X to metric space Y . Prove


that a function is continuous if and only if f −1 (A) is closed in X for every closed set
A⊂Y.

Problem 8 Assume f : R → R is continuous and define A = {x : f (x) = 0}. Show


A is closed.

Problem 9 Suppose f is a continuous bijection of a compact metric space X onto a


metric space Y . Prove that the inverse map f −1 defined on Y by f −1 (f (x)) = x (where
x ∈ X) is a continuous mapping of Y onto X.

Problem 10 Let (X, d) be a metric space and F, G ⊂ X with F closed, G open, and
F ⊂ G. Show that there is a continuous function f : X → R such that 0 ≤ f (x) ≤ 1,
f (x) = 1 for x ∈ F , and f (x) = 0 for x ∈ Gc .

Problem 11 Let f and g be continuous maps of a metric space X into a metric space
Y and let E be a dense subset of X. (a) Prove that f (E) is dense in f (X). (b) If
g(x) = f (x) for all x in a dense subset E of X, prove that g(x) = f (x) for all x ∈ X.
Explain why these results are important for our everyday experience with functions and
numbers.

Problem 12 (a) Show that the requirement in the definition of uniform continuity can be
rephrased as follows, in terms of diameters of sets: For every  > 0 there exists a δ > 0
such that diam(f (E)) <  for all E ⊂ X with diam(E) < δ. (b) Let E be a dense subset
of a metric space X, and let f be a uniformly continuous function into a complete metric
space Y defined on E. Prove that f has a continuous extension from E to X in two ways
152
using part (a) and then using Cauchy sequences.

Problem 13 Let f : R → R be a continuous function that is periodic with period T > 0.


(a) Show that f has a finite maximum and minimum values on R. (b) Show that f is
uniformly continuous on R.

Problem 14 Let X be a metric space.


(a) Call two Cauchy sequences (pn ), (qn ) in X equivalent if

lim d(pn , qn ) = 0.
n→∞

Prove that this is an equivalence relation.


(b) Let X ∗ be the set of all equivalence class so obtained. If P ∈ X̃ and Q ∈ X̃, (pn ) ∈ P
and (qn ) ∈ Q, define
∆(P, Q) = lim d(pn , qn ).
n→∞
Note that by an exercise from lecture 1 that this limit exists. Show that the number
∆(P, Q) is unchanged if (pn ) and (qn ) are replaced by equivalent sequences, and hence
that ∆ is a distance function in X̃.
(c) Prove that the resulting metric space X̃ is complete.
(d) For each p ∈ X, there is a Cauchy sequence all of whose terms are p; let Pp be the
element of X̃ which contains this sequence. Prove that ∆(Pp , Pq ) = d(p, q) for all
p, q ∈ X. In other words, the mapping φ defined by φ(p) = Pp is an isometry of X
into X̃.
(e) Prove that φ(X) is dense in X, and that φ(X) = X̃ if X is complete. By part (d),
we may identify X and φ(X) and thus regard X as embedded in the complete space
X̃. We call X̃ the completion of X.
Problem 15 Let (X, d) be a metric space and suppose f is uniformly continuous on the
compact subsets E1 , . . . , En of X. (a) Prove that f is uniformly continuous on ∪ni=1 Ei .
(b) Give an example of a function f that is continuous on R and a sequence of compact
intervals E1 , E2 , E3 , . . . on each of which f is uniformly continuous, yet f is not uniformly
continuous on ∪∞ i=1 Ei .

Problem 16 Prove that if f is a continuous mapping of a metric space X into a met-


ric space Y and E ⊂ X is connected, then f (E) is connected.

14.5. Sequence of Functions


x
Problem 1 Prove the sequence ( n+x 2 ) converges uniformly on R with the usual metric.

Problem 2 (a) Define what it means for a sequence of functions to be uniformly Cauchy
and prove that a uniformly convergent sequence of functions is uniformly Cauchy. (b)
Prove that every uniformly convergent sequence of bounded functions is uniformly bounded.
153
Here, we say that a sequence of real-valued functions (fn ) is uniformly bounded if there
exists M > 0 such that kfn k∞ ≤ M . (c) Show there exists a sequence of functions that
converges pointwise to the function that is identically 0, but each function in the sequence
is unbounded Hint: Consider the sequence {x/n} on [0, ∞).
Problem 3 (a) Prove that if the sequences of real-valued functions (fn ) and (gn ) converge
uniformly on A ⊂ X, then (fn + gn ) converges uniformly on A. (b) If fn and gn are also
bounded for each n ∈ N, prove that (fn gn ) converges uniformly on A. (c) Show that
the boundedness is necessary in (b) by constructing an example where (fn gn ) does not
converge uniformly (although it must converge pointwise).
Problem 4 Let (X, dX ) and (Y, dY ) be metric spaces, A ⊂ X, and (fn ) a sequence of
continuous functions from A to Y that converges to a function f : A → Y uniformly on
A. Given any x ∈ A, prove that limn→∞ fn (xn ) = f (x) for every sequence (xn ) in A with
xn → x. Construct an example showing the converse is not true, but if A is compact,
then the converse is true.

Problem 5 (General Arzelà Ascoli Theorem) If X is a compact metric space, (fn ) ⊂ C(K),
(fn ) is pointwise bounded and equicontinuous on X, prove that

(a) (fn ) is uniformly bounded on X, and


(b) f (n) contains a uniformly convergent subsequence.

Problem 6 Let (X, d) be a metric space and suppose K ⊂ X is compact, fn → f on K


(and all functions are real-valued and continuous), and fn (x) ≥ fn+1 (x) for all x ∈ K and
n ∈ N. Prove that fn → f uniformly on K. Construct an example showing the necessity
of compactness.

 n

Problem 7 Is the set of functions x2 + 1+n
x n=1
taking R to R (with the usual metric)
equicontinuous on [0, 1]?

Problem 8 Compute and plot the Bernstein polynomials for 1/(1 + x) on [0, 1] of de-
grees 1, 2, and 3.

Problem 9 Assume that (fn ) is a sequence of functions with fn : [a, b] → R for all n
where we take the usual metric on [a, b] and R.

(a) Assume that {fn } is uniformly Lipschitz continuous on [a, b] with constant L. Prove
that {fn } is equicontinuous on [a, b].
(b) Assume that {fn } is uniformly Lipschitz continuous on [a, b] with constant L and
further that (fn (x)) is a uniformly bounded sequence of numbers for some a ≤ x ≤ b.
(1) Prove that (fn ) has a subsequence that converges uniformly on [a, b]. (2) Prove
the second assumption is needed with an example.
154
(c) Assume that (fn ) is a bounded subset of C([a, b]). Show that the sequence of functions
(Fn ) from [a, b] into R defined by
Z x
Fn (x) = fn (x) ds
a

contains a subsequence that converges uniformly on [a, b]. (This is quite a remark-
able result when considering the sequence of functions (sin(nx)) as an example of a
bounded sequence in C([a, b]) that fails to have any convergent subsequence.)

Problem 10 Suppose K = {x1 , . . . , xn } is a finite space with the discrete metric d. Show
C(K) is linearly isomorphic to the finite-dimensional space Rn with the ∞-norm (which
is just the maximum norm in finite dimensions).

Problem 11 Suppose f is a real-valued continuous function with dom(f ) = R. Let


fn (x) = f (nx) for n ∈ N and any x ∈ R. If {fn } is equicontinuous, prove that f must be
a constant function.

Problem 12 Suppose that (fn ) is an equicontinuous sequence of functions on a com-


pact set K and (fn ) converges pointwise on K. Prove that (fn ) converges uniformly on
K.

Problem 13 Let X and Y be two metric spaces and A ⊂ X. Let (fn ) be a sequence
of functions from A to Y that converges to f pointwise on A. Prove the following theo-
rems.

(a) Let Mn = supx∈A dY (fn (x), f (x)). Prove that fn → f uniformly if and only if Mn →
0.
(b) If fn is continuous on A for each n, then f is continuous on A. This is a generalization
of the theorem in the notes.
(c) Suppose Y is complete, then fn converges uniformly on A if and only if for every  > 0
there exists an integer N such that m ≥ N , n ≥ N , x ∈ E implies dY (fn (x), fm (x)) <
. Construct an example showing the necessity of Y being complete.
(d) Suppose Y is complete, and fn converges uniformly on A. Let x be a limit point of A
and suppose that limt→x fn (t) = yn for each n. Then (yn ) converges and limt→x f (t) =
limn→∞ yn . In other words, prove limt→x limn→∞ fn (t) = limn→∞ limt→x fn (t). Con-
struct an example showing the necessity of Y being complete.
R1
Problem 14 Prove that if f ∈ C([0, 1]) and 0 f (x)xn dx = 0 for all n ∈ N, then f (x) = 0
R1
on [0, 1]. Hint: If 0 f 2 (x) dx = 0, then a result from elementary real analysis states that
f ≡ 0. Consider a sequence of polynomials that converge uniformly to f and take the
product with f to get a uniformly convergent sequence to f 2 .

Problem 15 Let (X, d) be a metric space and pick any fixed but arbitrary a ∈ X. Define
155
the real-valued function fp for each p ∈ X by

fp (x) = d(x, p) − d(x, a),

where x ∈ X.

(a) Prove that kfp k∞ ≤ d(a, p) and fp ∈ C(X).


(b) Prove that
kfp − fq k∞ = d(p, q)
for all p, q ∈ X.
(c) Let Φ(p) = f (p), so that Φ is an isometry of X onto Φ(X) ⊂ C(X). Let Y be the
closure of Φ(X) in C(X). Show that Y is complete which implies X is isometric to a
dense subset of a complete metric space Y . Note: This is a different proof of what we
previously showed in lecture 3 examples about defining the closure of a metric space.

14.6. Differentiation

Problem 1 Define the discontinuous real-valued function f : R2 → R by



 xy , if(x, y) 6= (0, 0),
2 2
f (x, y) = x +y
0, else.

prove that (the Gateaux derivatives) Dx f (x, y) and Dy f (x, y) exist everywhere in R2 even
though f is discontinuous at (0, 0).

Problem 2 Suppose that f : A ⊂ Rn → R where A is an open set and the partial


derivatives D1 f, . . . , Dn f are bounded in A. Prove that f is continuous in A.

Problem 3 Suppose that f : A ⊂ Rn → R is differentiable, where A is an open set


and that f has a local maximum at a point x ∈ A. Prove f 0 (x) = 0.

Problem 4 Suppose that f : A ⊂ Rn → R is differentiable, where A is an open connected


set and f 0 (x) = 0 for every x ∈ A. Prove that f is constant in A.

Problem 5 Let f : R2 → R2 be defined by


! !
f1 (x, y) ex cos y
f (x, y) = = .
f2 (x, y) ex sin y
(a) Show that the range of f is R2 \ {(0, 0)}.
(b) Show that the Jacobian of f is non-zero on R2 . Thus, every point of R2 has a
neighborhood in which f is 1-1, but f is not 1-1 on R2 .
156
(c) Let (x̄, ȳ) = (0, π/3) and (ū, v̄) = f (x̄, ȳ). Let g = f −1 denote the continuous inverse
of f defined in a neighborhood of (ū, v̄) such that g(ū, v̄) = (x̄, ȳ). Find an explicit
formula for g, compute f 0 (x̄, ȳ), g 0 (ū, v̄) and show that g 0 (f (x))f 0 (x) = IR2 .
Problem 6 If X = Y = Z = R in the implicit function theorem, interpret the theorem
and its proof graphically.

Problem 7 Define f : R3 → R by f (x, y, z) = x2 y + ex + z. Show that f (0, 1, −1) = 0,


Dx f (0, 1, −1) 6= 0, and that there exists a differentiable function g in some neighborhood
of (1, −1) in R2 such that g(1, −1) = 0 and f (g(y, z), y, z) = 0. Find Dy g(1, −1) and
Dz g(1, −1).

Rb
Problem 8 Suppose f ≥ 0, f is continuous on [a, b], and a
f (x) dx = 0. Prove that
f (x) = 0 for all x ∈ [a, b].

Problem 9 Let p and q be positive real numbers such that 1/p + 1/q = 1. Prove the
following statements.
(a) If u ≥ 0 and v ≥ 0, then
up v q
uv ≤ + ,
p q
p q
and equality holds if and only if u = v .
Rb Rb Rb
(b) If f, g ∈ R([a, b]), f, g ≥ 0, and a f p = 1 = a g q , then a f g ≤ 1.
(c) If f, g, ∈ R([a, b]), then
Z b Z b 1/p Z b 1/q
p q


f g ≤
|f | |g| .
a a a

157

You might also like